Teraz jest 28 mar 2024, o 12:36

Strefa czasowa: UTC + 2 [ DST ]




Utwórz nowy wątek Odpowiedz w wątku  [ Posty: 11 ]  Przejdź na stronę 1, 2  Następna strona
Autor Wiadomość
PostNapisane: 3 lut 2010, o 21:15 
Offline
Général de brigade
Général de brigade
Avatar użytkownika

Dołączył(a): 3 lut 2010, o 06:45
Posty: 1248
Temat rozpoczęty przez użytkownika Unclean 26.04.2005

Pozycja obowiązkowa dla wszystkich, którzy nie chcą aby ich pytanie zostało skwitowane krótkim - "było już" :) Oraz dla wszystkich miłośników epoki chcących dowiedzieć się czegoś nowego bądź z sentymentem spojrzeć na przeszłe dzieje

_________________
"W polityce głupota nie stanowi przeszkody."
Napoleon


Ostatnio edytowano 3 lut 2010, o 21:53 przez Louis Nicolas, łącznie edytowano 1 raz

Góra
 Zobacz profil  
 
PostNapisane: 3 lut 2010, o 21:15 
Offline
Général de brigade
Général de brigade
Avatar użytkownika

Dołączył(a): 3 lut 2010, o 06:45
Posty: 1248
Post użytkownika Unclean

PYTANIA O KAMPANIE, WOJNY, BITWY i WOJSKA.


1. Czym "wsławiła się" X kohorta gwardii narodowej ? - Tym, że była główną siłą "napędową" spisku Malleta z 24 X 1812 r.

2. Jeden z regimentów rosyjskiej piechoty został uhonorowany przez cara Aleksandra - za męstwo w boju - w ten sposób, że zachował swoje tradycyjne nakrycia głowy podczas gdy w pozostałych regimanetach wprowadzono nowe czaka. Jaki to regiment i gdzie wykazał się męstwem ? - Pawłowski Pułk Grenadierów, za zasługi pod Frydlandem

3. Pierwsza jednostka gwardii cesarskiej poddała się po przegranej bitwie pod .. ? - batalion marynarzy gwardii, którzy skapitulowali wraz z korpusem Duponta pod Baylen w 1808 r.

4. Paweł Jerzmanowski w Lettre a M. de Lamartine en refutant ses assertions napisał: "Kiedy Duńczycy porzucili sprawę cesarza, regiment kawalerii polskiej pod wodzą pułkownika Brzechwy znajdował się w ich korpusie. Nie mogąc pociągnąć go do zdrady, chcieli go rozbić. Ale dumna postawa tego regimentu narzuciła taki szacunek i podziw Duńczyków, że nie odważyli się przeciwstawić jego połączeniu z marszałkiem Davoutem". Który z pułków jazdy polskiej wzbudził tak wielki respekt Duńczyków? - 17 litewski pułk ułanów

5. W kampanii 1806 roku tylko jeden pułk kawalerii pruskiej był wyposażony w lance. Który był to pułk ? - Pułk Bośniaków -regiment powstał z przekształcenia 10 pułku huzarów i nosił tradycyjną nazwę wyborowej kawalerii Fryderyka Wielkiego...(pełniła często straż na podczas obozów strzegąc armie przed dezercją)

6. Pod jaką miejscowością doszło do pierwszej bitwy francusko - hiszpańskiej w wojnie na Półwyspie. Dla ułatwienia dodam, że chodzi o czerwiec 1808 r - 6 czerwca przy moście Alcolea w pobliżu Kordowy Dupont pogonił wojska Castanosa

7. Ile salw oddali hiszpańscy kanonierzy podczas szarży naszych szwoleżerów pod Somosierra- 5

8. W którym roku powstał Legion Irlandzki w służbie Napoleona ? - Pierwsze bataliony zaczęto formować na mocy dekretu Pierwszego Konsula z października 1803 r.

9. Jakie miejscowości opuścili Francuzi jako ostatnie w 1814 r. i kiedy to miało miejsce? - Miasta Figueras i Hostalrich zostały ewakuowane przez Francuzów dopiero 4 czerwca 1814 r.

10. Gdzie i kiedy została rozegrana bitwa, która otworzyła drogę przez Alpy armii Eugeniusza de Beauharnais w trakcie kampanii 1809 ? - Malborghetto z 16 maja 1809 r.

11. Kto przejął dowodzenie nad 9 półbrygadą lekkiej piechoty po śmierci gen. Desaix podczas ataku pod Marengo? - Gen. Jean Boudet

12. Jakie pułki zaliczano do ciężkiej jazdy gwardii rosyjskiej? - kawalergardów i konny lejbgwardii

13. W której bitwie miała miejsce "szarża 80 szwadronów" ? - W bitwie pod Pruska Iławą (8 lutego 1807 r.)

14. Gdzie i kiedy weszły do walki oddziały dywizji piechoty Księstwa Warszawskiego wysłane do Hiszpanii ? – w grudniu 1808 r. pod Almaraz

15. W 1810 r. ułani nadwislańscy przeprowadzili szarżę w pewnym wąwozie hiszpańskim. Ich wyczyn porównywano czasami z atakiem szwoleżerów pod Somosierrą. O jaki wąwóz chodzi? - San Estevan lub Santi Estevan

16. Na jesieni 1806 r. Cesarz powołał do istnienia pewną jednostkę Gwardii, która od początku była solą w oku dla zwykłych żołnierzy (ze względu na charakter rekrutacji). Toteż po kilkunastu miesiącach ją rozwiązał. Co to była za jednostka? - Żandarmeria ordynansowa

17. Które regimenty gwardii francuskiej, podczas kampanii 1815 roku, nazywano mylnie średnia gwardią, do jakiej gwardii je oficjalnie zaliczano, a także co było powodem nadania tej nazwy - 3e (Middle Guard) Grenadiers Régiment, Général de Brigade Morvan, 1er Bn./3e Grenadiers, 2e Bn./3e Grenadiers, 4e (Middle Guard) Grenadiers Régiment, Général de Brigade Harlet, 1er Bn./4e Grenadiers
oraz:3e (Middle Guard) Chasseurs Régiment Colonel Mallet, 1er Bn./3e Chasseurs
2e Bn./3e Chasseurs, 4e (Middle Guard) Chasseurs Régiment, Général de Brigade Henrion, 1er Bn./4e Chasseurs, 2e Bn./4e Chasseurs - większość gwardzistów z "średnich" pułków była rekrutami z pułków lekkich (czyli niskimi wzrostem), mająca za sobą przegrane kampanie 1812,1813,1814 co w opinii armii nie stawiało ich na równi z weteranami z lat 1805-1809.

18. W jakiej armii (oprócz rosyjskiej) istniała w okresie napoleońskim regularna (nie ochotnicza!) formacja kozaków? - Chodzi tu o armię pruską i tzw. Guard Kosaken Eskadron

19. Kto i gdzie poprowadził ostatnią polską szarżę podczas kampanii 1814 roku - Saint-Dizier 26 marca - szarża szwoleżerów

20. Ilu polskich generałów poległo podczas kampanii 1812 roku i w jakich okolicznościach nastąpiła ich śmierć? - W kampanii rosyjskiej zginęło 4 generałów:1) Stanisław Fiszer - pod Winkowem 18 X 2) Michał Grabowski - 17 VIII pod Smoleńskiem 3) Michał Ignacy Kamieński - zmarł w niewoli pod koniec roku 4) Czesław Pakosz - zmarł z ran po przeprawie berezyńskiej

21. Gdzie i kiedy Brytyjczycy zdobyli pierwsze Orły francuskich pułków ? – Pod Trafalgarem 21 X 1805 r. Wpadł wówczas w ich łapy orzeł 93 regimentu liniowego

22. Przed kampanią 1815 r. późniejszy IV korpus Gerarda stanowił samodzielny związek operacyjny, nie zaliczany do składu Armii Północy. Jaka była jego nazwa? - Armia Mozeli

23. Pięciuset, albo według innych źródeł siedmiuset, rekrutów z Wielkiego Księstwa Litewskiego zostało włączonych do dwóch niepolskich i nielitewskich pułków Wielkiej Armii. Wymieńcie jeden z nich. - 129 pułk piechoty liniowej i 2 ilyryjski pułk piechoty

24. Pod koniec XIX wieku Turcja zaczęła reformować swoje przestarzałe formacje wojskowe, tworząc oddziały wzorowane na europejskich i szkolone przez instruktorów francuskich. Powstała tzw. Armia Nowego Wzoru - to właśnie te oddziały broniły Akry w 1799. Jak brzmiała turecka nazwa tych oddziałów ? - Nizam-i Jedid
25. prosze wymienić dowódców korpusów Armii Hiszpanii, z którą Napoleon w listopadzie 1808 r. zaczął swoją kampanię na Płw. Iberyjskim? - I korpus - MdE Victor; II korpus - MdE Soult; III korpus - MdE Moncey; IV korpus - MdE Lefebrve, potem gen. Sebastiani; VI korpus - MdE Ney; VII korpus - gen. St. Cyr; VIII korpus Junot.
26. Kto z oficerow V korpusu stracil zycie pod Smolenskiem? Ograniczmy sie do stopnia od majora w gore. - general bryg. Grabowski; pulkownik Zakrzewski; pulkownik Zawadzki; podpułkownik Gaward; mjr Dembicki; mjr Polkowski
27. Podczas wojny 1806-1807 król Prus zezwolił na tworzenie oddziałów ochotniczych (tzw. freikorpsów) do walki z Francuzami. Ile ich było (wystarczy podać 5), kto nimi dowodził i w jakich rejonach walczyły? -freikorps Schilla - Pomorze, Kołobrzeg
freikorps Moellendorffa - działał w okolicach Królewca
freikorps Ravena (może być znany także jako freikorps Alvenslebena lub Borstella) - działał j.w.
freikorps Wilamowitza - Pomorze, następnie Gdańsk
freikorps Krockowa - j.w.
freikorps Marwitza - Pomorze Szwedzkie
freikorps Meyerna - Prusy Wschodnie, okolice Tylży
freikorps Kalckreutha – Gdańsk
28. Od udziału w jakich bitwach, dwaj słynni dowódcy kawalerii, Jan Konopka i Aleksander Rożniecki, zaczynali swą wojenną karierę? - Swój chrzest bojowy towarzysz Jan Konopka przeszedł w szeregach 1 Brygady Ukraińskiej w dniu 18 VI 1792r. pod Zieleńcami.
Pod Dubienką 18 VII 1792r. walczył mjr. Aleksander Rożniecki. Był w składzie 2 Brygady Wielkopolskiej.
29. Jakie polskie oddziały oprócz ułanów nadwiślańskich walczyły pod Albuherą? - były to dwie kompanie grenadierów z 4 p.p. Księstwa, dowodzone prawdopodobnie przez Ludwika Babskiego i Franciszka Brochockiego. Znajdowali się w składzie kombinowanego oddziału płk. Vare, który nie brał bezpośredniego udziału w walce, ale poniósł straty od ognia artylerii.
30. Pierwsze pododdziały (3 kompanie) szwoleżerów znalazły się w Hiszpanii już w marcu 1808 r. w składzie kombinowanego pułku marszowego kawalerii Gwardii. Kto dowodził tymi kompaniami? - Dnia 4 marca rzekę przekroczyły kompanie 1,5 i 2. Pierwsze dwie istotnie składały 1 szwadron Tomasza Łubieńskiego, który też sprawował dowództwo nad całością oddziału. Kompaniami tymi dowodzili kapitanowie: Franciszek Łubieński i Stanisław Gorayski. Wobec nieobecności tego drugiego kompanię prowadził por. Józef Załuski. Kompanią 2, która istotnie należała do 2 szwadronu Kozietulskiego dowodził Jan Paweł Jerzmanowski.
31. Pewien polski magnat w 1793 r. zawarł z pewnym europejskim państwem umowę na stworzenie pułku ułanów w celu walki z wojskami rewolucyjnej Francji. Pułk powstał i wsławił się tak walecznością jak i brutalnością wobec ludności cywilnej Holandii. I choć ułani nosili bardzo polskie mundury, to mimo zamierzeń prawdopodobnie nie było w nim Polaków. Jak nazywał się ów magnat, pułk oraz państwo któremu służył? - Chodzi o pułk Ułanów Brytyjskich, a ich założycielem był książę Jerzy Marcin Lubomirski
32. Wiadomo, że na Wołyniu znalazł schronienie korpus ks. Conde. Przechodził potem różne koleje i w rezultacie jego oficerowie po 1800 r. zostali częściowo "wchłonięci" przez armię rosyjską. Jednak wcześniej niektóre oddziały należące do tego, sformowanego na Wołyniu korpusu wzięły udział w walkach ze swymi rodakami. Kiedy i gdzie (nie chodzi tylko o kraj, ale o miejscowość). Podkreślam, że chodzi o oddziały zreformowane w Rosji. - ks. Condee na czele swych oddziałów opuścił Włodzimierz wołyński 2 lipca 1799 r. Poprzez Austrię i Bawarię dotarli w październiku do miasta Konstancji (u ujścia Renu do jeziora Bodeńskiego). 7 października rojaliści starli się tam z dywizją Gazana będącą częścią Armii Helwecji. Rojaliści stracili ponad 200 ludzi (podobno stawali dzielnie) i zostali po przybyciu Suworowa przeniesieni do Landsbergu w Bawarii, a na leże zimowe do Linzu w Austrii. Potem Paweł nie bardzo wiedział co z tym "fantem" zrobić. Korpus przeszedł na żołd angielski i został przewieziony najpierw do Włoch a potem do Austrii i stopniowo się "rozpływał" przez choroby i dezercje Po Luneville reszta albo przyjęła służbę w armii angielskiej, (niektórzy rosyjskiej), a niektórzy wrócili do Francji.
33. Na Litwie w 1812 r. doszło do kilkunastu wystąpień miejscowej ludności przeciwko wojskom rosyjskim. Podczas jednego z nich powstańcy opanowali pewną miejscowość kilkakrotnie odpierając ataki Rosjan, wzięli 80 jeńców, zagarnęli wojskową kasę i magazyny oraz liczne wozy z amunicją i tabor rosyjskiego pułku ułańskiego. Pytanie brzmi - gdzie to było? – Pińsk
34. Proszę wymienić bitwy stoczone przez Francuzów z Sardyńczykami w kampanii 1796 r. - 17 IV Ceva; 19 IV La Bicocca; 21 IV Mondovi.
35. Wojsko Księstwa Warszawskiego w 1807 roku pod względem organizacyjnym dzieliło się na trzy zupełnie samodzielne "legie" czyli dywizje.
Jakie miały nazwy i kto nimi dowodził ? - 1 dywizja (Warszawska)
dowódca gen.dyw. ks. Józef Poniatowski; 2 dywzja (Kaliska) dowódca gen.dyw. Józef Zajączek; 3 dywizja (Poznańska) dowódca gen.dyw. Jan Henryk Dąbrowski.
36. Jakie jednostki (w rozumieniu typy broni) francuskie używały w okresie napoleońskim własnych formacji saperskich? - Saperów miały: piechota liniowa i lekka, dragoni, huzarzy oraz oczywiście wojska inżynieryjne.
37. W 1805 roku, na froncie włoskim, miała miejsce bitwa, w której polski batalion odparł dwie szarże kirasjerów austriackich, a potem zmusił 700 z nich, do złożenia broni. Kto dowodził tym batalionem, jaka to bitwa? - Chłopicki pod Castel Franco - było to 24 XI, a wojskami austriackimi dowodził Książe de Rohan.
38. Jaka francuska jednostka wspierała Polaków pod Fuengirolą i ile liczyła sobie żołnierzy? - 82 dragonów z 21 pułku
39. W kampanii 1815 czynny udział z polskich jednostek (składających się z Polaków) wziął Szwadron Elby oraz...no właśnie. Kto(co), jak, i gzie? - pierwszy batalion trzeciego pułku cudzoziemskiego (pułkownika Gołaszewskiego) i lansjerzy Adama Huppego bronią mostu pod Sevres w dniu 3 lipca 1815 roku.
40. W okresie napoleońskim, a przede wszystkim w czasach I Republiki istniało wiele oddziałów, głównie ochotniczych, które z rzymska nazywano legiami lub legionami. Dla nas najważniejsze są oczywiście legiony Dąbrowskiego i Kniaziewicza, ale było także wiele innych. Proszę wymienić przynajmniej trzy niepolskie i niefrancuskie legiony na służbie Francji. - Legion Germański-Powołany 4 września 1792 przez barona Anacharisa Clootza.Skład: rekruci z zachodnioniemieckich państewek, Prusacy, Holendrzy, oraz troche Francuzów(z alzacji). Służył w nich Augerau.
Legion Liege- Powołany1 maj 1792
Skład: Belgowie.
LegionBelgijski- Powołany1 maj 1792
Skład: Belgowie.
Wolny Legion Cudzoziemski-Powołany 1 sierpień 1792. organizowany przez Daedelsa, De Wintera, Chassego.
Skład: Holendrzy
Legion Batawski-Powołany 5 marzec 1793
Skład: Holendrzy
Drugi Legion Batawski-Powołany 5 marzec 1793
Skład: Holendrzy
Legion Vadoise(Korpus Strzelców Aosty)
Skład: mieszkańcy włoskiej Aosty.
1 Legion Piemoncki-Powołany 18 maj 1803
Skład: Piemontczycy.
Legion Lombardzki.
Skład: Włosi z Republiki Cisalpińskiej.
Legion Cispadańskiego.
Skład: Włosi z Republiki Cisalpińskiej.
Legon Włoski.
Skład: Włosi z Republiki Cisalpińskiej

41. Na jakich warunkach kapitulował Zamość? -"Przez wzgląd na piękną obronę, którą zrobił garnizon, oficerowie zatrzymają szpady, konie, własności i ekwipaże swoje, podoficerowie i żołnierze tornistry, ubiór i wszelkie własności" Garnizon pozostał w Księstwie, nie został odesłany w głąb Rosji, chętnych zwolniono na słowo honoru.
42. Jakie oddziały stanowiły eskortę ks. Józefa Poniatowskiego podczas bitwy lipskiej? -krakusi i kirasjerzy (polscy)
43. Proszę wymienić wszystkie pułki kawalerii zaliczane do gwardii Napoleona które miały "ułańską" formę - 1 (polski), 2 (holenderski) i 3 (litewski) pułki szwoleżerów, 1 (w korpusie grenadierów konnych), 2 (w korpusie dragonów) i 3 (w korpusie szwoleżerów) pułki eklererów. W lance uzbrojony był też szwadron Tatarów litewskich, najpierw włączony w skład 3, a w marcu 1813 r. - 1 pułku szwoleżerów. Oraz lansjerzy W. Księcia Bergu.
44. do którego z korpusów gwardii były zaliczane poszczególne pułki Gwardii Honorowej (z którymi pułkami jazdy były "powiązane"). Ponieważ kwestia numeracji regimentów Gwardii Honorowej też budzi wątpliwości, przyjmijmy za większością opracowań, że 1 pułkiem był ten, który organizował się w Wersalu, 2 - w Metz, 3 pułk w Tours, a 4 w Lyonie. - 1 p. - przy grenadierach konnych; 2 p. - przy strzelcach konnych; 3 p. - przy dragonach; 4 p. - przy szwoleżerach polskich
45. Jaka była pierwsza wojna w jakiej brał udział Chłopicki? - Chłopicki walczył pod Oczakowem w wojnie Rosyjsko-Tureckiej (1787 – 1792)
46. Kim byli "przyboczni łucznicy gwardyjscy" i jaki był związek tej formacji z Polską? - W 1781 cesarz Józef II utworzył złożoną z Polaków i odzianą na polską modłę "królewską gwardię szlachecką galicyjską" (inaczej "Nobelgarde") której szefem został ks. Adam Kazimierz Czartoryski. Kolejny cesarz - Leopold II na miejsce tej formacji powołał w 1791 r. oddzielny pluton galicyjski "przybocznych łuczników gwardyjskich" ("Arcieren-Leibgarde") który istniał aż do 1811 r.
47. Proszę o podanie dowódców obu stron w bitwie pod Maidą w 1806 roku. –Reynier; John Stuart
48. W lipcu 1812 jeden z pułków jazdy Księstwa Warszawskiego poniósł bardzo duże straty. Stało to się w wyniku bitwy pod miejscowością, o której nazwę chcę Szanownych Kolegów zapytać? W bitwie tej, oprócz w/w pułku brał jeszcze udział inny polski pułk jazdy. Który?
Dla utrudnienia pytania dodam, że miejscowość ta w XXw, jeszcze raz weszła do historii polskiej wojskowości, jako miejsce bitwy . Miała wtedy jednak inną nazwę. Jaką? - 1szy p. strz. konnych pod Romanowem (lipiec 1812) wspierany przez szwadron 12 p. ułanów. Romanów był też miejscem bitwy 12 X 1943, gdy nazywał się – Lenino.
49. Podczas kampanii 1812 r. pod Katyniem doszło do starcia ("żywej potyczki") pomiędzy Rosjanami a jednym z polskich pułków. Jaki to był pułk? - Był to szwadron Kozietulskiego oczywiście z 1 pułku Szwoleżerów Lansjerów Gwardii Cesarskiej.
50. W Księstwie Warszawskim istniały dwa pułki huzarów 10 i 13. Nie były jednak pierwsze. W 1807 roku powstawał inny pułk huzarów. Jak nazywał się jego dowódca i jakie były losy pułku? - no wiec ojcem tego pułku był ksiąze Jan Sułkowski. Niestety nic dobrego z tym pułkiem się nie wiąże jeno rozboje i rabunki. Ostatecznie wysłano go jesienią do Westfali i rozproszono po Legii Nadwiślańskiej, gwardii króla Hieronima i piechocie.
51. Proszę wymienić dowódców Napoleona i Francuzów pod Tulonem w 1793 r - Jean-Francois Cartreaux, gen. Dopet, gen. Dugommiera.
52. I-szy Pułk Szwoleżerów Gwardii podczas przeglądów przynajmniej dwukrotnie skompromitował się przed Napoleonem mieszając szyki. Za sprawką , których wyższych oficerów do tego doszło? - Delaitre komenderował słabym i piskliwym głosem i Krasiński pomylił komendy nakazujace formowanie szyku.
53. Proszę o krótkie opisanie manewru francuskiego pod Albuerą w 1811 r. Główne założenia. - Polegał on na uderzeniu z flanki (V korpus Girarda) na prawe skrzydło koalicjantów (stali tam Hiszpanie) i zrolowaniu go, przy jednoczesnym zaangażowaniu przeciwnika w jego centrum przez uderzenie na Albuherę, wykonane przez gen. Godinot.
54. Który, pułk WA wkroczył jako pierwszy do Moskwy w 1812 r.? - 10. regiment huzarów polskich płk. Jana Umińskiego.
55. 21 lipca odbyła się słynna bitwa którą nazwano pod "piramidami". Moje pytanie brzmi: w jakiej wiosce ona się odbyła i naprzeciw jakiego przedmieścia Kairu? – wioska Embabeh, przedmieście Boulacq.
56. W jakich miejscowościach koncentrowały się pułki Legii Nadwiślańskiej (1-3 oraz 4) po kampanii rosyjskiej? - Erfurt i Wittenberga
57. "Piekielna brygada" proszę podać jakie pułki ja stanowiły i kto był jej dowódcą ? - były to 5 i 7 pułki huzarów (kawaleria V korpusu Lannesa), dowodzone przez Ludwika Lasalle'a (1775-1809)
58. Z jakich jednostek składała się dywizja Lasalle'a w 1809 r.? - Dywizja lekkiej kawalerii w IV K; Brygada Pire: 8 p huz, 16 pstrzk; Brygada Bruyera: 13 i 24 pstrzk; Brygada Marulaza: 3, 14 i 23 pstrzk; Dragoni badeńscy Szwoleżerowie hescy.
59. W jakiej bitwie Józef Sułkowski został odznaczony orderem Virtuti Militari? - Za postawe pod Zelwą w 1792 lub/ i w bitwe pod Grannem która rozegrała się 22 i 23 lipca 1792.
60. Jaki pułk kirasjerów wielokrotnie odznaczył się w walkach pod dowództwem Sucheta w Hiszpanii w latach 1809-1812? - 13 pułk kirasjerów
61. W epoce napoleońskiej istniała formacja wojskowa złożona w dużej części z... intelektualistów (jakbyśmy dziś powiedzieli). Większość jej składu stanowili studenci, ale byli także poeci, kompozytorzy i malarze. Co to była za formacja, gdzie walczyła i kto był jej dowódcą? Dla ułatwienia dodam, że formacja ta dała początek barwom narodowym pewnego państwa. - korpus ochotniczy Adolfa Ludwika von Lutzowa (1782-1834), powstały we Wrocławiu w lutym 1813 r. Został rozbity w czerwcu pod Kitzen przez Francuzów i Polaków (brygada Krukowieckiego). A te barwy (czarny, czerwony, złoty) są kolorami flagi Niemiec.

_________________
"W polityce głupota nie stanowi przeszkody."
Napoleon


Ostatnio edytowano 3 lut 2010, o 21:37 przez Louis Nicolas, łącznie edytowano 6 razy

Góra
 Zobacz profil  
 
PostNapisane: 3 lut 2010, o 21:16 
Offline
Général de brigade
Général de brigade
Avatar użytkownika

Dołączył(a): 3 lut 2010, o 06:45
Posty: 1248
Posty użytkownika Unclean

PYTANIA o OSOBY.


1. Generał francuski, dowódca twierdzy, która poddała się jako ostatnia w kampanii 1815 roku (dopiero w grudniu) - ponieważ stracił nogę pod Wagram nazywany był "drewnianą nogą" i na propozycje kapitulacyjne w 1815 roku oświadczył, że odda twierdzę kiedy zwrócą mu nogę. Jak się nazywał ? - Pierre Daumesnil

2. Był tym generałem, który kapitulował bezpośrednio przed Daumesnilem (choc kilka miesięcy wcześniej - 26 VIII 1815 r.) Dowodził twierdzą na pograniczu francusko-szwajcarsko-niemieckim Huningue. Miał 135 ludzi, a oparł się kilkudziesięciu tysiącom Austriaków arcyksięcia Jana - gen. Joseph Barbanegre

3. Jak nazywał się oficer, który wprowadził w 1796 do uzbrojenia kawalerii brytyjskiej nowy lekki typ szabli ? - major Le Marchant

4. Jak powszechnie wiadomo Wielki Książę Bergu i Kliwii – Joachim Murat - dostał od księcia Poniatowskiego szablę Batorego. Jakim podarunkiem odwdzięczył się księciu Pepiemu? -"...przepyszny kaszkiet karmazynowy ułański, bogato złotem wyszywany (podobno własnoręcznie przez Karolinę Murat) oraz parę bogatych pistoletów.."

5. Najwyższy rangą Polak - oficer armii carskiej w kampanii 1812 to..? - Adam Czaplic

6. Kto objął naczelne dowództwo wojsk polskich po śmierci księcia Józefa Poniatowskiego? - General Sulkowski

7. Podczas obrony Portu Genua w 1800 roku, Gen. Massena wysłał jednego z oficerów z depesza do 1-go Konsula. Oficer ten przedarł się przez blokadę 28 kwietna i płynąć wpław z szabla w zębach dostarczyl meldunek do sztabu Napoleona. Następnie wrócił do oblężonego Portu Genua 27 maja. Kim był ten oficer? dla ułatwienia dodam ze w 1805 wkroczył do Austrii na czele 8 Pułku Huzarów, w kampanii hiszpańskiej był już generałem. - Jean Baptiste Franceschi-Delonne

8. Kto, na propozycję wstąpienia do armii Królestwa Polskiego, odpowiedział: "Kto służył pod Napoleonem, temu wolno służyć już tylko Panu Bogu do mszy"? - Józef Antoni Kossakowski

9. Kim była "Kawaleryjska Dziewica " ? - Nadieżda Durova

10. Jeden z najpiękniejszych bluffów militarnych doprowadził do zdobycia przez Francuzów ważnego mostu na Dunaju (w trakcie kampanii 1805 roku). Kim byli uczestnicy tego zdarzenia ? - Murat, Lannes i Bertrand. No i oczywiście batalion grenadierów ukrytych w krzakach

11. Jak miał na imię Mameluk, który trzykrotnie rzucał Cesarzowi do stóp zdobyte sztandary pod Austerlitz? Miał on niebywałego pecha, bowiem zginął w Madrycie w czasie Dos de Mayo w 1808 r. Był jedyna ofiarą Mameluków w czasie tych zajść. Nota bene był jednym z nielicznych "autentycznych" Mameluków, bowiem większość tej formacji gwardii to byli Francuzi. – Mustafa

12. Kolega szkolny Napoleona, w dużym stopniu przyczynił się do fiaska kampanii w Syrii - Antoine le Picard de Phelippeaux.

13. W 1807 roku gen. Jan Henryk Dąbrowski został ranny. Gdzie i kiedy (dzień i miesiąc) to się zdarzyło? - 23 lutego w walkach o Tczew.

14. Jak nazywali się generałowie, którzy polegli w "szarży 80 szwadronów" (Pruska Iława)? – gen. Hautpoul i gen. Nicolas Dahlmann

15. Jak nazywali sie dwaj generałowie, którzy razem z Malletem organizowali próbę zamachu z 24 października 1812 r.? - Guidal i Lahorie

16. Trzech braci miało główny wpływ na politykę Wielkiej Brytanii w Indiach na początku XIX wieku. Jak się nazywali ? - Bracia Wellesley: Richard, Arthur i Henry

17. Jak nazywał się dezerter z armii francuskiej, który przekazał sprzymierzonym plany kampanii Napoleona niedługo przed bitwą pod Waterloo? - generał Ludwik August Bourmont

18. Jak nazywała się druga żona Lucjana Bonaparte? - Alexandrina de Bleschamps

19. Było wiele zamachów(oczywiście nieudanych) na Napoleona. Jeden zdarzył się w trakcie rewii wojsk w Schonbrunnie (o z przeglosem). Jak nazywał się zamachowiec? - Fryderyk Staps, lat 19, student.

20. W 1813 roku pod Szymanowskim w 2 pułku piechoty liniowej służyło dwóch szefów batalionów. Proszę o podanie ich pełnych nazwisk, stopni jakie osiągnęli i okoliczności śmierci - Ludwik Bogusławski - generał, zmarł w 1840 w Warszawie,
Stanisław Trębicki -generał, zabity podczas Nocy Listopadowej

21. Christoph Duroc - zanim został wielkim marszałkiem pałacu, podkochiwał się w pewnej osobie z rodziny Napoleona. Mówiono nawet o małżeństwie. Kto był tą osobą? - Hortensja Beauharnais

22. Jak nazywał się znany oficer Księstwa Warszawskiego, który jednocześnie nosił order pruski Pour le Merite nadany mu za walkę z Napoleonem i Legię Honorową za walkę pod sztandarami tegoż Napoleona? - ppłk Józef Sowiński

23. Jak nazywał się generał francuski (od 1808 r.), prawa ręka Berthiera, jego oficjalny zastępca od 1809 r. Jeden z bardziej uzdolnionych sztabowców w wojsku francuskim, cały czas pozostawał w cieniu księcia Neufchatel, mimo iż wielokrotnie go zastępował, szczególnie w latach 1812-1814. - BAILLY de MONTHION

24. Kim był i jakie funkcje pełnił Florian Kobyliński? - w 1806 r. jako adiutant Dąbrowskiego sprowadził do Berlina na spotkanie z Napoleonem Wybickiego Płk Kobyliński był w 1812 r. w sztabie Davouta i zastępował szefa sztabu I korpusu gen. Romeufa. W 1815 wział dymisję w stopniu gen. bryg. Pełnił funkcje cywilne. W czasie powstania pomagał w organizacji Legii lit.-woł.

25. Kto poprowadził szarże pod St-Dizier? –Zygmunt Kurnatowski

26. Kto był pierwszym głównodowodzącym rosyjskiej Armii Naddunajskiej w trakcie wojny Rosyjsko -Tureckiej w 1810 roku ? – Bagration

27. Jak nazywał się generał, który poddał Santo Domingo w lipcu 1809 r., po bohaterskiej i heroicznej obronie? - gen. Barquier

28. Jak nazywał się oficer w wojskach Księstwa Warszawskiego, którego kariera o mały włos ległaby w gruzach poprzez przystąpienie w czasach I Rzeczpospolitej do Konfederacji Targowickiej. Osobnik ów był dość częstym gościem na salonach wszechwładnego ambasadora Katarzyny II - Igelstroma. Spowodowało to, iż zaocznie wydano na niego wyrok śmierci i skazano na infamię. Na szczęście wyroku nie udało się wykonać, a nasz bohater skutecznie się w latach następnych swymi czynami zrehabilitował. - Jan Henryk Dąbrowski

29. Kto przywiózł Napoleonowi do Rosji wieść o klęsce Raguzy pod Salamanką? - pułkownik Fabvier ze sztabu Marmonta

30. Jak nazywał się jeden z adiutantów Napoleona, który, w odróżnieniu od innych, nie zrobił wielkiej kariery wojskowej i zawsze pozostawał w cieniu?
Dla ułatwienia dodam, że był świadkiem Napoleona na ślubie z Józefiną w 1796 r., a w latach 1813-1814 dowodził obroną Magdeburga. - gen. Jean Lemarois

31. Jak nazywał się polski oficer, który towarzyszył Cesarzowi na Św. Helenie ? - Karol Piątkowski

32. W grudniu 1806 roku, urodził się pierwszy syn Napoleona. Jak miął na imię i kim była jego jego matka ? - Miał na imie Leon. Jego matką była Eleonora Denuelle de la Plaigne, lektorka Józefiny, podsunięta Cesarzowi przez Karolinę, aby wywołać rozdźwięki między Napoleonem i Cesarzową i doprowadzić do rozwodu.

33. Jak nazywał się kapitan francuskiego liniowca "Tonnant", bohater batalii abukirskiej, który po stracie obu nóg (lub jednej nogi i ramienia, wg innej wersji) kazał się osadzić w beczce trocin, aby nadal wydawać rozkazy?
Tam dosięgła go śmierć od angielskiej kuli. - kpt. Dupetit Thouars

34. Jak nazywał się pierwszy minister marynarki i kolonii Napoleona, mianowany przez niego wkrótce po zamachu stanu? - Pierre-Alexandre FORFAIT
35. Jak nazywał się oficer, który wysadził w 1812 r. (zresztą mało skutecznie) moskiewski Kreml? -kapitan Franciszek Koss
36. W czasach cesarstwa nadano 251 tytułów hrabiowskich, wśród nich Napoleon wyróżnił tytułem hrabiego dwóch Polaków. Prośże podąć nazwiska Polaków i daty nadania tytułów. - Wincenty Krasiński - 13 czerwca 1811; Aleksander Walewski - 5 maja 1812.
37. W 1799 roku Pierwszy Konsul trzymał do chrztu pewnego chłopca - przyszłego króla. Jakie imię otrzymał ów chłopiec i kim byli jego rodzice? -Oskar, jedynak Desiree Clary i Bernadotte'a, przyszły król Szwecji
38. Kto jest autorem podręcznika wydanego w Paryżu w 1811 roku "Essai sur le maniement de la lance" (co można tłumaczyć "Szkic o posługiwaniu się lanca")? - Wincenty Krasiński.
39. Proszę szanownych kolegów wymienić trzech znanych francuskich malarzy-batalistów którzy żyli i tworzyli w epoce napoleońskiej, i którzy oprócz pędzla nosili też broń. Jak się nazywali, i gdzie walczyli lub w jakich jednostkach służyli. Dla ułatwienia dodam, że jeden z nich dosłużył się stopnia generała, a obrazy wszystkich trzech z pewnością znacie, bo służyły często za ilustracje dla różnych publikacji. Aha! Jeszcze jedno zastrzeżenie - chodzi o Francuzów służących w wojskach francuskich! I niekoniecznie napoleońskich. - Horace Vernet (1789-1863); Theodore Gericault (1791-1824); Louis-Francois Lejeune (1775-1848).
40. Dzięki komu, według legendy, Aleksander I podpisał słynny akt "W imię Najświętszej Niepokalanej Trójcy"? - Dzięki baronowej Krudener, lwicy salonowej tego okresu i mistyczce, która wywierała duży wpływ na Aleksandra w latach 1814 – 1816

41. W sławnej scenie pożegnania na dziedzińcu Pałacu Fontainebleau, po abdykacji Cesarza w 1814 roku przed wyjazdem na Elbę, było obecnych trzech Polaków.
Kto to był? - Józef Kossakowski, Michał Pac, Stanisław Dunin-Wąsowicz.
42. Kogo nazywano "najlepszym poetą wśród generałów i najlepszym generałem wśród poetów"? - Chodzi o gen. Ludwika Kropińskiego [1764 (1767) - 1844]. W latach 1792-1794 osiągnął niższe stopnie oficerskie, aż Kościuszko mianował go pod koniec Insurekcji pułkownikiem. Kropiński rozpoczął później karierę literacką, wiążąc sie ze środowiskiem Puław. Jest autorem m.in. Ludgardy, Wiosny i Julii i Adolfa. Były to głównie tragedie i powieści. Na fali mobilizacji przed wojną 1812 r. wrócił do szeregów i 17 VII 1812 r. został gen. brygady. Został wysłany na Wołyń, aby organizować oddziały polskie, jednak nie zdziałał zbyt wiele. Od listopada przejął dowodzenie nad siłami polskimi w południowo - wschodniej części Księstwa od gen. Kosińskiego. Widząc rezultaty kampanii, a jednocześnie będąc blisko związany z Czartoryskimi, podał się do dymisji w lutym 1813 r.
43. Kim byli rodzice poległego pod Smoleńskiem gen. Michała Grabowskiego? - Elżbieta Grabowska z Szydłowskich król Stanisław August
44. Który z polskich pułkowników, który dochrapał się gen. za Królestwa Polskiego, ukończył szkołę politechniczną w Paryżu? Jak umarł? - Pierre Francois Charles Bontemps (1777-1840)
45. Był jeszcze jeden (oprócz Grabowskiego) oficer armii Księstwa, którego uważano za potomka Stanisława Augusta. Kto to był? - Chodziło o Michała Cichockiego (1769-1828), dowódcę 9 p.p., od stycznia 1815 r. generała brygady. Przekonanie to wzięło się stąd, iż król sprawował cały czas opiekę nad Cichockim i jego siostrą, łożąc na ich utrzymanie i edukację (a tą miał Cichocki dobrą - ukończył Szkołę Koronną Artylerii). Sam generał był przekonany, że jest synem kanclerza w. litewskiego Aleksandra Sapiehy.
46. Proszę wymienić dowódców 3 pułków piechoty posłanych za Pireneje jako tzw. Dywizja Księstwa Warszawskiego. - 4 p.p. - Feliks Potocki, 7 p.p. - Maciej Sobolewski, 9 p. p. Był nim Antoni Paweł ks. Sułkowski.
47. W czasie walk w 1799 we Włoszech generał Kniaziewicz otrzymał zaszczytną misję zawiezienia chorągwi zdobytych na wrogu do Paryża i "rzucić do stóp Dyrektoriatu". W 1812 roku podobną misję powierzono w Hiszpanii, pewnemu polskiemu pułkownikowi. Kto mu ją zlecił, kim był pułkownik i ile wiózł sztandarów. NO i czy wrócił potem do Hiszpanii? - Był to Stanisław Klicki, któremu misję zawiezienia 22 sztandarów i kluczy Walencji zlecił marszałek Suchet. Klicki nie wrócił później do Hiszpanii, lecz poprzez zakład Legii w Sedanie dołączył do sztabu Berthiera i wziął udział w kampanii rosyjskiej.
48. Jak nazywali się oficerowie, którzy przejęli dowodzenie jej regimentami po ich pierwszych pułkownikach (Potockim, Sobolewskim i ks. Sułkowskim)? - 4 p.p. - Maciej Wierzbiński; 7 p.p. - prowizorycznie dowodził kpt. Karpiński, potem przysłano Feliksa Grotowskiego z 9 p.p., a pełnoprawnym dowódcą został wraz z nominacją pułkownikowską Stanisław Jakubowski, także z 9 p.p., dopiero w październiku 1809 r.; 9 p.p. - Michał Cichocki ze sztabu armii Księstwa, w marcu 1810 r.
49. Kto zastąpił dowódcę batalionu polskich grenadierów gwardii cesarskiej Kurcyusza po ucieczce tego oficera po bitwie pod Lipskiem? - Stanisław Florian de Smett
50. Kto w marcu 1813 roku zajął Hamburg, po wyjściu z miasta garnizonu gen. Carre St. Cyra i kto odzyskał to miasto dla Francuzów? - Zajął je rosyjski gen. Tettenborn, a odzyskał Vandamme.
51. Jak nazywał się polski oficer, dowódca armii Republiki Rzymskiej w 1798 r.? -Był to gen. lejtnant (kościuszkowski) Paweł Jerzy Grabowski (1758-1812 ?), niefortunny dowódca obrony Wilna w 1794 r. i polityczny przeciwnik gen. Dąbrowskiego.
52. Gdzie dostał się do niewoli gen. Alfons d'Hautpool i jakim stopniem wówczas mógł się pochwalić? - Alphonce d'Hautpoul (1789 - 1865), późniejszy generał piechoty, ranny pod Salamanką 22 lipca 1812 r. Był wtedy porucznikiem (kapitanem?) i został zwolniony z niewoli dopiero w 1815 r.
53. Walczył w Algierii za Ludwika Filipa. Odznaczył się w 1808 r. dowodząc strażą tylną 116 p.p.l. - jedynego, który nie skapitulował pod Baylen. W 1815 r. w Armii Alp marsz. Sucheta. Kto to taki? - Thomas Robert Bugeaud, duc d'Isly
54. Podczas kampanii 1812 roku kilku generałów wojsk Księstwa Warszawskiego dostało się do rosyjskiej niewoli. Proszę podać ich nazwiska i miejsca "kuracji" - Dostało się do niewoli 7 generałów: Dominik Dziewanowski; Jan Konopka; Stanisław Małachowski; Jean Pelletier; Tadeusz Tyszkiewicz; Józef Wasilewski; Józef Zajączek. Bielecki podaje rejony zgrupowań polskich jeńców: gubernie kałuska, saratowska, tulska, tambowska, a także miasta powiatowe na Ukrainie i Powołżu. Wspomina nawet o Archangielsku, Wiatce i Astrachaniu oraz o tzw. liniach wojsk kozackich: kaukaskiej i orenburskiej.
55. Kogo ze swoich późniejszych współpracowników (przyjaciół ?) Napoleon poznał pod Tulonem w 1793 r.? – Duroca
56. Kto otrzymał od Nelsona w czasie bitwy pod St.Vincent szpadę hiszpańskiego dowódcy okrętu liniowego "San Jose " i jeszcze prosiłbym o podanie przyczyny – Był to dowódca okrętu „Captain” – Miller, na którym Nelson był zaokrętowany w czasie bitwy.
57. Podczas kampanii 1812 roku na Litwie usiłowano utworzyć m. in. regiment huzarów, pułk piechoty mohylewskiej oraz artylerię konną. Niestety jednostki te zgromadziły po... kilkanastu ludzi, lub wręcz ograniczały się zgoła do... samego dowódcy. Proszę podać nazwiska twórców tych trzech oddziałów. - dowódcą huzarów był Mikołaj Abramowicz, artylerii konnej - Rudolf Tyzenhauz.
Natomiast piechoty mohylewskiej - Paweł Sapieha
58. Książę Pepi w swej wiedeńskiej młodości zaprzyjaźnił się z dwoma młodymi oficerami służącymi wówczas wspólnie z nim pod sztandarami Habsburgów. Obydwaj odegrali później w epoce napoleońskiej znaczącą rolę i stawali jako samodzielni d-cy armii. Dla ułatwienia dodam, iż żaden z nich nie pochodził z rodziny cesarskiej, a jeden z nich na wieść o śmierci ks. Józefa podobno publicznie się rozpłakał. - Mack i Schwarzenberg
59. Który ze znanych polskich oficerów zajmował się organizacją 3 pułku eklererów gwardii? - Jan Kozietulski
60. Proszę wymienić przynajmniej czterech Francuzów w randze generała, którzy w okresie napoleońskim zrobili karierę w armii rosyjskiej -Langeron, Lambert, Saint-Priest, Armand Emmanuel du Plessy duc de Richelieu, Damas i i de Olone.

61. Junot miał pewien przydomek, pochodzący jeszcze z czasów republikańskich. Jak brzmiał ów przydomek? -"la Tempete" czyli burza
62. Gdzie rozpoczął swą karierę wojskową gen. Alexandre-Louis Andrault de Langeron, który dał się we znaki swoim rodakom w 1812 r. (pod Berezyną w składzie armii Czyczagowa) i 1813 (oblegał Toruń)? -W wieku 15 lat (a więc w 1778 r.) wstąpił do regimentu gwardii francuskiej. Brał udział w wojnie o niepodległość Stanów, ale nie w wojskach Rochambeau, bo stacjonował pod wodzą Viomesnila na San Domingo i przez pewien czas w Caracas. Po powrocie do Francji został pułkownikiem en second regimentu Medoc (1786), potem płk nadliczbowym regimentu Armagnac (1788). W 1790 r. zdecydował sie na emigrację.
63. Kto kryje się pod tytułem hrabiego de Saint - Leu? - Ludwik Bonaparte.
64. Generał francuski, rojalista, odznaczył się w szeregach armii rosyjskiej w wojnie z Turcją, w 1806 kierował odwrotem armii neapolitańskiej tak umiejętnie, że uzyskał uznanie przeciwników. O kim mowa? - Roger Damas
65. Kto był ostatnim napoleońskim gubernatorem Prowincji Iliryjskich? - książę Otranto Joseph Fouche
66. Kto był Wielkim Jałmużnikiem Cesarstwa? - kardynał Józef Fesch
67. 12 października 1809 roku podczas rewii wojskowej doszło do próby zamachu na życie Napoleona. Gdzie to było i kim był zamachowiec? - Fryderyk Stapps (Stepps) w Wiedniu (Schöenbrunn ).
68. Jeden z napoleońskich dowódców miał na piersiach tatuaż, który w związku z późniejszą karierą naszego bohatera zasługiwałby na miano " najbardziej paradoksalnego tatuażu w historii " . O kim mowa ? - Jean Baptiste Bernadotte'a, który na piersiach wypisał sobie: "Śmierć tyranom".
69. Jeden z francuskich generałów miał okazję popełnić samobójstwo w czeskich Budziejowicach. Który i z jakich powodów? - Jean Boudet, generał, który stracił swą artylerię pod Aspern, co spotkało się z wyjątkowo ostrą krytyką cesarza.
70. Załoga zamku Fuenglirola miała ponoć tylko cztery armaty zdatne do użytku. Nie było jednak artylerzystów. Trzech artylerzystów - Hiszpanów, uciekło przed nadejściem Anglików i dołączyło do swoich. W momencie gdy okręty angielskie zaczęły ostrzeliwać zamek, ten przez jakiś czas nie odpowiadał. Szukano na gwałt kogoś, kto by potrafił obsługiwać działa. Wreszcie pewien sierżant, który służył kiedyś w artylerii, z pomocą dwóch piechurów zdołał otworzyć ogień. Jak ów sierżant się nazywał? - sierżant Zakrzewski.
71. Jak nazywał się pewien osobnik, który prawdopodobnie (jak sam twierdził) zaczynał swą karierę w wojsku Księstwa Warszawskiego walcząc najpierw w 1809 z Austrią, a następnie już jako porucznik 7 p. lansjerów z Hiszpanami. W marcu 1812 r. (lub lipcu 1811) przeniósł się do pułku "czarnych" huzarów pruskich (6 p.huz. potem 2 p.huz.) z którym wziął udział w kampanii 1812 r. a potem już po drugiej stronie frontu - 1813 i 1814 r. W pruskich huzarach dosłużył się stopnia podpułkownika. Potem zwolnił się ze służby pruskiej (1822), wyjechał za ocean i odegrał pewną rolę w historii Meksyku przyczyniając się do niepodległości tego kraju i służąc wiernie innemu cesarzowi, (który zresztą chciał powtórzyć 100 dni Napoleona). W Meksyku znany był jako "polski pułkownik", bowiem wbrew epizodowi w armii pruskiej stale uważał się za Polaka. - Karol Beneskie (vel Benecke).
72. Jak nazywał się generał hiszpański, dowodzący hiszpańską dywizją posiłkową w 1807 roku, po wybuchu wojny francusko-hiszpańskiej ewakuował się z Danii do Hiszpanii na statkach angielskich? - gen. La Romana.
73. Jakimi tytułami i funkcjami państwowymi mógł poszczycić się marsz. Berthier? - Minister Wojny, książę Neufchâtel i Vallengin, książę Wagram, Wielki Łowczy, Głownódowodzący i Szef Sztabu Wielkiej Armii, Vice Konetabl Francji, Senator, Par Francji (to już za Ludwika), Generał Pułkownik armii szwajcarkiej, Wielki Orzeł Legii Honorowej, dowództwo I Kohorty LH. I Marszałek Państwa.
74. Pod Mondovi francuska armia straciła zdolnego oficera kawalerii o którym Napoleon wyrażał się w samych superlatywach wróżąc mu wielką karierę. Jego nazwisko cesarz przywoływał na łożu śmierci na Św. Helenie (tak przynajmniej twierdzi Antommarchi) razem z Desaix'em i Masseną. Jak nazywał się ów oficer i w jakich okolicznościach zginął? - General - Henri Christian Michael baron de Stengel (1744-1796) - ścigając Piemontczyków pod Mondovi został otoczony przez piemonckich dragonów i kilkakrotnie raniony pałaszem oraz postrzelony z pistoletu. wzięli go do niewoli. Widząc to ruszył mu na odsiecz Murat, który odbił ciężko rannego Stengela. Niestety "wspaniały huzar" jak go nazywano w armii zmarł wkrótce. Podobno umierając powiedział "Ten przeklęty korsykanin chciał mnie wykończyć i udało mu się...".
75. Ile lat miał i jak się nazywał najmłodszy Polak w Gwardii Cesarskiej odznaczony krzyżem Legii Honorowej? - Był to szwoleżer Michał Wilczek. Odznaczony został za zdobycie armaty pod Madrytem w grudniu 1808 r., a miał wtedy lat 16.
76. Kto dowodził w Wandei w 1815 r., walcząc tam z powstaniem rojalistycznym, które wybuchło po powrocie Napoleona z Elby? -gen. Jean Maximilien Lamarque (1770 - 1832).
77. W kampanii 1812 wziął udział pewien kapitan dragonów nazwiskiem Henri Beyle. Przeszedł do historii pod zupełnie innym aliasem. Kim był ów kapitan? – Stendhal
78. W latach 1810-1811 komendę nad Armią Portugalii sprawował marszałek Massena. Spośród jego 14 adiutantów.. jeden wyróżniał się nader foremną budową ciała. O kim mowa ? - Była to "przyjaciółka" Masseny - Henriette Lebreton, żona pewnego kapitana dragonów, którego pośpiesznie odesłano do odległego garnizonu. Nazywano ja "kurą Masseny".
79. Kogo Massena wysłał spod Torres Vedras do Napoleona z wieściami o kłopotach Armii Portugalii? - Generała Maksymiliana Foya.
80. Kim była Bellilotte? - Kochanka Napoleona w Egipcie - Paulina Margueritta Foures.

81. Kto reprezentował interesy Szwajcarii (nie poszczególnych kantonów ale całości Związku) na Kongresie Wiedeńskim? - Pictet de Rochemont.
82. Jedną z faworyt Ludwika XVIII nazywano "królewską tabakierą". Która to faworyta i skąd się wzięło to przezwisko ? - Madame du Claya, jej przezwisko wzięło się od zwyczaju króla do zażywania tabaki spomiędzy jej pośladków.
83. Kto według rozkazu Napoleona był odpowiedzialny za pozyskanie Kościuszki do planów Cesarza odnośnie Księstwa Warszawskiego ? – Fouche.
84. Jeden z wielkich polityków XX wieku jest także biografem Metternicha. Kto? - Henry Kissinger.
85. Który z francuskich dowódców nazywany był " Generałem bez skazy" ? dla ułatwienia - był dowódcą sił francuskich podczas angielskiego desantu na Walcheren, zanim dowództwo objął Bernadotte. - "general sans tache" to Jean Baptiste Dumonceau.
86. Znakomity generał francuski, poprzez małżonkę spokrewniony ze swoim własnym zwierzchnikiem - marszałkiem (a też w pewien sposób z Cesarzem)...Jego nazwisko ? - Louis Friant.
87. W hiszpańskim piekle na wyspie Cabrera więzionych było 11 polskich oficerów. Jeden z nich, główny chirurg 1 pułku ułanów i kawaler Legii Honorowej trafił w niewolę pod Jevanes i, jak sam pisał, "został przewieziony na wyspę Cabrera, gdzie pozostawał przez półtora roku w wilgotnych grotach, gdzie utracił zdrowie". Kim był ów oficer i jaki był jego stopień wojskowy? - Jan Franciszek Grill, major w pułku lansjerów polskich.
88. Który z dostojników Cesarstwa nosił tytuł księcia Feltre? - Henri Clarke, minister wojny od 1807 r.
89. Kto nosił tytuł księcia Parmy? - Tytuł księcia Parmy otrzymał z rąk Napoleona w 1808 r. Jean Jacques Regis de Cambaceres – Arcykanclerz Państwa.
90. Jeden z wysokich dygnitarzy w czasach cesarstwa został skazany na śmierć przez Burbonów, jednak w przededniu egzekucji zdołał uciec z więzienia. Jak się nazywał, jaką sprawował godność i jak udało mu się uciec? - Antoine Marie Chamans hrabia de La Valette (1769-1830) był Generalnym Dyrektorem Poczty. Poparł cesarza w czasie 100 dni za co otrzymał godność para Francji. W listopadzie 1815 skazany na śmierć za zdradę. W przededniu egzekucji jego żona dostała się do więzienia, i zamieniwszy z nim ubranie umożliwiła mu ucieczkę. Do Francji wrócił z emigracji ułaskawiony w 1822 r.
91. W okresie I restauracji (kwiecień 1814 - marzec 1815) Francja miała 3 ministrów wojny. Proszę podać ich nazwiska. - gen. Beurnonville kwiecień -maj 1814; generał Dupont de l'Etang od maja do grudnia 1814 r.; od grudnia 1814r. do 11 III 1815 r. Soult.
92. Po wygnaniu cesarza na Św.Helenę w Europie i Ameryce egzystowali nadal członkowie jego licznej rodziny, próbując czasami odzyskać jakiekolwiek znaczenie polityczne. Należała do tego grona pewna dama, która znana była z tego, że jeździła konno i strzelała lepiej od niejednego mężczyzny. Była też prawdopodobnie jedyną osobą z rodziny Bonapartych, której udało się dotrzeć do Orlątka w Wiedniu. Jak nazywała się ta osoba, i jaki był jej stopień pokrewieństwa z cesarzem? - hrabina Napoleona Camerata która była córką Elizy Bacciochi - starszej siostry Napoleona.
93. Ks. Reichstadtu zostało przedstawionych w Wiedniu dwóch Polaków - poetę i malarza. O kim mowa? -Zygmunta Krasińskiego i malarza Januarego Suchodolskiego.
94. W powstaniu listopadowym, miło wziąć udział kilku wybitnych dowódców francuskich, którzy swoją sławę dobywali pod Cesarzem. Proszę podać jedynego marszałka, który był skłonny do współpracy w tej kwestii i co najmniej dwóch generałów. – generałowie: Clauzel, Exelmans i Lallemand; marszałek Grouchy.
95. Którzy marszałkowie, mieli wykształcenie wojskowe (teoretyczne)? - Berthier - Szkola Wojsk Inzynieryjnych w Mezieres, Davout - Szkola Wojskowa w Paryzu, Grouchy - szkola Wojskowa w Strasbourgu, Marmont - Szkola Artylerii w Chalons-sur-Marne
96. W 1789 roku. Chłopicki wrócił do armii polskiej i wstąpił do 11 regimentu grenadierów pieszych Ilińskiego. Dowodził nim Maksymilian Sierakowski, a jego zastępcą był oficer, który zrobił potem sporą karierę w armii rosyjskiej. O kim mowa? - Ignacy Przybyszewski.
97. Jak nazywał się francuski generał - Polak, uczestnik kampanii 1792 r., konfederata barski, blisko związany z gen. Dumouriezem? - Józef Miączyński.
98. Kto przejął dowodzenie Armii Północnej po zdradzie Dumourieza? - gen. August Henryk Maria Dampierre.
99. Jak nazywał się rosyjski dowódca prowadzący z powodzeniem działania przeciw Persom na Kaukazie w l. 1804 - 1806? - gen. P.D. Cicjanow.
100. Który z dowódców sił sprzymierzonych uczestniczył w uroczystościach pogrzebowych ks. Józefa Poniatowskiego? - Braclay de Tolly.
101. Podczas kampanii 1809 r. do niewoli polskiej oddał się austriacki feldmarszałek - o kim mowa? - ks. Adam Kazimierz Czartoryski.

102. Proszę wymienić napoleońskich ministrów spraw zagranicznych w l. 1799 - 1815. Jeżeli to możliwe to z datami sprawowania urzędu. - od 1797 do 1807 Talleyrand (z przerwą od czerwca do grudnia 1799 – w tym czasie funkcję sprawował Reinhard) ;od 1807 do 1811 J. B. Champagny ks. Cadore ;od 1811 do 1813 H. B. Maret ks. Bassano ;od 1813 do pierwszej abdykacji Armand Augustine Louis Calaincourt ks. Vincenzy.
103. Kto napisał "Oblężenie i obronę Saragossy w latach 1808 i 1809 ze względem szczególniejszym na czynności korpusu polskiego"? - Józef Mroziński, kapitan 1 p. Legii Nadwiślańskiej.
104. Jak nazywał się mer Bordeaux, który w lutym 1814 r. uroczyście witał w tym mieście Wellingtona i wygłosił płomienną mowę na cześć Burbonów? – Lynch.
105. Jak nazywali się dwaj kapitanowie hiszpańskiej artylerii, którzy w czasie Dos de Mayo bronili madryckiego Arsenału? Ich śmierć stała się jednym z licznych symboli oporu. –Luís Daoiz; Pedro Velarde.
106. W bitwie pod Ligny 16 czerwca 1815 r. został śmiertelnie ranny jeden z francuskich dywizjonerów, weteran walk we Włoszech i Hiszpanii (był pod Albuherą ) odkomenderowany ze składu II korpusu Reille'a. O kogo chodzi? -Jean Baptiste Girard
107. Kto objął dowodzenie dywizją gen. Bourmont po jego dezercji 15 czerwca 1815 r.? -gen. Etienne Hulot
108. Który z rosyjskich generałów swoją dwuznaczną postawą umożliwił Austriakom zajęcie Sandomierza w 1809 roku? -Arkadij Aleksandrowicz Suworow
109. Ilu i jacy generałowie polegli lub zmarli z ran otrzymanych pod Akką w 1799 r.? -gen. Marie-Joseph Maximilien Caffarelli du Falga; gen. Francois Rambeaud; gen. Louis-Andre Bon
110. Dwóch polskich arystokratów mniej lub bardziej skrycie marzyło o tym, że dostaną koronę Księstwa Warszawskiego z rąk Napoleona. Pytanie brzmi - kto miał wówczas tak wielkie apetyty? - Wincenty Krasiński i Antoni Sulkowski.
111. Który z dostojników nosił tytuł księcia Massa di Carrara? - Félix Pasquale Bacciochi - Książę Massa-Carrara e La Garfagnana od 30 marca 1806 oraz Claude Ambroise Regnier (1746 - 1814), minister sprawiedliwości od 1802 r., wielki sędzia na dworze (Grand Juge) oraz duc de Massa et Carrara (pierwszy miał tytuł rince a drugo Duc)
112. Podczas wojny Księstwa z Austrią w 1809 r. miasto Sanok opanował oddziałek Księstwa Warszawskiego, lecz wkrótce nadeszły przeważające siły austriackie. Wojsko po krótkiej obronie zamku musiało ustąpić. Jak nazywał się oficer, który kierował obroną zamku, co ponoć wycofał się z niego jako ostatni? - kapitan Xawery hrabia Krasicki.
113. Francuz rojalista, uratował Blüchera od niewoli. O kogo chodzi? - płk Besancon szef sztabu Osten-Sackena.
114. Pośród francuskiej generalicji doby Rewolucji i Cesarstwa trafiali się także ludzie parający się piórem. Był pośród nich autor jednej z najpoczytniejszych powieści schyłku ancien regime'u, obnażającej w wyśmienity sposób kulisy libertynizmu. O kogo chodzi? - Pierre Choderlos de Laclos
115. Kto był zwierzchnikiem żandarmerii w I Cesarstwie, czyli takim "pierwszym żandarmem" Napoleona? – marszałek Moncey
116. Jak nazywał się osobisty lekarz Napoleona? - Dr. Jean Nicolas Corvisart
117. Jak się nazywał przyjaciel Józefa Sulkowskiego, (neofita) wieloletni francuski agent dyplomatyczny w krajach Wschodu, członek Towarzystwa Geograficznego? Zginął pod Akką podczas wyprawy egipskiej. - Venture de Paradis
118. Kim był Jean Pierre Augereau? - Jean Pierre Augereau był bratem przyrodnim marszałka; gen. bryg. (08.05.1804), Komandor LH i Kawaler Świętego Ludwika, baron (13.08.1811), gen. dyw. (01.07.1815). Początkowo dowodził brygadą w VII korp. swego imiennika, potem w Armii Hiszpanii (III korp. Monceya), potem w Armii Katalonii, dowódca 31 okręgu wojskowego (1811-1812), w 1812 ponownie pod rozkazami brata (31 dyw. Lagrange'a), przeniesiony w sierpniu do dyw. Baraguey'a d"Hilliers, z którą skapitulował 09.11. pod Lachowem. Wrócił do Francji w sierpniu 1814, w czasie Stu Dni w Korpusie Obserwacyjnym Varu (marsz. Brune). Od 1831 w rezerwie.
119. Kto był chrzestnym ojcem Wojciecha Kossaka? - Horacy Vernet
120. Który z polskich generałów epoki napoleońskiej był teściem austriackiego feldmarszałka? - Jan Henryk Dąbrowski był teściem Józefa Fryderyka Palombiniego.
121. Kto kryje się pod tytułem hrabiego de Peluse i czym zasłużył się Napoleonowi? - Gaspard Monge - wielki matematyk francuski, przyjaciel Napoleona – stał na czele Instytutu Egipskiego.
122. W bitwie pod Marengo odznaczył się odwagą bliski krewniak sławnego adiutanta Napoleona Józefa Sułkowskiego. O kim mowa? - Był to przyrodni brat Józefa Sułkowskiego - mjr Aleksander Sułkowski, który jako oficer huzarów walczył pod Marengo ale po stronie... austriackiej. Sułkowski był podobno ulubionym adiutantem gen. Melasa.
123. W 1796 w bitwie pod Arcole w ręce Francuzów wpadł austriacki sztandar. Napoleon w nagrodę za bohaterstwo w walce ofiarował go jednemu ze swych oficerów, potem bardzo znanemu. O kogo chodzi? – Lannesowi.
124. Generał rosyjski, brał udział w wyprawie do Szwajcarii Rimskiego-Korsakowa, dostał się do niewoli i dopiero w 1801 roku powrócił do Rosji. O kim mowa? - książę Fabian Wilgielmowicz Osten – Sacken
125. Uczestnik wyprawy Suworowa do Italii, podobno będący gwałtownego charakteru, został zabity na początku 1807 r. przez polskiego chłopa. O kogo chodzi? - Marszałek Aleksander Kamienskij
126. Jak nazywał się generał, odznaczający się osobistą odwagą w kampaniach 1811, 1812, 1813, szef sztabu Czyczagowa w 1812 i Barclaya w 1813? - gen.lejt. Iwan Wasiljewicz Sabaniejew.
127. Postać, która, choć stałą po drugiej stronie barykady, idealnie spinała epokę Rewolucji, napoleońską i powstanie listopadowe, a raczej to co po nim. Kontrkandydat Napoleona w wyborach 1792 r. Potem generał rosyjski i wreszcie ambasador Rosji w Paryżu - inwigilujący środowisko polskich emigrantów polistopadowych. Kto to taki? -Pozzo di Borgo, kontrkandydat do stopnia ppłk gwardii narodowej.
128. Który z napoleońskich marszałków uczestniczył w konfederacji barskiej? –Kellermann.
129. Zdobycie sztandaru kozackiego to nie lada wyczyn, choć nie niemożliwy o czym mógł sie przekonać pułk gen. Grekowa. Kto tego dokonał i z jakiego pochodził pułku? - Chodzi oczywiście o pułk krakusów.
9 września 1813 po wsią Strahwalde, krakusi wspólnie z kirasjerami z 14 p. starli się z rosyjskimi kozakami i dragonami. A sztandar zdobył wachmistrz Franciszek Godlewski (za co dostał od księcia Józefa Virtuti, a od Napoleona Legię)
130. Walki o wzgórze pod Uticą były tak zażarte że Tuczkow (dowodzący na tym odcinku) jeden z kontrataków przypłacił życiem. Jego stanowisko objął generał o bardzo wdzięcznym nazwisku. Kto? – Baggenhuffwood
131. Kto dowodził obroną Wysp Jońskich przed Anglikami w 1809-1810 r.? - Francois Xavier Donzelot (1764 - 1843), uczestnik kampanii egipskiej, Korfu bronił do 1814 r.
132. Do sławnej szarży pod Somosierrą przyłączył się pewien francuski oficer ordynansowy, czym przyczynił się do przypisania Francuzom współudziału w szarży, a nawet bezczelnego przypisania w "13 biuletynie Armii Hiszpanii" prowadzenia szarży gen. Montburn. Któż to był? - Filip Paweł de Segur (1780 - 1873) późniejszy autor słynnych "Pamiętników adiutanta Napoleona".
133. Kim był ojciec Filipa Pawła Segura i jaką funkcję pełnił w otoczeniu Napoleona? - LOUIS PHILIPPE SEGUR (1753-1830) członek rady państw, , wielki mistrzem ceremonii, senator w 1813 r.
134. Szwajcarski szef sztabu korpusu Neya, który opuścił szeregi armii francuskiej w 1813 r to... ??? - Antoine-Henri Jomini.
135. Kto dowodził obroną Burgos przed Anglikami w 1812 r.? - Gen. bryg. Jean Louis baron Dubretton (1773 - 1855).
136. Kto dowodził francuskimi pontonierami w Rosji ? - gen. dyw. Jean Baptiste Eble.
137. Kto dowodził - Pruska Tylnia Strażą pod Wavre w 1815? - gen. Johan Adolf Thielmann (1765-1824).
138. Kto zajmował się reorganizacją armii portugalskiej i sprawował jej dowództwo w wojnie hiszpańskiej? - William Carr Beresford (1768-1854), w marcu 1809 r. został marszałkiem portugalskim z misją odtworzenia sił zbrojnych kraju Luzytanów.
139. Kto dowodził jedyną dywizją, która stanowiła VII korpus WA Reyniera po dezercji Sasów na stronę koalicji pod Lipskiem w 1813 r.? - Pierre Francois Joseph Durutte (1767-1827).
140. 2 i 3 marca 1791 roku Clode Chappe przeprowadził pomyślne testy z telegrafem powietrznym, które to urządzenie doceniał także Napoleon i nakazał jego rozbudowę. Po samobójczej śmierci Clauda w 1805 roku jego bracia przejęli w administrowanie linie telegrafów powietrznych. Proszę podać ilu ich było i ich imiona? - braci Chappe było pięciu (licząc oczywiście nieszczesnego Claude'a) oto imiona pozostałych 1. Ignacy (1762-1829); 2. Piotr (1765-1834); 3. Rene (1769-1854); 4. Abraham (1773-1849).
141. W wieku 22 lat został mianowany generałem-majorem a w kampanii roku 1812 dowodził artylerią w armii Baclaya de Tolly. O którego rosyjskiego generała chodzi ???, w której wielkiej bitwie zginął ? - Aleksander Ivanovich Kutaisov (1784 -1812).
Jeden z młodych zdolniejszych dowódców rosyjskich, przed wybuchem wojny z Rosja, w latach 1810 -1812, studiował w Wiedniu języki arabskie i turecki, oraz w Paryżu studiował sztukę fortyfikacji, artylerie, i matematykę.
Walczył pod Golominem, Eylau, Friedlandem, zginął dowodząc cala artyleria rosyjska pod Borodino 1812.
142. Który brytyjski generał, wygrał bitwę stoczona 16 maja 1811 roku pod Albuera ? - William Beresford (1768-1854).
143. Kim był Brytyjczyk który walnie przyczynił się do przegranego przez Napoleona oblężenia w 1799 roku oraz pomógł przetransportować wojska tureckie pod Aboukir? - William Sidney Smith
144. Kto dowodził inwazja francuska na Irlandie w 1796 roku ? - Gen. Louis Lazare Hohe (1768-1797).
145. Jak się nazywał obrońca i protektor "króla galer", adwokat paryski, wydał jego biografię w 1857 roku? - Charles Ledru.
146. Brytyjski korpus ekspedycyjny w tym kraju w 1801 r. miał dwóch dowódców. Proszę ich wymienić. - gen. John Hely-Hutchinson (1757-1832); gen. Ralph Abercromby.
147. Który z generałów podczas kampanii egipskiej 25 lipca 1798 r. otrzymał rozkaz udania się do prowincji Menouf jako jej gubernator? Później został gubernatorem prowincji Beni-Souef i Fayoum. - Józef Zajączek.
148. Proszę wymienić francuskich generałów poległych pod Kanopą w 1801 r - Francois Lanusse; Auguste Baudot; Cesare Antoine Roize.
149. Proszę wymienić znanych potomków Cesarza i ich matki. - 1. Francois Charles Joseph Bonaparte zwany orlątko (1811-1832) - Maria Ludwika; 2. Alexander Florian Walewski hrabia (1810-1868) - Maria Walewska; Karol - nazwany hrabią Leonem. Matką była Eleonore Denuelle de La Plaigne.
150. Który z admirałów brytyjskich był członkiem parlamentu w 1807 dla pewności odpowiedzi dodam iż jego statek nazywał się "Szybki" - Thomas Cochrane (1775-1860).

151. Jacy polscy generałowie dostali się do niewoli pod Lipskiem? - L. Kamieniecki,; I. Krasiński; S. Grabowski; F. M. Paszkowski; K. MaLachowski; J. Umiński; Bronikowski; A.Rózniecki; Rautenstrauch; Sierawski; Żółtowski (może ktoś zna konkretną pewna odp. różne źródła inaczej podają)

152. Który z wybitnych polskich wojskowych epoki napoleońskiej był skoligacony z tym wielkim rodem Conde? -Jan Konopka był spokrewniony przez matkę - Annę ks. de Conde
153. Jak nazywał się francuski oficer, który z polecenia samego Napoleona poszukiwał grobu Kopernika na Warmii w 1807 roku? - Gerard Gley
154. Jak nazywał się mieszczanin z Olsztyna, który zamierzał (podobno) zabić Napoleona?- Rydzewski

155. Proszę o wymienienie przynajmniej pięciu polskich Chevalier de l'Empire - Ludwik Krukowski, kapitan Legii Nadwiślańskiej, Wojciech Jerzy Lipowski kapitan 1 pulku L.N., Jozef Falk Regulski, szef batalionu L.N., Franciszek Krzysztof Wilhelm Schutz, szef batalionu 3-go pułku L.N., Stanisław Smette (de Smette), kapitan 2-go pułku L.N., Kajetan Wysocki, kapitan 1-go pułku L.N.

156. Jak nazywał się generał polski, który w 1812 roku, po wkroczeniu Napoleona na Litwę został dowódcą brygady jazdy litewskiej, a w 1813 pod Sierakowem dostał się do niewoli rosyjskiej? - Gen. książę Romuald Giedroyć (1750 - 1824).

157. Był kolejnym generałem kościuszkowskim "odkurzonym" w 1812 r. Był także (a może przede wszystkim...) inspektorem generalnym piechoty litewskiej. O kogo chodzi? - Gen. mjr Franciszek Ksawery Niesiołowski

158. Który z polskich oficerów otrzymał Krzyż Neapolitański? Dla ułatwienia dodam że zginął w powstaniu. - Ludwik Kicki

159. Pod opieką ilu lekarzy znajdował się Napoleon na wyspie Św. Heleny? Proszę wymienić przynajmniej trzech. - Barry Edward O'Meara, Stoke - lekarz z okrętu "Conqueror", Antommarchi i na końcu Arnott, lekarz pułkowy 20 pułku piechoty, William Warden z "Northumberlanda", który przebywał na wyspie do 1816 r., Aleksander Baxter, szefa służby zdrowia na wyspie (teoretycznie)

160. Francuski kontradmirał, który zginął na pokładzie swojego okrętu, próbując dostarczyć zaopatrzenie dla Malty i Egiptu. Sytuacja miał miejsce 18 II 1800r. Dowódca poświęcił swój okręt, aby ratować pozostałe. Któż to taki? - Jean Baptiste Perree.

161. Kto pod Albuhera na wezwanie aby oddał sztandar odkrzyknął: "Never! Except with my life!"? - Był to podporucznik Edward Thomas, trzymający sztandar królewski 3 pułku (Buffs). Oddał - życie i sztandar.

162. W bitwie pod Fuentes de Onoro brygadą kawalerii w VI korpusie Loisona dowodził generał, który w 1802 r. jako pułkownik, spiskował przeciw I konsulowi i przesiedział nawet kilka tygodni w więzieniu. Kto to był? - generał Francois Fournier-Sarloveze (1773-1827).

163. Kto dowodził siłami francuskimi w Portugalii w czasie tzw. "wojny pomarańczowej" w 1801 r.? - gen Leclerc

164. Dnia 3 lutego 1793 roku Michał Ney dostał nominację na adiutanta i oto pytanie właśnie - adiutanta jakiego generała? - generała Francois’a Joseph’a Drouot’a, zwanego Lamarche (1733-1814), dowódcy awangardy Armii Holandii Dumourieza.

165. Pułkownik tej Legii Nadwiślańskieji, który zginął w czasie walk nad Berezyną, być może zabity przez Francuskich kirasjerów to...
Proszę jeszcze o podanie okoliczności, a raczej przyczyn takiej właśnie śmierci. - Chodzi o Mikołaja Kąsinowskiego. Brandt tak opisuje ten epizod spod Berezyny: "Przyszło już do ręcznych zapasów, kiedy niespodziewanie nadciągnęli francuscy kirasjerzy i wpadli na walczących. W tym zamieszaniu zarąbali dużo naszych, między innymi pułkownika Kąsinowskiego, który miał na sobie zielone futro". Pomyłka wynikała z tego, że jegrzy z którymi ucierali się legioniści mieli mundury ciemnozielone, i pewnie kirasjerzy wzięli go za oficera rosyjskiego.

166. Kto dowodził "dywizją straceńców"? - gen.Durutte który wzespół z dywizą Dąbrowskiego osłaniali odwrót pod Lipskiem, w Pfafendorf.

167. Na którego z marszałków podczas "jakiejś" bitwy Napoleon krzyknął : "Cóż za dzielny człowiek" ? Dla ułatwienia powiem że bitwa rozegrała się w 1809roku- MacDonald pod Wagram

168. Podaj stopień, imię i nazwisko dowódcy wojsk polskich broniących Torunia w maju 1809 roku? - Generał brygady Stanisław Wojczyński

169. Kto był szefem sztabu Armii Hiszpanii w czasie bitwy pod Ocaną w listopadzie 1809 r.? –marszałek Soult

170. Po pierwszym dniu bitwy pod Lipskiem Napoleon wysłał do sił sprzymierzonych jednego z generałów wziętych do niewoli tego samego dnia. Ten generał niósł im propozycje pokojowe. Jaki się nazywał ten generał i jakiej był narodowości? - Maksymilian Merveldt, dowódca II korpusu austriackiego.

171. Jak nazywali się dwaj dywizjonerzy Marmonta w 1814 r., którzy na sugestię marszałka przeprowadzili VI korpus na stronę sprzymierzonych? - Joseph Souham (1760-1837) był dowódcą jednej z dywizji piechoty i to de facto on był głownym wykonawcą "spisku". Etienne Bordessoulle (1771-1837) dowodził natomiast dywizją kawalerii.

172. Kogo Ney nazywał "kobietę swojego życia"? - Ida Saint-Elme

173. Kogo Napoleon nazwał "Talmą Północy"? - cara Aleksandra

174. W 1811 Napoleon zmienił Ministra Spraw Zagranicznych. Kto zastąpił na tym stanowisku Jana Champagny'ego? - Maret, ks. Bassano

175. Który z uczestników bitwy pod Albuherą popełnił kilka miesięcy później samobójstwo? - Generał Nicolas Godinot (1765-1811)

_________________
"W polityce głupota nie stanowi przeszkody."
Napoleon


Ostatnio edytowano 3 lut 2010, o 21:40 przez Louis Nicolas, łącznie edytowano 6 razy

Góra
 Zobacz profil  
 
PostNapisane: 3 lut 2010, o 21:16 
Offline
Général de brigade
Général de brigade
Avatar użytkownika

Dołączył(a): 3 lut 2010, o 06:45
Posty: 1248
Post użytkownika Unclean

PYTANIA RÓŻNE.



1. Co trzymał w ręce generał Antoine Lasalle podczas swej ostatniej i śmiertelnej szarży pod Wagram? –fajkę
2. W żyłach którego generała z epoki płynęła jednocześnie szkocka, niemiecka i rosyjska krew? - Michaiła Barclaya de Tolly
3. Wiele osób widziało na pewno obrazy przedstawiające Księcia Poniatowskiego z rozwianą burką na ramieniu. Było to podobno jego ulubione okrycie. Jakie jest pochodzenie tej burki i dodatkowo jak nazywał się ulubiony koń Księcia? - Burki takie nosił Pułk 2 Wiernych Kozaków formowany w 1792 roku. Była to jedna z ulubionych formacji Księcia, którą dowodził jego imiennik płk. Józef Poniatowski.
Ale nie sądzę by Książe miał przez całe życie tylko jedną burkę. Ulubionym wierzchowcem Księcia był kary ogier, rasy arabskiej lub anglo-arabskiej, "Szumka". Pochodził on ze stadniny Sanguszków. Także burkę z sierści wielbłądziej książę zakupił podobno za sto dukatów od Żyda z Astrachania, w czasie pobytu w Lubarze w 1792 roku. Kolejna teoria - prezent od Murata.
4. Jak powszechnie wiadomo Wielki Książę Bergu i Kliwii – Joachim Murat - dostał od księcia Poniatowskiego szablę Batorego. Jakim podarunkiem odwdzięczył się księciu Pepiemu? -"...przepyszny kaszkiet karmazynowy ułański, bogato złotem wyszywany (podobno własnoręcznie przez Karolinę Murat) oraz parę bogatych pistoletów.."
5. W 1809 roku Napoleon padł ofiarą kradzieży, skardziony przedmiot udało się odzyskać dzięki pomocy pewnego nawróconego kryminalisty. Co zostało skradzione i kim był ów nawrócony kryminalista ? - W 1809 roku miał zostać skradziony szmaragdowy naszyjnik Józefiny, a człowiekiem, dzięki któremu miano go odzyskać był oczywiście Vidocq.
6. Co z wielkim upodobaniem jadali kozacy w Fontainbleau w 1814 roku? – karpie
7. W 1817 roku jeden z kamerdynerów Cesarza, Costant poznał pewnego kapitana gwardii cesarskiej(zranionego pod Lipskiem), który poprosił go(Constanta) o jakikolwiek przedmiot należący kiedyś do Napoleona Bonaparte. Jak się nazywał ów oficer i jaki przedmiot otrzymał? - krzyż legii honorowej z czasów konsulatu( Napoleon nosił go ostatni raz pod Austerlitz ).
8. W jakiej miejscowości i od kiedy działała manufaktura/fabryka robiąca białą broń i inne narzędzia wojskowe dla armii Księstwa Warszawskiego? – Michałowo od 1811
9. 26 lutego 1795 roku Nantes było świadkiem niecodziennej parady, do miasta wjechał Franciszek de Charette (jeden z legendarnych przywódców powstańców wandejskich) w towarzystwie rewolucyjnych generałów. Jakie wydarzenie umożliwiło te paradę ? - Podpisanie 17 lutego rozejmu z republikanami w la Jaunaye
10. Jak wiadomo, Szwecja uczetniczyła w wojnach z Francją III i IV koalicji (1805 i 1806-1807). Jednak nie zakończyła ich wraz z Rosją, Prusami i Austrią. Moje pytanie: gdzie i kiedy Szwecja podpisała pokój z Francją napoleońską? – Traktat paryski, 6 stycznia 1810 r. Ze strony francuskiej podpisał Champagny, ze strony szwedzkiej - hrabiowie Essen i Lagerbielke.
11. Jak była ostatnia prowincja włączona do Wielkiego Cesarstwa? - Katalonia włączona 26 stycznia 1812 r. - utworzono departamenty: Ter (Gerona), Segre (Puicerda), Bouches-de-l'Eble (Lerida) oraz Montserrat (Barcelona). Potem Leridę i Barcelonę złączono w jeden departament
12. Gdzie i kiedy nastąpiło pierwsze publiczne rozdanie krzyży Legii Honorowej? - Pierwsza dekoracja Orderem Legii Honorowej odbyła się 15 lipca 1804 roku w Pałacu Inwalidów
13. Czego szczególnego dokonał pułkownik Richard Fletcher w trakcie kampanii na Płw. Iberyjskim ? - Był konstruktorem fortyfikacji brytyjskich pod Torres Vedras
14. Ile razy Joseph Fouche zostawał Ministrem Policji? – 3 w 1799, 1804 i 1815
15. Która kolonia francuska wpadła jako ostatnia w ręce Anglików i kiedy się to stało? –Mauritius. Po krótkim oporze ostatnia posiadłośc francuska skapitulowała 3 grudnia 1810 r.
16. Wśród licznych polskich wyczynów, było też wzięcie do niewoli księcia Lichtenstein. Kiedy kto i jak? - Lichtensteina do niewoli wziął ponoć żołnierz Trandowski z Legii Naddunajskiej w pobliżu Lambach w grudniu 1800 roku. Stało się to podobno wskutek zakładu z kolegą o halbę wina, co sprowokowało naszego dzielnego ułana do szarży w kierunku sztabu nieprzyjaciela i, mimo ostrzału Austriaków, udało mu się księcia porwać i przyprowadzić do swego dowódcy
17. Którego dzisiaj mamy według kalendarza republikańskiego? (był 17 05 2003) - 28 Floréal an 211 de la Révolution
18. W październiku 1797 roku w Paryżu gościła delegacja amerykańska..rozmowy miały dotyczyć uregulowania stosunków pomiędzy Francją a Dyrektoriatem - ostatnio znajdujących się prawie w stanie niewypowiedzianej wojny. Negocjacje zostały jednak zerwane, wywołując olbrzymie wzburzenie amerykańskiej opinii publicznej... Co było powodem amerykańskiej irytacji ? - Była to tzw. "afera XYZ", czyli próba wymuszenia od posłów amerykańskich łapówki przez Talleyranda
19. Po zajęciu San Domingo w pierwszych miesiacach 1802 r. sytuacja wydawała sie tam ustabilizowana. Jednak w lecie wybuchło powstanie , które wkrótce rozszerzyło się na całą wyspę. Bezpośrednim impulsem, który pchnął Murzynów do buntu była pewna pogłoska, która dotarała do nich właśnie w tym czasie i dotyczyła innej kolonii francuskiej. Co to była za pogłaska i jakich wydarzeń dotyczyła? - Wprowadzenia powtórnie niewolnictwa na Gwadelupie
20. Która kohorta Legii Honorowej została przydzielona Berthierowi ? –Pierwsza

21. Jak się dokładnie nazywa i gdzie leży(dzisiejsze tereny Polski) ,zachowane w całości, dzieło fortyfikacyjne autorstwa generała Haxo? - Redita Napoleońska (nie mylić z redutą)
22. Kiedy i na czyj rozkaz zostało sprowadzone z Wiednia do Kościoła Inwalidów w Paryżu, ciało syna cesarza, Napoleona II "Orlątka"? -Prochy Napoleona II, Króla Rzymu, (20.03.1811 - 22.7.1832) syna Napoleona I-go cesarza Francji i Marii Ludwiki , zostały przekazane jako podarunek Adolfa Hitlera z Wiednia do Pałacu Inwalidów w Paryżu 15 grudnia 1940 roku.
23. W którym miesiącu został "poczęty" syna Napoleona i Marii Walewskiej - Aleksander? - Hrabia Walewski urodził się 4 maja 1810 roku, wygląda wiec na to ze został poczęty w sierpniu 1809 roku, podczas pobytu Napoleona w Wiedniu (Schoenbrunn) gdzie dojechała do niego Pani Walewska.
24. Napoleon oskarżał o zdradę w latach 1814-1815 Marmonta, Talleyranda i Fouche, ale miał także pretensje i to duże, do marszałka Augereau . Co mu zarzucał? - W 1814 roku, Augereau 13 lutego otrzymał rozkaz od Napoleona odcięcia Armii Austriackiej od granic Austrii, co by ich wyeliminowało z walki i Napoleon miał by przeciwko sobie tylko Rosjan i armie Bernadotta. Augereau dysponując 25000 armia nie wykonał rozkazu i nie ruszył się z pod Lyonu. 16 lutego Napoleon ponowił rozkaz z dopiskiem "Jeśli jeszcze jesteś tym dawnym Augereau spod Castiglione". Na co marszałek odpowiedział "Daj mi ludzi spod Castiglione", i nie ruszył się aż do 28 lutego, ale wtedy już było za późno.
25. W 1811 r. Cesarz wydał dekret ograniczający liczbę ukazujących się w stolicy tytułów prasowych do 4. Jakie to były tytuły? - "Le Moniteur"; "La Gazette de France"; “Le Jurnal de Paris"; “Le Jurnal de l'Empire"
26. Książę oprócz tego, że jak wiadomo był miłośnikiem płci pięknej i koni, kochał także muzykę. Nawet podczas wojen woził ze sobą niewielki klawikord który nazywał "obozowym szpinecikiem". Otóż książę był uczestnikiem pewnego bardzo ważnego wydarzenia muzycznego, które przeszło do historii nie tylko muzyki poważnej, ale chyba w ogóle kultury. Co to było za wydarzenie i kiedy miało miejsce? - Pepi był w 1788 r. obecny na wiedeńskiej premierze "Don Giovanniego" (czyli inaczej mówiąc - Don Juana)
27. Kiedy Czaplic zaatakował 3 Pułk Szwoleżerów Gwardii, w Słonimiu znajdowały się dwie żony polskich generałów. Proszę podać nazwiska tych pań oraz czy udało im się uniknąć niewoli. - Te generałowe to Barbara Dąbrowska (z domu Chłapowska) żona gen. Dabrowskiego, oraz Dominica Pierdilucca, żona Jana Konopki.
Obie dzielne dziewczyny zdołały uniknąć niewoli, odesłane przez Konopkę w kierunku na Wilno pod osłoną plutonu szwoleżerów, który w większości poległ, broniąc kobiet, ale także pułkowej kasy.
28. Z czego zrobiła armia Wellingtona drabiny użyte do szturmu na Ciudad Rodrigo 19.o1.1812 r.? - Z chłopskich wozów, na których przywieziono zapasy amunicji.
29. Jak nazywał się zamek, będący kwaterą Napoleona w momentach, gdy przebywał on na wybrzeżu La Manche w l. 1803-1805? -Clocheville w Pont-de-Brique.
30. Podczas odwrotu z Rosji samo słowo "cosaque" budziło wielki postrach. Jakim słowem w swych rozmowach określali kozaków Polacy, by nie budzić paniki wśród swoich sprzymierzeńców? -Polak kozaka nazywał "gościem".
31. W pierwszym roku XIX stulecia Duroc odwiedził stolicę pewnego europejskiego mocarstwa. Wizyta zaowocowała traktatem pokojowym. Cóż to było za państwo, z którym podpisano pokój dzięki temu pośrednictwu ? I kiedy to miało miejsce? -Chodzi o traktat z Rosją z 8 października 1801 roku a stolicą jest Sankt Petersburg
32. Jaki tytuł książęcy miał Androche Junot i od czego on pochodził? – duke d'Abrantes - swój książęcy "nick" Junot wziął od zamku Abrantes, który generał zajął podczas kampanii portugalskiej. Junot w drodze do Lizbony opanował ten opuszczony zamek i obwarował, po czym pośpieszył zająć stolicę.
33. Macka ogólnie kojarzy się z jego kapitulacją w 1805 r. Ale kilka lat wcześniej miał jeszcze jeden "występ", równie nieudany i zakończony klapą. O jakie wydarzenia chodzi? -4 XII 1798 pobity przez Championneta pod Civita Castellana, dostał się do niewoli, a dwór wiedeński odmówił wymienienia go na jeńców francuskich.
34. Skąd został usunięty Ludwik XVIII w 1801 r., po chwilowym odprężeniu francusko - rosyjskim? -Chodzi o Mitawę w Kurlandii, gdzie Ludwik został osadzony przez Pawła I w 1798 r.
35. Na jesieni 1808 roku ambasador Portugalii w Londynie zlożył oficjalny protest w pewnej sprawie, która bardzo wzburzyła angielską opinie publiczną. Czego dotyczył ten protest ? - Chodziło o warunki konwencji w Cintra (30 VIII), która przewidywała ewakuację wojsk francuskich z Portugalii razem z całym "inwentarzem" tzn. tym, co zrabowały
36. Kiedy Artur Wellesley otrzymał swój tytuł książęcy i od czego on pochodził? - Arthur Wellesley, Duke of Wellington. Zwycięska bitwa pod Talavera 28 lipca 1809 roku dala Arthurowi Wellesley godność Para i tytuł Viscount Wellington of Talavera. Nazwa Wellington została przybrana od posiadłości rodzinnych w zachodniej części kraju, miasteczka w hrabstwie Samorset, niedaleko posiadłości Welleslie, które od tej pory można tytułować księstwem.
37. W początkach 1813 roku, w Wittenberdze(gdzie Polacy stanowili większość garnizonu) została utworzona pewna tajna organizacja. Jak brzmiała jej nazwa? - Związek im. Jana Henryka Dąbrowskiego.
38. Pasierb Napoleona - Eugeniusz Beauharnais posiadał kilka tytułów. Znany jest przede wszystkim jako wicekról Włoch i książę cesarstwa. Miał jednak jeszcze cztery inne tytuły książęce. Roszę podać dwa spośród nich nadane przez Napoleona (księciem Leuchtenberg i Eichstaedt został z nominacji swego teścia - króla Bawarii Maksymiliana) - Eugeniusz był księciem weneckim i Wielkim Księciem Frankfurtu (tymczasowo jako następca po Teodorze Dalbergu).
39. Odczytanie hieroglifów egipskich przez Champolliona było możliwe dzięki pewnemu przedmiotowi odnalezionemu przez żołnierzy francuskich w 1799. Co to był za przedmiot i jak nazywał się dowódca oddziału, który go odnalazł ? - tzw. kamień z Rosetty, czyli granitową stellę z napisem na cześć jednego z królów Egiptu z dynastii Ptolemeuszy. Wykonany był trzema alfabetami: hieroglificznym, demotycznym i greckim. Analiza porównawcza pozwoliła rozszyfrować hieroglify, do czasów Champolliona stanowiące zagadkę. A oficer, który dowodził oddziałem to Francois Bouchard, porucznik inżynierii.
40. Książę Józef Poniatowski w okresie starań u władz pruskich o zwrot niesłusznie zagarniętych majętności i dochodów został odznaczony przez króla pruskiego Jakimi odznaczeniami ? -Dostał dwa Orły: Czarnego i Czerwonego.

41. W języku polskim kobiety towarzyszące armii jako służba pomocnicza określa się najczęściej jako markietanki. Jednak i wśród nich istniało pewne zróżnicowanie, co nie ma właściwie odniesienia w naszym ojczystym języku. Stąd pytanie: Kogo określano mianem Vivandiere oraz Cantiniere? - Wiwandierka to kobieta prowadząca sklepik z żywnością, tytoniem i różnymi potrzebnymi przedmiotami, a kantynierka prowadziła rodzaj stołówki. Panie musiały posiadać specjalne zezwolenia na swą działalność i były rejestrowane.
42. Wellington w 1812 roku otrzymał tytuł książęcy od hiszpańskich Kortezów.. jak brzmiał ten tytuł i za co Wellington go otrzymał ? -tytuł księcia Ciudad Rodrigo, za zdobycie... Ciudad Rodrigo.
43. No to następne pytanie. W styczniu 1799 roku gen. Dąbrowski miał okazję poznać feldmarszałka Macka. Gdzie i w jakich okolicznościach ? - Po kapitulacji Neapolitańczyków Ich głównodowodzący gen. Mack przeszedł na francuską stronę z obawy przed zemstą ludności i właśnie Dąbrowski był pierwszy z wyższych oficerów, który musiał z nim rozmawiać i odesłać pod konwojem do sztabu armii francuskiej. Było to pod Capuą.
44. W jakich okolicznościach Massena stracił oko? - We wrześniu 1808 roku Massena był zaproszony przez Cesarza na polowanie w Fointainebleau. Podczas strzelania przez nieuwagę Napoleon przyczynił się ze proch z wystrzelonej niego broni pozbawił Massene lewego oka, ale Berthier lojalnie wziął cala winę na siebie.
45. Arcyksiążę Karol był jednym z najlepszych dowódców epoki...był też wielkim reformatorem armii austriackiej...Jego reformy wojskowe nie zostały jednak doprowadzone do końca, ze względu na duży opór stawiany miedzy innymi przez pewien tradycyjny organ doradczy Świętego Cesarstwa Rzymskiego. Jak nazywała się ta instytucja ? - Rada Aulicka- a tym terminem określa się zarówno Reichshofrat, jak i Hofkriegsrat.
46. Jaka kara była przewidziana dla delikwenta, który we Francji w czasie I Restauracji publicznie wydałby z siebie okrzyk "Vive l'Empereur!"? - dwa lata więzienia.
47. Zanim pojawiła się Legia Honorowa żołnierzy Republiki nagradzano honorowym orężem (choć to może mało precyzyjne słowo). Była ona przydzielana w zależności od broni i funkcji w jakiej służył delikwent. Jaka to była broń (i nie tylko), i kto ją otrzymywał? Proszę o podanie przynajmniej czterech przykładów (nie chodzi o obdarowanych, ale o rodzaj "darowizny" i funkcję obdarowanego) - Kawalerzyści (i generałowie) - honorowe szable. Piechota - honorowe karabiny. Artylerzyści - honorowe granaty. Trębacze - honorowe trąbki. Dobosze - honorowe pałeczki do werbli.
48. Na początku 1818 roku 400 byłych żołnierzy napoleońskich przybyło z Missisipi i założyło kolonie na terenie teksasu w dolnym biegu rzeki Trinity, w pobliżu dzisiejszego Libety. Planowali zorganizowanie stąd wyprawy na Wyspę Świętej Heleny i uwolnienie Napoleona. Brak żywności i groźba hiszpańskiego ataku zmusiły Francuzów do porzucenia kolonii zaledwie po 8 miesiącach jej istnienia.
Kto stał na ich czele, i jaką nazwę nosiła owa osada? - gen. Charles François Antoine Lallemand - prezydent Societe Agricole et Mecanique, osada Champ d'Asile (Pole Azylu).
49. W 1813 w okolicach Naumburga grasował partyzancki odział gen. Adolfa. Odział ten został rozbity przez kawalerie Krukowieckiego. Podaj nazwę tej partyzantki. - Rache-Legion (Legia Zemsty).
50. W 1806 r. Duroc sygnował dwa traktaty międzynarodowe: rozejmowy i pokojowy. Z kim i gdzie były one podpisywane? - Jeden to traktat z Prusami z 16 listopada 1806 roku w Dessau, a drugi w Poznaniu z Saksonią (11 grudnia).
51. W 1814 roku Łazarz Carnot dowodził obroną pewnego belgijskiego miasta. Jakiego? – Antwerpia
52. Co powiedział Napoleon Henrykowi Dembińskiemu mianując go kapitanem? gdzie to się wydarzyło? - Napoleon zapytał się "kim jesteś?", a Dembiński odpowiedział "porucznik Dembiński". "A więc mianuję cię kapitanem". Potem znów się zapytał o to samo, a Dembiński, myśląc, ze Cesarz zapomniał, znów powiedział "porucznik Dembiński", na co Napoleon odpowiedział "nie, przecież mianowałem cię przed chwilą kapitanem".
53. Podczas kampanii 1806/1807 Bluecher dostał się do niewoli francuskiej, ale jeszcze podczas trwania wojny wrócił do służby pruskiej. Jak do tego doszło? - Bluecher dostał się do niewoli pod Schwartau. Po kapitulacji pod Ratkau został wkrótce wymieniony na gen. Victor'a, którego 20 stycznia 1807 r. wzięło do niewoli kilkunastu huzarów i strzelców z freikorpsu Schilla, kiedy ten ze Szczecina udawał się pod Gdańsk. Wymiana odbyła się 8 marca.
54. Proszę o wymienienie co najmniej 4 francuskich marszałków, którzy sympatyzowali ze sprawą polską i w ogóle z Polakami - Davout, Murat, Macdonald, Bessires
55. W pobliżu placu Clichy. Stoi na nim pomnik poświęcony pewnej postaci (...), pewnej formacji (...) i pewnemu wydarzeniu (...) z przedostatniego etapu wojen napoleońskich. Wypełnijcie owe "pewne" miejsca, pisząc o kogo, jaka formacje i wydarzenie chodzi. - Chodzi o obronę rogatek Clichy przez gwardie narodową i studentów, którą dowodził marszałek Moncey. I to jemu i im poświęcony jest pomnik. Miało to miejsce 30 III 1814.
56. Dlaczego Chłapowski opuścił służbę francuską w 1813 r.? - Ta decyzja wynikała z szoku, jakiego doznał po usłyszeniu w czasie rokowań pielaszkowskich (czerwiec 1813 r.) propozycji rozejmowych Cesarza przewidujących m.in. inkorporację Księstwa Warszawskiego do Rosji.
57. Kim byli mieszkańcy tzw. "zielonego salonu"? - Chodzi o lokatorów rzymskiego mieszkania Madame Mere czyli matki cesarza – Letycji Bonaparte.
58. Jak w polskich jednostkach brzmiał wymawiany z francuska rozkaz formowania czworoboku? – Formuj Karre!
59. Jakie były osobiste związki Bluechera z Polską? -po wydaleniu z armii w poł. lat 70-tych XVIII w. poślubił córkę oficera regimentu gwardii pieszej koronnej- Polaka.
60. Jakie cechy powinien posiadać kandydat na wodza armii wg. H. Jominiego? - 1) Silny charakter (odwaga duchowa) 2) Zimna krew (odwaga fizyczna) 3) Wiedza

61. Podczas składania ocalałych orłów polskiego korpusu przed ks.Poniatowskim w „ pierwszym rzucie „ zabrakło jednego. Dlaczego? - Jeden z żołnierzy niosących orły nie chciał swojego złożyć przed księciem bowiem znak miał oberwaną główkę i poczciwiec wstydził się pokazać takiego Wodzowi. Jednak zachęcony przez innych w końcu dołączył go do kompletu.
62. Podczas sławnej kampanii 1800 r. poniósł w dramatycznych okolicznościach śmierć szef sztabu Kniaziewicza i przyjaciel Kościuszki - płk Ambroży Gawroński. Jak zginął i jakie to były okoliczności? -Chodziło o walki frakcyjne w Legii Naddunajskiej i próbę wysadzenia z siodła Kniaziewicza. Gawroński podpisał adres do Moreau o odsuniecie Kniaziewicza od komendy. Moreau odesłał go z upoważnieniem do ukarania niesubordynowanych oficerów. Kniaziewicz chciał załatwić sprawę polubownie, drąc ten dokument, ale Gawroński wziął to sobie za bardzo do serca i popełnił samobójstwo, skacząc do Renu.
63. Jak nazywało się dziełko Napoleona, wydane w 1793 r., w którym w formie dialogu bierze on w obronę politykę jakobińską na południu Francji? - Uczta, bądź "Wieczerza w Beauciare” (Souper de Beauciare).
64. Komu generał Górecki zameldował o wyborze Piłsudskiego na prezydenta w 1926 roku? - Gen. Górecki meldował warszawskiemu pomnikowi ks. Józefa autorstwa Thorvaldsena
65. W czasie kampanii 1800 r. Kniaziewicz nakazał francuskim artylerzystom, aby ci w określonych przez niego okolicznościach "walili do niego kartaczami". Dlaczego, i co to miały być za okoliczności? - W Kehl jeden z batalionów piechoty Legii Naddunajskiej chciał porzucić służbę. Kniaziewicz wydał taki rozkaz dowódcy francuskiej baterii artylerii, ponieważ gdyby nie udałoby mu się zapobiec dezercji, nie chciał on aby Polacy się zhańbili..
66. Co to była "afera XYZ"? - Sprawa dotyczyla incydentu w relacjach dyplomacji Francusko-Amerykanskiej (1797-8). Konsekwencja Traktatu Pokojowego Stanow z Anglia w 1795.
Prezydent USA wysłał 3 dyplomatów do Francji w celu wypracowania porozumienia, które pozwoliło by uniknąć uwikłania Stanów w wojnę z Francja.. Propozycja została zdecydowanie odrzucona przez ówczesnego ministra spraw zagranicznych Charles'a Talleyranda .
Nieoficjalnie zaproponowano Talleyrand'owi 250 000 dolarów, za pozytywne załatwienie sprawy. Ludzie którzy prowadzili rozmowy w tej sprawie (łapówki) dwóch Szwajcarów i amerykański bankier z Hamburga zostali oznaczeni kryptonimami XYZ.
67. Jaki był maksymalny zasięg, francuskiego działa 8- funtowego? - zasięg francuskiego działa 8- funtowego w tym okresie wynosił, maksymalnie 1350 metrów.
68. Trzykrotnie 1szy Pułk Szwoleżerów Gwardii ratował cesarza z opresji
gdzie i kiedy ? - Małojarosławiec - 25.X.1812; Brienne - 29.I.1814; Arcis-sur Aube- 21.III.1814
69. W czyje urodziny został podpisany pokój w Amiens, 25.III ? – w 35 urodziny Murata.
70. Ile wynosiła suma wszystkich donacji Chłopickiego, które otrzymał jako nagrody za służbę w Legii? - były to donacje 3 x po 4000 franków, więc razem 12 tys.
71. D-ca naszych sławnych szwoleżerów - Wicek Krasiński był człowiekiem... delikatnie mówiąc trudnym. Jego wyniosłość i egoizm powodowały, że popadał w konflikty z otoczeniem. Pojedynkował się m.in. ze swoim podkomendnym Henrykiem Ignacym Kamieńskim. Zdarzyło się jednak, że w innym "pojedynku" starszy od niego oponent rozkwasił nos młodemu hrabii- kto to był, i w jakich okolicznościach odbyła się ta bijatyka? - Chodziło o znanego wówczas warszawskiego wierszopisa, panegirystę i grafomana płk Marcina Molskiego (weterana bodajże insurekcji i chyba także legionów), który z równym zapałem sławił każdą nową władzę. Ale Krasińskiego bardziej denerwował jego związek ze znienawidzonymi Poniatowskimi, których Molski był piewcą. Krasiński dążył wszelkimi sposobami do wykluczenia wierszoklety z Towarzystwa Przyjaciół Nauk, co doprowadziło w marcu lub kwietniu 1807 do pojedynku (a raczej bijatyki) jaka odbyła się przy blasku pochodni (a więc w obecności świadków) na ulicy Koziej. I oto jak pisze Brandys "Warszawa przeżyła nie lada sensację: stary, słaby, ślepy na jedno oko wierszopis rozkwasił nos młodemu dziarskiemu pułkownikowi kawalerii. Była to pierwsza rana Krasińskiego odniesiona w kampanii napoleońskiej”.
72. czy w epoce Rewolucji - Cesarstwa jakiś członek wielkiego rodu de Conde zapisał się w annałach historii i na czym polegała jego "aktywność"? - Louis Joseph DE BOURBON-CONDÉ (1736 - 1818), na emigracji utworzył Armee de Conde i walczył z siłami republiki, np. szturmując Thionville w roku 1792.
73. Jaki był ulubiony strój Napoleona w podróży i podczas pracy w swoim gabinecie ??(chodzi o dwa różne stroje) - chodzi o mundur pułkownika strzelców konnych Gwardii i słynny szary szynel.
74. Proszę o wymienienie dwóch miejscowości leżących na obu brzegach rzeki Berezyny, gdzie w 1812 r. wzniesiono słynne mosty. - Studzianka i Bryły
75. Skąd wzięła się nazwa tzw. „Wojny pomarańczowej” 1801 r.? - w pierwszym dniu tejże wojny żołnierze hiszpańscy atakowali zamek (pałacyk chyba bardziej) Elvas. Było kilku rannych i kilku zabitych, pierwszą wiadomość o zdobyciu zamku dostarczyli Godoyowi (Manuel Godoy dowodził w owej pamietnej chwili "spaniardom") wraz z dwiema gałązkami drzewa pomarańczowego z ogrodów tego zamku. Ten nie omieszkał przesłać owych gałązek Marii Luizie jako pierwszej zdobyczy wojennej. Następnego dnia w poczytnym periodyku (Gazeta de Madrid) można było przeczytać o owej "zdobyczy", ludność Madrytu miała podobno płakać ze śmiechu a od tej chwili nazywano tą wojnę właśnie "wojną pomarańczową"
76. Kogo Talleyrand złośliwie (jak zwykłe) określał mianem "Hic-Haec-Hoc" (Ten-Ta-To)? - Hic – Napoleon/ Haec - Jean Cambaceres/ Hoc – Lebrun
77. W jakich okolicznościach Dezydery Chłapowski dostał się do niewoli, kiedy i gdzie to miało miejsce i kiedy z niej powrócił. - do niewoli dostał się pod gdańskim Chełmem, 26 marca 1809 r. Przetrzymywano go w Rydze, wrócił zaraz po Tylży i we wrześniu 1807 r. - "Dwa szwadrony dragonów (pruskich) puściły się w linii. Jeden z nich prosto na nas uderzył i przeleciał. Pomiędzy dwoma końmi przewróciłem się, uderzony w kark. Straciłem przytomność i nie wiem, jak długo leżałem, anim wiedział, kto ze mnie mundur ściągnął. Gdym przyszedł do siebie, leżałem na ziemi, przede mną i koło mnie kilku kozaków i jeden tylko z woltyżerów także już obdarty z płaszcza, ale nie z munduru"

_________________
"W polityce głupota nie stanowi przeszkody."
Napoleon


Ostatnio edytowano 3 lut 2010, o 21:44 przez Louis Nicolas, łącznie edytowano 6 razy

Góra
 Zobacz profil  
 
PostNapisane: 3 lut 2010, o 21:17 
Offline
Général de brigade
Général de brigade
Avatar użytkownika

Dołączył(a): 3 lut 2010, o 06:45
Posty: 1248
Post użytkownika Unclean


PYTANIA - CYTATY.


1. "W gwardii oddawano broń i tornister tylko razem z życiem"- Kapitan Coignet.
2. " Walczycie za Napoleona i cóż z tego wam przyjdzie ? A dziś czyż to za waszą sprawę przelewacie krew pod murami Madrytu ? Cóż ma wspólnego Wisła z Tagiem ?" - Arcyksiążę Ferdynand przed wkroczeniem do Księstwa Warszawskiego 14 kwietnia 1809 r
1. O jakiej bitwie Napoleon miał powiedzieć - "To jest wnuczka Marengo"? - Chodzi o bitwę nad rzeką Raab pomiędzy ks. Eugeniuszem Beauharnais a arcyksięciem Janem
2. "Tam mu podziękuję!"
Z jakim wydarzeniem kojarzą Wam się te słowa? - Biwak armii napoleońskiej noc przed szarżą szwoleżerów pod Somosierrą. Polak podchodzi do ogniska cesarza, bierze węgielek podpala fajkę i odchodzi. Ktoś domaga się aby przynajmniej podziękował a w odpowiedzi pada cytowane zdanie
3. Kto, do kogo i w jakich okolicznościach zwrócił się z takimi słowami: "Wie pan, gdy Moskale zobaczą amarantową czapkę ułańską, to wściekają się jak byki"? - Ową kwestię wygłosił cesarz niemiecki Wilhelm II podczas wizyty w pracowni Wojciecha Kossaka w reakcji na jego opis kłopotów z wystawieniem panoramy "Samosierra" w W-wie
4. O kim as rosyjskiego wywiadu - hr. Czernyszew (zwany przez paryżanki le beau Tchernitcheff) napisał w raporcie następujące słowa: "... to człowiek grubiański i okrutny. Znienawidzony przez wszystkich, którzy otaczają cesarza Napoleona. Serdeczny stronnik Polaków i wielki wróg Rosjan" - O marszałku Davout
5. Kto, do kogo i w jakich okolicznościach zwrócił się z takimi słowami: "Wie pan, gdy Moskale zobaczą amarantową czapkę ułańską, to wściekają się jak byki"? - Ową kwestię wygłosił cesarz niemiecki Wilhelm II podczas wizyty w pracowni Wojciecha Kossaka w reakcji na jego opis kłopotów z wystawieniem panoramy "Samosierra" w W-wie
6. O kim as rosyjskiego wywiadu - hr. Czernyszew (zwany przez paryżanki le beau Tchernitcheff) napisał w raporcie następujące słowa: "... to człowiek grubiański i okrutny. Znienawidzony przez wszystkich, którzy otaczają cesarza Napoleona. Serdeczny stronnik Polaków i wielki wróg Rosjan" - O marszałku Davout
7. Kto i gdzie napisał te słowa:
"Chciał (Napoleon) zrobić z naszych synów mameluków bez Boga, bez rodziny i ojczyzny. Zdaje się, że ten wróg wszystkiego zaciekł się aby zniszczyć Francję u podstaw. Bardziej zepsuł ludzi, więcej złego uczynił rodzajowi ludzkiemu w dziesięć lat, niż wszyscy tyrani rzymscy razem, od Nerona aż do ostatniego prześladowcy chrześcijan (...) Niedowiarstwo, żądza uciech i wydatków nad stan, lekceważenie więzów moralnych, awanturnictwo, gwałt i tyrania, schodzą od tronu w rodziny. Jeszcze jakiś czas takiego panowania, a Francja stałaby się jaskinią opryszków". "Niedorzeczny jako gospodarz, zbrodniczy jako polityk, i cóż on miał, ten cudzoziemiec, aby uwieść Francuzów? Chwałę wojenną? Stracił ją. Był to w istocie wielki wygrywacz bitew; ale pozatem najmniejszy generał zręczniejszy jest od niego".
Podpowiem, że autor tego, jakby nie było, paszkwilu kończy go słowami "Niech żyje król!", choć wcześniej pracował dla Napoleona, a później będzie występował przeciwko władzy Burbonów - Rene de Chateaubriand.
8. Kto i w jakich okolicznościach wypowiedział te słowa: "Najjaśniejszy Pan nigdy nie miał na widoku odbudowania Polski". - hr. Montalivet. Miało to miejsce 13 XII 1809 r. na sesji Ciała Prawodawczego, podczas prezentacji dorocznego raportu ministerialnego o stanie Cesarstwa. Był to jednocześnie propagandowy ukłon w stronę Rosji - Napoleon był w przeddzień rozwodu i nadal liczył na rękę siostry cara.
9. O kim Napoleon miał powiedzieć następujące zdanie: "Mój Boże! Jakaż szkoda, że nie jest Pan moim ministrem! My dwaj podbilibyśmy świat!"? - o Metternichu.
10. Kto powiedział Izabeli Czartoryskiej następujące słowa "Polska ma trzech wrogów: Prusy, Austrię i Rosję, oraz jednego przyjaciela: mnie" - Cesarz Wszechrusi – Aleksander
11. "Za satysfakcję, jakiej doznawałem przez cały czas, kiedy miałem zaszczyt odbywać służbę z oddziałami tak zdyscyplinowanymi i tak cennymi, jakimi są właśnie odziały wchodzące w skład (.) dywizji... Historia powinna zachować wszystko, o czym mieliśmy okazję pozytywnie pisać, i przekazać potomności wiele pięknych cech, którymi szczycą się dowódcy i żołnierze..."

Czyje, o kim, i w jakich okolicznościach? -Gen. Grandjean, dowódca 7 dywizji, podczas zdawania komendy 28 grudnia 1813, o i do polskich żołnierzy.
12. A: -...Trzeba przepuścić ich więcej, po czym silnym uderzeniem zepchnąć do rzeki i przypieczętować straszliwą ich klęskę.
B:- Dobre to, tylko ich nie wpuść za wiele.
Kim jest A oraz B? - "A" to arcyksiążę Karol, "B" to cesarz austriacki Franciszek I. rzecz, znaczy się dialog miał miejsce w czasie walk pod Wagram.
13. "Powtórzę zdanie, które już mówiłem panu poprzednio. Nie jestem wcale pewien, czy zupełne zniszczenie Napoleona i jego armii byłoby dobrodziejstwem dla świata. Dziedzictwo jego przypadłoby nie Rosji i nie żadnemu z mocarstw kontynentalnych, lecz państwu, które już w chwili obecnej króluje na morzach i którego panowanie stałoby się wówczas nie do zniesienia". – Kto, kiedy i do kogo wypowiedział to zdanie? - marszałek Kutuzow w październiku 1812 roku tracąc cierpliwość, właśnie w tej formie odpowiedział komisarzowi angielskiemu Wilsonowi.
14. "Rozbiję waszą monarchię tak jak tą porcelanę !"
Kto i w jakich okolicznościach wykrzyknął te słowa? - Napoleon do Coblenza w końcowej fazie rokowań przed zawarciem pokoju w Campo Formio w październiku 1797 r. na zamku w Passariano. Napoleon rozbił wtedy zastawę podarowaną Cobenzlowi przez Katarzynę II w Kaniowie dziesięć lat wcześniej.
15. "Twoje serce, którego powodzenia nie uczyniły głuchym na jęki cierpiącej ludzkości, krwawi się bez wątpienia na samo wspomnienie tylu nieszczęśliwych istot, które czekają jeszcze zbawienia od Francyi... Piętnaście milionów Polaków, kiedyś niepodległych, dzisiaj ofiar sil i okoliczności, zwracają swe spojrzenia na Ciebie. Chcieliby obalić zaporę rozłączającą ich od Ciebie, aby dzielić Twe niebezpieczeństwa, aby Cię uwieńczyć nowymi laurami i dodać do wszystkich tytułów, jakieś sobie zdobył, tytuł ojca uciśnionych!". – Kto i kiedy napisał te słowa? - Michał Kleofas Ogiński.
16. Gdyby Ojczyzna Pańska miała podnieść się z upadku i gdyby Wam trzeba było człowieka zdatnego do pchnięcia rzeczy naprzód, wezwijcie mnie do siebie, a gdziekolwiek wypadnie, dumnym będę przyłożyć się do przywrócenia wolności narodowi, który tak bardzo jest jej godzien" - który ze znanych napoleońskich generałów złożył tę propozycję służenia sprawie polskiej? – gen. Kleber.
17. "Przysiegam przebić pierś własnego brata, jeżeli będzie godził w wolność Republiki!”
Kto i w jakich okolicznościach wypowiedział te słowa? - Lucjan Bonaparte przy okazji zamachu swojego brata w 1799 r.... Powiedział to po wyjściu z sali do grenadierów, aby przekonać ich co do czystych zamiarów Napoleona. Zaraz potem rozegrała się słynna scena z wypędzeniem deputowanych.
18. "Strzelajcie i nie bójcie się, czyńcie wolę Boga! Strzelajcie!" – Kto wykrzyczał te słowa? - Joachim Murat ( w czasie egzekucji)
19. Kto zostawił takie oto wspomnienia z sekcji zwłok Napoleona:

"6 maja (...) O wpół do trzeciej otwarto ciało. Obecnych było siedemnaście osób (...).
Stwierdzono, że pęcherz jest mały, co zmuszało cesarza do częstego oddawania moczu. Znaleziono coś w sercu, co sprawiało, że krążenie było powolne - rodzaj wady budowy w lewej części; wreszcie okazało się, że najciekawszy jest żołądek, siedlisko choroby. Zwłóknienia odźwiernika spowodowały powstanie naokoło licznych wrzodów, w sąsiednich częściach żołądka; jeden z tych wrzodów zrakowaciał i przebił żołądek powodując dziurę takiej wielkości, że można w nią było włożyć mały palec. Żołądek w tym miejscu przylegał do wątroby, która stała się większa niż normalnie i nieco przekrwiona, choć protokół angielski o tym nie wspomina." - jest to opis gen. Bertranda, zawarty w jego "Zeszytach ze Świętej Heleny".
20. któż wypowiedział te słowa?
"Nie boję się niepowodzeń. Cesarstwo twoje ma dwóch potężnych obrońców: przestrzeń i klimat... a gdyby nawet okoliczności nieszczęśliwe zmusiły cię do odwrotu przed zwycięzkim wrogiem, cesarz Rosji będzie zawsze potęzny w Moskwie, groźny w Kazaniu a niezwyciężony w Tobolsku" - gubernator wojenny cara Aleksandra I Fiodor Roztopczyn

_________________
"W polityce głupota nie stanowi przeszkody."
Napoleon


Ostatnio edytowano 3 lut 2010, o 21:45 przez Louis Nicolas, łącznie edytowano 3 razy

Góra
 Zobacz profil  
 
PostNapisane: 3 lut 2010, o 21:18 
Offline
Général de brigade
Général de brigade
Avatar użytkownika

Dołączył(a): 3 lut 2010, o 06:45
Posty: 1248
Post użytkownika Unclean


PYTANIA "MORSKIE".

1. Jak nazywał się okręt, którym dowodził brat Napoleona – Hieronim? –Vétéran
2. Po Trafalgarze Francuzi zanotowali tylko jeden sukces na morzu, ale za to jakże piękny -cztery jednostki angielskie zniszczono lub zdobyto, ponad 1000 Anglików dostało się do niewoli. Gdzie i kiedy miała miejsce ta bitwa? - Grande Porte 23-28 VIII 1810
3. Jakie francuskie liniowce miały nazwy związane z Polską ? - "Polonais", "Danzig", "Pultusk" i "Golymin".
4. 14 września 1805 r. Kiedy inwazja na wielka Brytanie staje się niemożliwa, Napoleon rozkazuje, by flota z Kadyksu pożeglowała przez Gibraltar na Morze Śródziemne, dla wsparcia armii francuskiej, atakującej sprzymierzone z Anglia Królestwo Neapolu. Jednocześnie Cesarz odwołuje wiceadmirała Pierre Villeneuve do kraju. Kogo Napoleon mianował nowym dowódcą floty francuskiej - Admiral Francois Rosily-Mesrosa
5. O tym że rozpoznanie jest bardzo ważne wiedział również Nelson. 1 października 1805 roku do Kadyksu wdarła się angielska fregata by "policzyć" wrogą flotę.
Jaką nazwę nosiła owa fregata? - Była to "Euryalus" Henry'ego Blackwooda.
6.6. Gdzie i kiedy Nelson stracił ramię, a gdzie i kiedy oko? - Nelson stracil oko 12 VII 1794 r. w czasie oblężenia Calvi a rękę pod Teneryfą bodajże 14 VII 1797 r. w czasie ataku na fort Santa Cruz.
7. Ogromny francuski trójpokładowiec, zwodowany w 1790r., najczęściej znany pod nazwą „La Montagne” („Góra”), nosił w okresie swej służby jeszcze cztery inne miana. Wymieńcie przynajmniej dwa. - Okręt ten w czasie budowy zaplanowany został jako "Etats de Bourgone" - Stany Burgundii, ,bo za pieniądze tej prowincji został on zbudowany. Jeszcze przed wodowaniem jego nazwę zmieniono na "Cote d`Or" Potem nosił on nazwę "La Montagne" i tak najczęściej nazywany jest w źródłach. W czasie swej służby nosił one jeszcze dwa miana: "Peuple" - Lud i "Ocean".
8. O kim Napoleon miał powiedzieć: "Gdybym mógł jemu powierzyć komendę, nie byłoby Trafalgaru"? - Entienne-Eustache Bruix

_________________
"W polityce głupota nie stanowi przeszkody."
Napoleon


Góra
 Zobacz profil  
 
PostNapisane: 3 lut 2010, o 21:37 
Offline
Général de brigade
Général de brigade
Avatar użytkownika

Dołączył(a): 3 lut 2010, o 06:45
Posty: 1248
Post użytkownika Oudinot

PYTANIA O KAMPANIE, WOJNY, BITWY i WOJSKA.

62. Której jednostki wojskowej dotyczył dekret z 31 marca 1809 o podziale na półbrygady? - Dywizja grenadierów Oudinota

63. Zalążkiem tego pułku był powstały w roku 1803 batalion armii francuskiej "Pionners noirs", w składzie którego byli wyłącznie Murzyni ( przybysze z San Domingo). W roku 1806, batalion ten, zasilony Europejczykami, w sile pułku przeszedł na służbę jednego z państw sojuszniczych, gdzie otrzymał nazwę "Royal African" ( od 1810 - "Real Africano"). Wziął udział w kampanii rosyjskiej 1812, walczył też w kampanii 1813. Dla ułatwienia dodam, ze dowódca tego pułku po roku 1815 "spokrewnił" się z samym Napoleonem. Jaki to pułk? - Przynależący do armii Neapolu 7 pułk, dowodzony przez Francesco Madonalda

64. W czasie Stu Dni Napoleon obok szwoleżerów i lansjerów stworzył jeszcze jeden oddział stworzony z Polaków, którzy zostali ściągnięci z niewoli hiszpańskiej, angielskiej i lazaretów. Było to kilkaset żołnierzy. Jak Napoleon nazwał ten pułk, którego pierwszy batalion zdąży oddać ostatnie strzały w kampanii? - Trzeci pułk cudzoziemski

65. W armii sprzymierzonego z Francją Królestwa Bawarii w latach 1811-1812 istniały pułki piechoty liniowej o następującej numeracji: 1-11 , 13. Dlaczego nie istniał 12.pułk piechoty liniowej? - 12. pułk piechoty (bez nazwy) powołany został 15.03.1803 i sformowany z poborowych z rejonu Würzburga. W lipcu 1806 przeszedł do nowo powstałego W.Ks. Würzburg, a w armii bawarskiej pozostał na następne lata vacat

66. Czym (jeśli chodzi o wyposażenie) różnili się kirasjerzy polskiego 14 pułku z 1812 r. i 1813 r.? Jaka była szerokość ostrza ich pałaszy, czym różnił się wzór AN IX od AN XIII, a czym pochwa kirasjerskich pałaszy od dragońskiej? - W 1813 nie mieli kirysów, tylko kaski. Szerokość ostrza pałaszy wynosiła dokładnie 35 mm. Pałasze AN IX i AN XIII, różniły się jedynie tym, ze późniejsza wersja miała dwie szerokie bruzdy na ostrzu i wzmacniającą ość pośrodku. Pochwy kirasjerskie były metalowe a dragońskie drewniane obciągnięte skórą i okute.

67. Kto powołał w 1792 roku pierwszą jednostkę pontonierów i pod jaką nazwą? - Generał Armand Louis Biron de Gontant, duc de Lauzun (1747-1793) utworzył w Alzacji, z rybaków i innych pływających po Renie "Bataillon de Matelots et Volontaires du Rhin"

68. W trakcie jednej z kampanii armia francuska i jej sprzymierzeńcy stoczyli w przeciągu kilku miesięcy, na jednym i tym samym polu 4 bitwy. Gdzie i kiedy? - Pod Berg Isel w 1809 roku.

69. W 1796 roku powołano do życia gwardię Dyrektoriatu. Kto został jej dowódcą i z jakich oddziałów była złożona? - Pierwszym dowódcą został generał dywizji Jean-Ernest Krieg. Skład gwardii to dwie kompanie grenadierów pieszych i dwa szwadrony grenadierów konnych oraz orkiestra.

70. Podczas oblężenia Gdańska w 1807 r. miała miejsce udana operacja desantowa. Kiedy i gdzie została przeprowadzona, i z jakiego oddziału pochodzili biorący w niej udział Polacy? - W nocy na 7 maja miał miejsce desant na ostrów, dowodził nim pułkownik Aime. Polacy to kompania grenadierów z 3 (11) p.p. dowodzona przez kpt. Ludwika Kamińskiego

71. W roku 1809 gen. książę Bagration nieudanie oblegał pewne tureckie miasto, proszę podać jego nazwę i dowódcę, przez którego został zastąpiony w marcu następnego roku w wyniku przerwania tegoż oblężenia. - Miasto to Silistra, zastąpił Bagrationa graf Kamieński

72. Jaka twierdza, jako ostatnia kapitulowała na Śląsku i jak nazywał się jej ostatni francuski dowódca? - Głogów, generał Laplane.

73. Jaka polska formacja uczestniczyła w rozbiciu oddziału mjra Schilla w 1809 roku? - 4 pułk strzelców konnych Walentego Kwaśniewskiego.

74. Armia Królestwa Westfalii była na wzór Cesarstwa podzielona na dywizje terytorialne (division militaire). W roku 1808 istniały trzy dywizje (czwarta powstała w roku 1810), a ich dowódcy byli równocześnie gubernatorami miast, a w jednym przypadku dodatkowo jeszcze komendantem twierdzy. W jakich miastach stacjonowały te dywizje? - Kassel, Brunszwik, Magdeburg, Hanower.

75. W trakcie trwania kampanii 1805 Napoleon powołał nowy korpus ? VIII. Kto nim dowodził i które dywizje ( proszę o podanie ich dowódców) weszły w jego skład? - Dowodził marszałek Mortier, dywizje Gazana, Duponta, Dumonceau i Kleina.

76. Ile razy i w jakich okolicznościach w okolicach Zatoki Aboukir dochodziło do starć zbrojnych w kampanii egipskiej? - Trzy razy (1798, 1799, 1801)

77. W bitwie pod Medina de Rioseco dnia 14 lipca 1808 roku odznaczył się nie tylko walecznością, ale i łagodnością wobec pokonanych Hiszpanów jeden polski szwadron. Pod czyim był dowództwem i z jakiej pochodził formacji? - Kapitan Radzimiński, dowódca 6 kompanii, ze szwoleżerów

78. Jakie jednostki kawalerii były w okresie około-Yevenes dołączone do ułanów nadwiślańskich, tworząc z nimi prowizoryczną brygadę? - 3 pułk huzarów holenderskich i 1 pułk szwoleżerów westfalskich

79. Kilka dni przed potyczką ułanów nadwiślańskich pod Jevenes 10. pułk strzelców konnych płk. Subervie został pobity przez 1200 Hiszpanów (brygada?) dowodzonych przez gen. Hemestrosa. Straty francuskie wynosiły według danych hiszpańskich 63 zabitych i 70 rannych. Kiedy i gdzie rozegrana została ta potyczka? - Miajadas ? 21 marca 1809.

80. W roku 1812 artyleria konna Księstwa Warszawskiego, składająca się z 4 kompanii, została podzielona. 1. kompania kpt. Ostrowskiego została wysłana do garnizonu Gdańska, a trzy pozostałe przyłączone zostały do pewnej jednostki. Jakiej? - 4. korpus kawalerii rezerwowej gen. Latour-Maubourga

81. 30.IV Najsłynniejsza istniejąca na świecie jednostka wojskowa obchodzi swe święto. Jest to rocznica ich chwalebnej klęsk. Czyim rozkazem została ona powołana do życia? - Legia Cudzoziemska powołana rozkazem Nicolasa Soulta.

82. W kampanii 1806-1807 V.korpus Wielkiej Armii miał aż trzech dowódców (oczywiście nie równocześnie). Jak się nazywali? - Lannes, Savary, Massena

83. W trakcie kampanii rosyjskiej jeden z polskich pułków stoczył 28.07.1812 potyczkę z jednostką dowodzoną przez gen. Kulniewa. O który pułk chodzi? - 8. pułk szwoleżerów-lansjerów płk. Łubieńskiego

84. Podczas bitwy pod Waterloo, w czasie kolejnych szarż francuskiej kawalerii wyginęli wszyscy wyżsi oficerowie jednego z angielskich regimentów. Widząc zbliżających się kolejny raz kawalerzystów francuskich Wellington, który był akurat w pobliżu osobiście przejął komendę nad tym regimentem. Jaka to jednostka? - 95 pieszy regiment Rifles

85. Co to były "szwadrony huzarów tabaczkowych i pięknych dragonów błękitnych z różowymi wyłogami i bośniaków z lancami"? - 6 pułk huzarów von Schimmelpfeniga, 3 pułk dragonów von Irvinga i ułani Prittwitza

86. W okresie luty-lipiec 1807 roku jednostki Wielkiej Armii oblegały twierdzę Kołobrzeg. Proszę o podanie dowódców tego oblężenia. - Pierre Teulie, Edouard Mortier i generał Loison

87. Kto dowodził Szwedami na Pomorzu w chwili odkomenderowania tam Grandjeana? - Jan Henryk Essen.

88. Gouvion St. Cyr, Macdonald, Championnet to kolejni dowódcy pewnej armii francuskiej. Jakiej? - Armia Rzymu

89. Przez pułk lekkokonny polski gwardii przewinęło się w sumie 11 oficerów zdrowia. Wśród nich było 5 Polaków i 6 Francuzów. Proszę o podanie ich nazwisk. - Boulay, Capel Achilles, Courtiade Grzegorz, Deplace Ludwik, Gadowski Wawrzyniec, Girardot Franciszek, Kuszański Aleksander, Lewkowicz Sykstys, Lucy Onufry, Mańkowski, Maugra Mikołaj

90. Ile armat zdobyły polskie wojska podczas kampanii 1809 r.? - 68

91. W czasie kampanii 1806-1807 większość twierdz pruskich została przez Wielką Armię zdobyta lub poddały się bez walki. Pięć z nich jednak odparło ataki wojsk napoleońskich broniąc się aż do zawieszenia broni w lipcu 1807. Jakie? - Srebrna Góra, Kłodzko, Koźle, Grudziądz i Kołobrzeg

92. W jakiej bitwie stanęli naprzeciw siebie jako głównodowodzący generałowie: Reynier i Stuart? - Pod Maidą 4 lipca 1806.

93. Nie tak bardzo znany, ale z wojskowego punktu widzenia chyba cenniejszy był inny pułk "cudzoziemski" utworzony w 1813 roku i walczący w szeregach armii pruskiej. O jaki pułk chodzi? - Pułk huzarów meklemburskich

94. W listopadzie (23) 1798 roku wojska neapolitańskie rozpoczęły ofensywę skierowana przeciwko Francji. Kiedy i gdzie doszło do pierwszego spotkania polskich legionistów z Neapolitańczykami? - 1 grudnia pod Magliano.

95. Co stało się z dywizją markiza de la Romany, po tym jak Hiszpania przestała być francuskim aliantem? - Hiszpanie przy pomocy Anglików wydostali się z Danii statkami do ojczyzny. 8 tys. uciekło, 5 tys. poszło do francuskiej niewoli. Dywizja była potem 5 Dywizją Armii Galicji.

96. Pruska landwehra podzielona była według prowincji w których była formowana. Proszę wymienić nazwy dziewięciu prowincji-okręgów wojskowych które w latach 1813-1815 wystawiły pułki landwehry i nadały im tym samym swe nazwy. - Prusy Wschodnie, Prusy Zachodnie, Pomorze, Śląsk, Neumark, Kurmark, Westfalia, okręg Łaby, Nadrenia

97. Jaką twierdzę Polacy mieli zdobywać, zatknąwszy swoje nakrycia głowy na bagnety? - San Leo

98. W 1808 roku Napoleon miał pomysł reorganizacji piechoty Legii Nadwiślańskiej. Pomysł był moim zdaniem kuriozalny i uniemożliwiłby (albo przynajmniej bardzo utrudnił) dowodzenie jednostką na polu bitwy. Na czym miała polegać ta reorganizacja? - Napoleon projektował stworzyć ze wszystkich pułków piechoty LN jeden wielki regiment z 40-oma kompaniami

99. Proszę o podanie dowódców obu walczących stron w potyczce pod Labiawą (Labiau) - Generałowie Bachelu i Szepielew

100. W 1813 roku litewskie pułki ułanów wcielono do pewnego korpusu posiłkowego, a następnie w 1814 proponowano Litwinom przejście na służbę jednego z państw koalicji. Cóż to był za korpus posiłkowy i do jakiej armii chciano potem wcielić Litwinów? - Korpus duński, do Szwecji.

_________________
"W polityce głupota nie stanowi przeszkody."
Napoleon


Góra
 Zobacz profil  
 
PostNapisane: 3 lut 2010, o 21:41 
Offline
Général de brigade
Général de brigade
Avatar użytkownika

Dołączył(a): 3 lut 2010, o 06:45
Posty: 1248
Posty użytkownika Oudinot

PYTANIA o OSOBY.

176. Kto zastąpił na czele 2 dywizji generała Frianta po tym, jak został ranny pod Możajskiem? - Francois Dufour (1765-1832)

177. Jaki hinduski książę dowodził 27 marca 1799 pod Mallavelley w Indiach, kiedy pułkownik Arthur Wellesley atakiem na bagnety rozstrzygnął losy bitwy? - Tipu Sahib.

178. Kto towarzyszył Karolowi Lasalle, gdy ten po raz pierwszy zameldował się przed gen. Bonaparte w Armii Italii? - Francois Kellermann

179. Kto, z rozkazu pierwszego konsula, próbował dostarczyć do Egiptu posiłki w 1801 r.? - Admirał Honore Joseph Ganteaume.

180. W roku 1779 10-letni Arthur Wellesley miał następującą przygodę. Otóż w upalny dzień przechadzając się brzegiem Tamizy spotkał kąpiącego się chłopaka. Postanowił zrobić mu kawał i zaczął rzucać w niego grudkami ziemi. Kąpiący zdenerwował się i po wyjściu z wody nawiązał bójkę z Wellesleyem. Niestety Arthur okazał się silniejszy i pokonany musiał ratować się ucieczką do Tamizy. W przyszłości obaj gentelmani spotkali się podczas walk Wellingtona w Indiach. Kim był adwersarz Wellesleya i jakie w Indiach zajmował stanowisko? - Robert Smith, prokurator generalny Bengalu.

181.Jak nazywał się generał, który w połowie 1813 r. organizował ostatnią już napoleońską Armię Włoch, a potem był jednym z podkomendnych ks. Eugeniusza w kampanii 1813-1814? - Paul Grenier (1768-1827)

182. Jaki generał odznaczył się w bitwie pod Jeną i pod Gołyminem, a zmarł w lazarecie polowym w Landsberg na skutek ran odniesionych w bitwie pod Preussisch Eylau? - Jacques Desjardin

183. W bitwie pod Brienne (29 I 1814 r.) poległ pewien admirał francuski. Jak się nazywał i jakie stanowisko zajmował w dniu batalii? - Pierre Baste. Był generałem brygady i dowodził jedną z dywizji Młodej Gwardii

184. Był synem kupca, urodził się 9 września 1768 roku. mając 15 lat zaledwie wstąpił do jednego z regimentów piechoty, gdy wyszła na jaw prawda o jego wieku został wyrzucony z armii. Niespełna dwa lata później znowu ląduje w piechocie. Jako że był lubiany szybko awansuje na kolejne szczeble kariery. W końcu ląduje jako szef sztabu Augereau i zostaje generałem brygady. W listopadzie 1795 roku wysłany do Armii Włoch bierze udział w wielu potyczkach w tym m.in. pod Lodi i Castiglione, w tej ostatniej stracił życie. O jakiej osobie mowa? - Martial Beyrand

185. Który z generałów dostał sie w 1799 roku do niewoli korsarzy tunezyjskich? - Saint Michel Lacombe (1751-1812)

186. Kto był dowódcą rozbitej w roku 1797 w potyczkach pod Castel Bolognese, nad rzeką Senio ( 03.02.) oraz pod Anconą (Loreto) (09.02) Armią Państwa Kościelnego? - Baron Colli.

187. Kto zażądał od Napoleona odbudowania Polski "od Rygi do Odessy i od Gdańska do Węgier" oraz chciał by miała ustrój "taki jak w Anglii"? - Tadeusz Kościuszko.

188. Był bodajże najmłodszym generałem naszego okresu: w chwili nominacji liczył 18 lat. Był członkiem arystokratycznego rodu, autorem ciekawych "Pamiętników" i odegrał po 1815 r. ważną rolę w dziejach Francji. Kto to? - Ludwik Filip Orleański, książę Chartres, późniejszy król Francji.

189. Jaki wybitny francuski generał, zmarł w niecałe trzy lata po odniesieniu rany nad Berezyną? - Claude-Juste Legrand

190. Starzy wiarusi legionowi, uwielbiali jednego z generałów francuskich i darzyli go wielką sympatią, nazywali go "najstarszym Bartkiem". O kogo chodzi? - Generał Napoleon Bonaparte

191. W żyłach księcia Karola i jego synów z lady Di - a więc następców tronu brytyjskiego, płynie niewielka domieszka krwi polskiej. Otóż owa domieszka pochodzi od polskiego generała z czasów napoleońskich. Jakiego? - Maurycy Hauke

192. Wybitny dowódca kawalerii, generał brygady (1807), generał dywizji (1812), uczestnik wszystkich kampanii 1806-1815. Zięć marszałka Oudinot. O kogo chodzi? - Claude-Pierre Pajol

193. Po bitwie pod Kanopą doszło do znacznych rozdźwięków wśród generalicji Armii Wschodu. Jeden z oficerów ostro krytykował głównodowodzącego. Został za to z jego rozkazu aresztowany przez jednego ze swoich kolegów i odesłany do Francji. Po powrocie Francuzów nasz bohater wyzwał adwersarza na pojedynek i zabił go, za co został odsunięty od wszelkich funkcji na kilka lat. Jak nazywał się ów krewki generał i kogo pozbawił życia? - Generałowie Reynier i Destaing

194. Jak nazywał się oficer francuski, przysięgający 10 IV 1796 r. bronić do śmierci ze swoimi żołnierzami reduty Montelezzimo przed nagłym atakiem Austriaków, za co już następnego dnia został mianowany przez Bonapartego gen. brygady? - Antoine Guillaume Rampon.

195. Jak nazywał się polski żołnierz, którego zgon poprzedził swoją śmiercią pójście ta drogą marszałka Bessieresa pod Weissenfels? - Wachmistrz Jordan

196. Który z podanych poniżej generałów nie brał udziału w kampanii rosyjskiej 1812 roku: Castex, Clausel, Curial, Delzons, Dessolle, Dommanget, Joubert, Ledru, Pire, Razout? - Bertrand Clauzel

197. Jak nazywał się generał dowodzący oddziałami Mulatów wiernych Francuzom w kampanii 1802-03 i wcześniej? - Andre Rigaud

198. Pewien młody porucznik jako adiutant Napoleona meldował mu coś na polu bitwy i kiedy uchylił kapelusza, pocisk wyrwał mu go z ręki. Napoleon ponoć uśmiechnął się i rzekł: "Dobrze, że nie jest pan wyższy, prawda?" Który z polskich oficerów był bohaterem tegoż epizodu? - Dezydery Chłapowski

199. Który z polskich wyższych oficerów z czasów Księstwa Warszawskiego miał swe korzenie na w Irlandii? - Ignacy Blumer

200. W kampanii 1809, w dniach 20-23 kwietnia zginęli w bitwach szefowie sztabu dwóch korpusów Wielkiej Armii (Armii Niemiec). Jak się nazywali i do jakich korpusów należeli? - Generałowie Cervoni z II Korpusu Lannesa i Hervo z III Korpusu Davouta.

201. Szwagier Wellingtona, generał w wojnie na Półwyspie, poległ w 1815 r, lecz nie w kampanii belgijskiej. Kto to był i w jakich okolicznościach pożegnał się z tym światem? - Edward Pakenham (1778-1815), zginął pod Nowym Orleanem w bitwie z Amerykanami.

202. Na przełomie 1794 i 1795 roku, panowała w Europie wyjątkowo sroga zima. Część floty holenderskiej, została unieruchomiona przez lody, co wykorzystali Francuzi, zdobywając te jednostki w szybkim i bezczelnym ataku. Przy straszliwym mrozie udali się oni na zamarznięte wody zatoki przy wysepce Texel. Kto dowodził tymi jednostkami, jaki miał stopień? - Major Louis Joseph Lahure

203. Ostatnia potyczka wojen napoleońskich odbyła się pod Bavilliers 8 VII 1815 r. Kawaleria francuska szarżowała tam austriackie czworoboki ze skutkiem jak najbardziej pozytywnym. W ataku brał udział dowódca Korpusu Obserwacyjnego Jury gen. Lecourbe. Kto dowodził dywizją kawalerii, na czele której Lecourbe wykonał swój ostatni atak? - Generał Bertrand Pierre Castex

204. Jacy byli napoleońscy ministrowie kultów religijnych? - Jean Etienne Portalis i Felix Julien Jean comte Bigot de Preameneu

205. Kto otrzymał po marszałku Bernadotte tytuł: prince de Ponte Corvo? - Napoleon Lucjan Murat.

206. Kto był autorem "Mów do narodu niemieckiego"? - Johann Gottlieb Fichte.

207. Jacy byli kolejni rezydenci napoleońscy w Księstwie Warszawskim? - Etienne Vincent, Jean Serra, Louis Bignon, Dominique Pradt, Bignon

208. 3 maja 1803 roku w warszawskiej Pomarańczarni odbyło się przedstawienie utworu dramatycznego "Zaślubiny Mieszka z Dąbrówką", w której główną rolę zagrał pewien młody człowiek. Tą rolą (między innymi) zdobył serce i rękę pasierbicy marszałka Małachowskiego. W grudniu 1806 roku gen. Dąbrowski mianował go pułkownikiem nieistniejącego jeszcze 3. pułku szaserów. W Warszawie, wiosną 1807 odniósł swoją pierwszą ranę w wojnach napoleońskich: w bijatyce ulicznej (niektórzy mówili o pojedynku) rozkwaszono mu nos. Niemcewicz pisał o nim w swoich "Pamiętnikach": "...we wszystkich przygodach przychylny temu, co w obecnej chwili mocniejszy". O kim mowa? - Wincenty Krasiński.

209. Na początku 1800 r. Napoleon wysłał do Armii Renu zaufanego oficera, aby ten wydał opinię o stanie zaopatrzenia i morale wojsk. Była to misja, o której nie wiedział gen. Moreau. Kim był ten oficer? - Geraud Duroc

210. Kto był dowódcą dywizji lekkiej kawalerii 5. korpusu w kampanii 1805? - Anne Francois Charles Treillard

211. Kto i kiedy stanął na czele Uniwersytetu cesarskiego? - Louis Marcelin de Fontanes w 1808 roku.

212. Podczas bitwy pod Waterloo ranny został polski oficer służący w sztabie cesarskim. O kim mowa? - Józef Giedroyć.

213. Kto był głównodowodzącym tzw. Armii Bobru? - Marszałek Macdonald.

214. Jeden z "nieśmiertelnych" w momencie powstania Wielkiej Armii nie był dywizjonerem. Proszę podać jego nazwisko, a także nazwisko dywizjonera po którym przejął w 1805 funkcję i co się z poprzednikiem naszego bohatera stało. - Charles-Antoine Morand, poprzednik to Ludwik Cafarelli.

215. W kampanii 1813 pod Yorkiem służył brat pewnej monarchinii. Kim był i kim dowodził w Pruskim Korpusie Armii? - Karl von Mecklenburg-Sterilitz, dowódca 2 Brygady.

216. Eugeniusz Beauharnais wziął ze sobą na kampanię 1812 r. artystę, który miał dokumentować bohaterskie czyny jego, oraz korpusu włoskiego. Artysta ten pozostawił najlepszy chyba wizerunek wojny 1812 r. i tragicznego odwrotu. Jak się nazywał? - Albrecht Adam.

217. W 1792 r. pierwszym sukcesem francuskim w walce z wojskami koalicji była udana obrona Thionville. Kto był wówczas komendantem twierdzy i jakie były jego związki z Polską? - Załogą Thionville dowodził syn szambelana Stanisława Leszczyńskiego, Georg Felix von Wimpfen.

218. W korpusie ochotniczym von Lützowa w roku 1813 walczyła również kobieta. Jak brzmiało jej imię i nazwisko? - Eleonora Prochaska

219. W 1812 roku w ręce żołnierzy Marmonta w Hiszpanii wpadł pewien angielski oficer, który zapuścił się za daleko w stronę pozycji francuskich. Oficer ten wypełniał bardzo ważne zadania dla wojsk angielskich na półwyspie pirenejskim. Jak się nazywał, jaką sprawował funkcję oraz jakie były jego koleje losów zanim powrócił do swojej ojczyzny - Anglii? - Major Coquhon Grant, szef wywiadu Wellesleya na półwyspie pirenejskim. Po wzięciu do niewoli, zbiegł po dotarciu do Francji, udał się do Paryża przebrany za amerykańskiego oficera. Tam wykorzystując brytyjską siatkę szpiegowską nadal dostarczał informacje Wellesley'owi. Ostatecznie umknął na łódce rybackiej i powrócił od Wellingtona.

220. Jaki Polak po przejściu na dłuższy czas do rezerwy, zdjął z frontonu swego domu herb i powiesił w jego miejsce zegar? - Dezydery Chłapowski

221. Kto dowodził Francuzami w bitwie nad rzeką Coa? - Michel Ney

222. Kim był i do czego się przyczynił Francisco Zea Bermudez? - W latach 1812 -1820 był charge d'affaires ambasady Hiszpanii w Petersburgu. Mianowany przez Ferdynanda VII pierwszym ministrem w 1824 r. Przyczynił się do odnowienia nawiązania stosunków dyplomatycznych między Rosja a Anglią, zerwanych po podpisaniu pokoju w Tylży.

223. Jak nazywał się prefekt Rzymu, za którego urzędowania nastąpiła większość reform administracyjnych i gospodarczych przeprowadzonych w Wiecznym Mieście, stawiających starą stolicę Cesarstwa na wyższym poziomie rozwoju cywilizacyjnego, a cofniętych zaraz po powrocie Piusa VII? - Philippe-Camille-Marcellin-Casimir, hrabia de Tournon-Simiane

224. Jaki znany Polak służył (krótko) w 1813,w stopniu kapitana, w cieszącej się nie najlepszą opinią Gwardii Honorowej?- Aleksander Fredro.

225. Wraz z gen. de Bourmont przed bitwą pod Waterloo na stronę wroga przeszło dwóch jego oficerów sztabowych. Jak się nazywali? - Płk Clout i mjr Villoutreyes

226. W trakcie rozmów o kapitulacji Warszawy w 1809 doszło do wymiany zakładników. Zakładnikiem ze strony polskiej był płk Szumlański zaś austriackiej oficer, który krótko później nie popisał się w walce z naszymi żołnierzami, a parę lat później zapisał się w historii epoki. Proszę podać jego nazwisko. - Płk Neipperg

227. Był z Napoleonem na wyspie Św. Heleny. Wziął udział w powstaniu listopadowym w roku 1831. O kogo chodzi? - Francois Antomarchi

228. W lipcu 1812 roku jeden z oficerów 1.pułku szwoleżerów gwardii został oficerem ordynansowym Cesarza. Jak się nazywał? - Stanisław Wąsowicz-Dunin.

229. Trzech oficerów 1. pułku szwoleżerów otrzymało nominację na wyższe stopnie bezpośrednio na polu walki. Kto? - Datauncourt, Kozietulski, Dobiecki

230. Dawny towarzysz broni Xiecia Józefa, w 1813 roku ponoć mieli sie pojedynkować. "(...) Słyszycie, wódz polski nie żyje!- słysząc to X schwycił sie za czoło, powtarzał: - Ach mój Boże! Ach mój Boże - i na bok odjechał." Kim był? - Karol hr. Schwarzenberg

231. Kto dowodził Saksończykami pod Raszynem? - Generał Polentz

232. Jak nazywał się dowódca dywizjonu huzarów austriackich biorących udział w potyczce pod Kockiem? - Major Hoditz

233. 22 września 1812 r. wydano odezwę zapowiadającą utworzenie na Litwie pułku strzelców konnych złożonego z samych ochotników. Ze względu na mały stan osobowy połączono strzelców z 21 pułkiem ułanów. Kto dowodził owym pułkiem strzelców konnych? - Ignacy Moniuszko

234. W czasie kampanii 1807 pewien głównodowodzący zginął z ręki polskiego chłopa. Kto? - Aleksander Kamieński.

235. Jak nazywał się oficer - Polak, który podobno za rosyjskie pieniądze ofiarował się zabić Napoleona? - Hieronim Pągowski

236. Dekret królewski z dnia 25.02.1809 roku informował o przyznaniu dwóm osobom Wielkiego Krzyża Wojskowego Ks. Warszawskiego. Jedną z tych osób był ks. Poniatowski. Kto jako drugi otrzymał to odznaczenie? - Louis Nicolas Davout.

237. W jednym z pojedynków Józefa Bema pewien "zbyt lubiący kieliszek" oficer trafił go w udo. Kula ta mimo wielu starań utkwiła w ciele naszego "Mahometanina" na zawsze. Jak się nazywał ów oficer? - Szymon Nowicki

238. Kto stał na czele związku korespondencji legionowej? - Franciszek Rymkiewicz

239. Postać dosyć tajemnicza. Romantyk i awanturnik. Prawdopodobnie najbardziej zaufany współpracownik Dąbrowskiego w okresie legionów. Przez pewien czas pełnił funkcję pełnomocnika Legionów przy rządzie Cisalpiny. Jednak przede wszystkim był głównym realizatorem koncepcji jońskiej. Na jego kącie były tez inne akcje dyplomatyczno-szpiegowskie. O kim mowa? - Józef Chamand

240. Jak się nazywał główny szpieg rosyjski w Paryżu w roku 1812? - Aleksander Czernyszew

241. Jak nazywał się ulubieniec Hieronima Bonaparte, który dostał "bęck"? od "czarnej bandy"? - Generał Rewbell

242. W czasie kiedy I Legia ścierała się w 1799 r. nad Trebbią z koalicjantami, w środowiskach emigracyjnych ukazała się broszura "Sur Dombrowski" atakująca generała za rzekome spiskowanie z Rosjanami i zaprzedanie wojsk polskich Suworowowi. Jej redaktorami była trójka polskich "jakobinów". Kto? - Józef Kalasanty Szaniawski, Franciszek Ksawery Dmochowski i Józef Neyman

243. Kto otrzymał ostatnią nominację generalską epoki napoleońskiej? - Gaspard Gourgaud

244. Był dowódcą 2 pułku piechoty 1."Warszawskiej"Legii. Przyjaciel Xięcia Józefa. W kampanii 1812 dowodził brygadą. Czyim był synem i w jaki sposób zginął? - Syn Szczęsnego Potockiego, Stanisław Potocki zginął w noc listopadową.

245. Skromny nauczyciel nazwiskiem Krzysztof Suhr - pod jakim pseudonimem jest znany i czego takiego specjalnego dokonał? - Mieszczanin z Hamburga, namalował wszystkie jednostki przechodzące przez miasto.

246. Kto sprawował funkcję prefekta Pałacu Rządowego, czyli Tuileries? - Jean Baptiste Charles Legendre de Lucay

247. Który z generałów nosił przydomek "człowieka który ocalił Kanadę"? - Izaak Brock

248. W bitwie pod Hanau oprócz 2 p.p. wyróżniła sie także jedna z polskich baterii, wstrzymywała ona kartaczami atak piechoty bawarskiej. Kto dowodził tą baterią? - Franciszek Piętka

249. Pierwsze kroki swojej kariery wojskowej stawiał w pułku "Royal-la-Marine", stacjonującym na Korsyce. W wieku 25 lat, w roku 1788 dorobił się stopnia sierżanta. To właśnie wtedy, pewien handlarz jedwabiu odmówił mu prawa do kwaterowania w swoim domu, pod pretekstem, że jego pokoje zarezerwowane są dla oficerów. Nie spodziewał, że za dziesięć lat połączeni zostaną więzami rodzinnymi. I chociaż wojskowy, zajmował również stanowiska cywilno-dyplomatyczne. Osiągnął niemalże wszystko, co w cesarskiej Francji było do osiągnięcia i nie wiadomo, jak potoczyłyby się jego dalsze losy, gdyby któregoś dnia nie stanął na jego drodze Karl Otto Mörner. Kto to? - Marszałek Bernadotte

250. Był amerykańskim dyplomatą i poetą. W 1812 r. miał udać się, jako poseł USA, do Cesarza do Wilna, ale plany pokrzyżowała katastrofa rosyjska. W drodze powrotnej zachorował i zmarł na terenie Polski. Pochowany w Żarnowcu. O kogo chodzi? - Joel Barlow

251. Kto nazwał młodego Napoleona Buonaparte "kot w butach"? - Cecile Permon

252. W początkach XIX wieku trafił do twierdzy Kufstein oficjalnie za to, że obraził pewnego arystokratę, choć ponoć był to tylko pretekst, w rzeczywistości za zuchwałe pronapoleońaskie wypowiedzi. 18 maja 1804 roku uciekł z twierdzy i przez Bawarię i Saksonię dotarł na Śląsk. Kto to? - Jan N. Sułkowski

253. W 1798 Chłopicki był sekundantem w pojedynku. Zginął w nim mjr Zabłocki, kolega Chłopickiego. Osoba jakiego wyższego oficera była powodem tego pojedynku? - Jan Henryk Dąbrowski

254. Francuski garnizon pewnej twierdzy przed jej opuszczeniem i przebiciem się przez linie oblężnicze wysadził umocnienia twierdzy. Kto dowodził tym garnizonem? - Generał Brenier

255. W 1813 roku dowództwo jednej z brygad kawalerii objął generał o polsko brzmiącym nazwisku. Kto to? - Louis Charles Barthelemy Sopranski

256. Ks. Józef mianował go po wkroczeniu do Galicji generalnym organizatorem siły zbrojnej krakowskiego i kieleckiego. Był też niby-ministrem wojny w niby-rządzie Galicji. Kto to? - Józef Wielhorski

257. Gdy w wyniku trudów kampanii zimowej zachorował feldmarszałek Kutuzow, kto był p.o. NW Armii Rosyjskiej do momentu objęcia tej funkcji przez Barclay'a de Tolly'ego ? - Aleksander Tormasow

258. Jaki inny, młody wódz wkroczył na arenę wojen w 1796, swą karierę rozpoczął niemal tak błyskotliwie jak Bonaparte: od zwycięskich bitew nad doświadczonym i sławnym generałem, a zakończył ją równie smutno i w tym samym okresie? - Arcyksiążę Karol

259. 26. grudnia 1799 roku Bonaparte spotkał się z człowiekiem o nazwisku d'Andigne. Kim był ten człowiek? - Był emisariuszem rojalistowskim, chodziło o rozmowę na temat szuanów i propozycję rozwiązania tego problemu poprzez nadanie przywódcom epoletów generalskich i przejście na służbę Republiki.

260. Młodszy o dwa lata od Napoleona, dowodził pułkiem kirasjerów, w roku 1793 otrzymał rangę generała dywizji, niedługo po Wagram dostał tytuł hrabiego cesarstwa, w późniejszej historii Francja miała marszałka o takim samym nazwisku. Kto to? - Charles-Joseph Randon de Pully

261. Kto był autorem "Traktatu o ekonomii politycznej? - Jean Baptiste Say

262. W 1812 roku na powitanie Napoleona w Wilnie utworzona została gwardia honorowa. Kto stał na jej czele? - Gabriel Ogiński

263. Kto zastąpił Eliasza Tremo na stanowisku dowódcy pułku jazdy Legionów? - Aleksander Karwowski

264. Kto był przedstawicielem Rządu Narodowego w Paryżu, w trakcie Powstania Listopadowego? - Karol Kniaziewicz

265. Kto był pierwszym dowódcą pułku jazdy Legionów i w jakich okolicznościach pożegnał się z życiem? - Eliasz Tremo został spalony żywcem w niewoli.

266. Wraz z trzema pułkami piechoty armii Księstwa Warszawskiego do Hiszpanii wyruszyła jeszcze jedna jednostka Księstwa. Kto był jej dowódcą? - Antoni Kamiński

267. Jak nazywał się osobnik, który w przebraniu rosyjskiego chłopa miał dokonać w Moskwie zamachu na Napoleona? - Aleksander Figner

268. Jak nazywał się żołnierz, który zabił w boju księcia Ludwika Ferdynanda Pruskiego pod Saalfeld 10 października 1806 r.? - Marechal de logis Guindey

269. Jak nazywał się odznaczony Legią Honorową oficer wojsk Księstwa Warszawskiego - weteran kampanii od 1807 do 1813 (następnie w armii Królestwa Polskiego i nie tylko), który pod koniec swego niezwykle barwnego żywota był sygnatariuszem I Międzynarodówki? - Ludwik Oborski

270. Kto dowodził "wielką baterią" pod Wagram, która wspierała ogniem "kolumnę Macdonalda" w czasie ataku? - Bernard Lauriston

271. Po zdobyciu Malty Napoleon powiększył obsadę Komisji Nauki i Sztuki o kilka osób pochodzących z tej wyspy. Jedną z nich był potomek cesarzy bizantyjskich. Kto? - Theodore Lascaris

272. Kto dowodził w 1814 r. 1 p. grenadierów Gwardii, którego sztandar ucałował Cesarz w czasie pożegnania na dziedzińcu pałacu Fontainebleau? - Generał Petit

273. Napoleon spotkał go po raz pierwszy w 1796 r., kiedy nasz bohater był konsulem Francji w Turynie (partycypował przy rozejmie w Cherasco). Następnie został organizatorem administracji na odzyskanej przez Francję - Korsyce. W późniejszym okresie pełnił podobne funkcje w Neapolu przy Józefie, aby zostać w końcu ministrem spraw zagranicznych Józefa. O kogo chodzi? - Antoine Francois Miot de Melito

274. Jeden z naszych wojaków-pamiętnikarzy będąc w angielskiej niewoli spędził czas jakiś na pięknej skalistej Świętej Helenie. O kim mowa? - Andrzej Daleki

275. Legionista, uczestnik wyprawy na San Domingo, pod Albuerą, będąc oficerem sztabowym V korpusu, przyniósł Konopce rozkaz do słynnej szarży. Kto to? - Jakub Kierzkowski

276. Przemytnik angielski, który zawodowo zajmował się przerzucaniem oficerów francuskich przebywających w niewoli angielskiej. Cena: 100 funtów za czterech oficerów. Znany pod aliasem Herbert. Jakie było jego prawdziwe nazwisko? - Thomas Moore

277. Który z generałów napoleońskich miał w swoim nazwisku rodowym człon Casanova? - Jean-Touissant Arrighi

278. 8 grudnia 1809 roku książę Jozef Poniatowski, został ojcem drugiego już syna. Jak go nazwano i kim była jego matka? - Był to Józef Ponitycki, a jego matką była Zofia Czosnowska pv. Potocka

279. Kogo nazywano "pierwszym grenadierem Francji"? - Theophil-Malo Corret de La Tour d'Auvergne

280. Jak nazywał się najwyższy rangą oficer korpusu arcyksięcia Ferdynanda poległy podczas kampanii 1809 r.? Gdzie i w jakich okolicznościach stracił życie? - Pułkownik Brusch, szef sztabu dywizji straży przedniej gen. Mohra. Poległ 15 maja podczas ataku na przyczółek mostowy pod Toruniem, od polskiego kartacza.

281. Jak się nazywał oficer francuski, który zaraz po amputacji ramienia, kazał przywiązać się do konia i kontynuował walkę? Podczas jakiej bitwy miało miejsce to zdarzenie? - Jean Baptiste Sourd pod Genappe

282. Jak nazywał sie oficer francuski, prowadzący roboty oblężnicze pod Saragossą w 1809 r.? - Andre Lacoste

283. Kto był gubernatorem Erfurtu podczas kongresu władców w 1808? - Nicolas Oudinot

284. Kto został ministrem wojny Republiki zaraz po zamachu 9-10 listopada 1799 r. i jak długo urzędował? - Louis Alexander Berthier - od 10 listopada do 2 kwietnia 1800

285. Kto dostarczył Napoleonowi 5 zdobytych pod Albuerą sztandarów? - Chef d'escadron Lafitte

286. Kto podczas wojny 1809 roku przekonywał księcia Józefa, że "zajęcie Warszawy stanie się źródłem niepowodzeń Austriaków"? - Generał Pelletier

287. Kto dowodził obroną linii rzeki Var w kwietniu-maju 1800 r., powstrzymując atak Austriaków na Prowansję? - Louis Suchet

288. Kto dowodził obroną Kairu w 1801 r., podpisując jego kapitulację w lipcu tego roku? - Augustin Belliard

289. Jak się nazywał oficer francuski, który dostał sie do niewoli rosyjskiej w sierpniu 1813 roku, przesłuchiwany przez samego cara Aleksandra I-go, zasłynął z butnej odpowiedzi. Na zarzut rozboju zaripostował: "Przynajmniej ja nigdy nie byłem oskarżony o zabójstwo mojego własnego ojca ". - Dominique Vandamme

290. Generał Remy Joseph Exelmans będąc w bliskich stosunkach z Muratem w grudniu 1814 napisał list do niego. Ów list został przejęty przez policje i wyniku tego Exelmans został przeniesiony w stan spoczynku i zmuszony w 24 godziny opuścić Paryż. Jednak czując się niewinny odmówił posłuszeństwa. W związku z tym został zatrzymany 20 grudnia i postawiony przed sad woskowy 16-go okręgu woskowego w Lille. Został oskarżony o kontaktowanie sie z wrogiem (Murat), szpiegostwo i obrazę króla. 14 stycznia 1815 został osadzony w cytadeli w Lille. Kto przewodniczył sadowi nad Exelmansem? - Drouet d'Erlon

291. Kto, na krótko przed wkroczeniem sprzymierzonych do Paryża 31 marca 1814 r., wydał rozkaz spalenia sztandarów zdobytych przez Francuzów w wojnach Rewolucji i Cesarstwa, a przechowywanych w Pałacu Inwalidów? - Jean Mathieu Serurier

292. Kto był ministrem wojny w Hiszpanii (w rządzie króla Józefa) w latach 1808-13? - Gonzalo O?Farril Herrera z małą przerwą od 23.04.1811 do 15.07.1811 dla José Martínez Hervás

293. Który z generałów XW miał tatarski rodowód? - Łukasz Biegański

294. Ilu było generałów w Księstwie Warszawskim pochodzenia mieszczańskiego i proszę o ich nazwiska? - Filip Hauman, Maurycy Hauke i Józef Rautenstrauch

295. Kim był hrabia Mosloy? - Louis Otto hr. Mosloy, z pochodzenia Badeńczyk, dyplomata w służbie Francji, prowadził rozmowy pokojowe z Anglikami przed traktatem w Amiens.

296. Jednym z ostatnich kroków poczynionych przez gen. Bonaparte przed opuszczeniem Egiptu było utworzenie dwóch, 12-osobowych komisji do badań nad Górnym Egiptem. Proszę o podanie nazwisk przewodniczących tych komisji? - Costaz i Fournier

297. Poseł na Sejm Czteroletni, generał, w 1794 bierze udział w insurekcji, ośmielił się w 1795 roku wyzwać na pojedynek faworyta Katarzyny II Płatona Zubowa, bierze udział w wojnie 1806/7. Kto to jest ? - Ignacy Giełgud

298. Jaką rolę pełnił w środowisku emigrantów antyrewolucyjnych Wiliam Wickham? - Był szefem siatki szpiegowskiej w Szwajcarii oraz skarbnikiem emigrantów

299. Kto był szefem sztabu Korpusu Neya w bitwie po Lützen? - Louis Ann Marie Goure

300. Kto bronił w 1801 r. Aleksandrii? - Generał Menou

301. Kogo nazywano "wujaszkiem Jerzym"? - Tio Jorge

302. 25. XII. 1800 r. w Steyer podpisano zawieszenie broni francusko-austriackie. Kto podpisał je ze strony francuskiej, a kto austriackiej? - Francja - gen. Lahorie i Austria - gen. Grune

303. "Był synem rękawicznika z Grenoble i zażartym bonapartystą. 23 września 1814, mając 28 lat, potajemnie wylądował w Potro-Ferrajo z zamiarem konspirowania na rzecz Cesarza. Nie tylko zapewnił wygnańca o dobrym nastawieniu mieszkańców Grenoble do jego osoby, ale miał także, przy pomocy map wskazać najlepszą drogę do stolicy Delfinatu." O kim mowa? - Jean Demoulin

304. Generał francuski, dobry znajomy Cesarza, artylerzysta i inżynier, między innymi był pod Austerlitz i Jeną, zaliczył Hiszpanię, "posiadł umiejętność budowy jednostek pływających". Zmarł z wyczerpania i chorób po kampani 1812. Proszę o nazwisko jegomościa? - Jean Ambroise Baston de Lariboisiere

305. Szwagier Antoniego Sułkowskiego, adiutant Rożnieckiego, później gen. w Królestwie Polskim. Jak się nazywał ? - Ludwik Kicki

306. Uczony, ur. w Turynie, założył tam w 22 roku zycia Towarzystwo Naukowe. Przez 21 lat prezesuje Akademii Nauk w jednej ze stolic europejskich. Do Francji ściąga go Ludwik XVI. Napoleon nazywał go piramidą nauk. Mianowany senatorem, wielkim oficerem LH i hrabią Cesarstwa. Kim była owa piramida nauk? - Joseph Louis Lagrange.

307. Uczestnik wojny o niepodległość Ameryki, francuski marechal du camp, emigrant, 1797 - generał armii Królestwa Neapolu, w latach 1806-1815 w służbie króla Józefa i króla Joachima. O kogo chodzi? - Generał Jan Daniel Gambs

308. W 1808 r. mianowano nowego osobistego lekarza kardynała Fescha. Był on wynalazcą pewnego niezwykle użytecznego narzędzia medycznego. Kto to był? - Rene T.H. Laennec wymyślił stetoskop.

309. Jak nazywała się z domu matka Józefa Sułkowskiego? - Małgorzata Zofia de Fleville?Sułkowska

310. Generał, prawnik, był w Egipcie, ranny pod Austerlitz, uczestnik wojny 1806/7, 1809, 1812 dow. dywizji, 1813 dow. dywizji, w 1814 dowódca obrony jednego z miast, czynny w 1815, miał mieć związki z Polską. Kto on? - Charles Antoine Louis Morand.

311. Ustanowiony przez Napoleona w 1809 roku order Trois Toisons D'Or posiadał tylko jeden polski oficer. Kto nim był? - Maksymilian Fredro.

312. Był adiutantem Zajączka, w kwietniu 1808 szef szwadronu, we wrześniu 1808 major pułku, w kwietniu 1812 pułkownik. Poległ pod Możajskiem. Kto to był? - Aleksander Radzimiński

313. Kogo zwano "generałem bez skazy i zmazy"? - Latour-Maubourg

314. Kogo Napoleon nazywał "małą dżumą"? - Laura Permon

315. Schwytany pod Waterloo przez Prusaków, stojąc już przed plutonem egzekucyjnym, został rozpoznany i ocalony przez jednego z brytyjskich oficerów. O kim mowa? - Jean-Dominique Larrey

316. W listopadzie 1809 roku ks. Poniatowski, za zgodą Rady Stanu, mianował komisarzy demarkacyjnych dla ustalenia przebiegu nowej granicy między Księstwem i Austrią. Jak się nazywai? - Ks. H. Lubomirski i gen. A. Rożniecki. Davout dodał jeszcze gen. Pelletiera

317. Jak nazywał sie najsłynniejszy kartograf Napoleona, dyrektor archiwum wojskowego ? - Louis-Albert Bacler d'Albe

318. Kto, na pewne niewygodne pytanie dotyczące przeszłości, odpowiedział: "żyłem"? - Sieyes

319. W jaki sposób Kacper Hauser mógł być powiązany z rodziną Bonaparte? - Najprawdopodobniej był synem Stefanii de Beauharnais

320. W kampanii 1809 brał udział w mundurze austriackim. W kongresie wiedeńskim uczestniczył w delegacji rosyjskiej. W powstaniu listopadowym walczył po polskiej stronie. Kto? - Wacław Rzewuski

321. Były oficer armii saskiej, potem instruktor w jednym z pułków gwardii cesarskiej. Blisko spokrewniony z jedną z najwybitniejszych osobistości Księstwa Warszawskiego? - Łukasz Wybicki

322. Jeden z generałów Księstwa spisał w swojej książce "Dwuletnie doświadczenia" rady co do sposobu prowadzenia gospodarki. Proszę o podanie nazwiska tego generała. - Karol Kniaziewicz

323. W roku 1812 Napoleon przysłał do Księstwa pewnego oficera, który miał zastąpić płk. Malleta na stanowisku dowódcy wojsk inżynieryjnych V korpusu. Proszę o podanie nazwiska tego oficera. - Płk. Bouvier.

324. Wraz z siedmioma innymi oficerami, w wieku 17 lat, dokonał na czele 300 dragonów głośnej szarży pod Villiers-en-Couche, ratując samego cesarza. Kilka lat później za ten wyczyn dostał order wraz z tytułem barona Cesarstwa. O kim mowa? - Robert Thomas Wilson

325. Generał francuski, walczył między innymi w Hiszpanii, związany przez pewien czas z Polakami. Zagorzały Bonapartysta, nie zaprzestał swojej działalności po 1815. W 1822 r. skazany i rozstrzelany. Kto to był? - Jean Baptiste Berton

326. W 1816 roku Czernyszew poślubił pewną piękną Polkę. Jak nazywała sie wybranka Czernyszewa? - Teofila Starzeńską z domu Morawska

327. W 1808 roku powołano w Księstwie komisarza rządowego, którego zadaniem było prowadzenie negocjacji w sprawie wytyczenia drogi militarnej, handlowej i pocztowej łączącej Księstwo Warszawskie z Saksonią. Proszę o podanie nazwiska owego komisarza. - Gen. por. A. Gorzeński

328. Zginął w wieku 22 lat. Napoleon, z którym się nigdy nie spotkał, kilkakrotnie miał westchnąć nad okrucieństwem historii: "do czego mógł dojść". Kto to? - Henryk Rochejaquelein

329. Jak nazywał się Polak, który według pewnych (być może tylko legendarnych) przekazów, odegrał pewną rolę podczas zamachu 18 brumaire'a w słynnej oranżerii w Saint-Cloud? I jaka to była rola? - Major Schaltzer zasłonił własną piersią Bonapartego 19 brumaire?a

330. Komu Napoleon podarował majątek opinogórski? - Marszałkowi Bernadotte.

331. "Los infernos picadores" walczyli w niefortunnej potyczce pod Jovenes (24 marca 1809). Lekkomyślny Konopka dał się zaskoczyć Hiszpanom i stracił sztandar pułku. Jak zwał się dowódca sił hiszpańskich w tej bitwie? - Wicehrabia Zolina

331. W 1798 r. wybuchło antyangielskie powstanie w Irlandii. Francuzi z trudem przerzucili za kanał ok. 1000 ludzi, którzy zbyt wiele nie zdziałali mimo iż zapuścili się nawet dosyć daleko w głąb zielonej wyspy. Kto dowodził tym oddziałem Francuzów? - Jean Joseph Amable Humbert

333. Generał Fiszer dostał sie do niewoli austriackiej pod Altenburgiem. Miał tam prosić hr. Wallmodena aby ten nie oddawał go pod eskortę oficera ułańskiego (ułani austriaccy byli w więkrzości polakami). Na pytanie Wallmodena "Dlaczego?" miał krótko odpowiedzieć- "nie dziwię się prostym żołnierzom Polakom, że służą w armii austriackiej, zostali bowiem do tego przymuszeni, ale każdego oficera Polaka, który zgodził się dobrowolnie służyć jednemu z trzech mocarstw [...] poczytuję za niegodnego tego miana". Za te ostre słowa został uwięziony i wolność odzyskał dopiero po zabiegach generała Moreau i wymianie za innego jeńca francuskiego. Kim był ów jeniec za którego Fiszera wymieniono? - Książę Lichtenstein.

334. Którzy z przyszłych marszałków Francji zajmowali w swojej karierze wojskowej stanowisko szefa sztabu Armii Italii? - Suchet, Berthier, Oudinot.

335. Po bitwie drezdeńskiej mieli Polacy otrzymać co nieco świeżych rekrutów poprzez wcielenie zagarniętych tam Polaków służących w wojsku austriackim Miał wtedy Napoleon osobiście rozmawiać z pochwyconymi oficerami, jednego z nich miał zapytać "I ty będąc Polakiem przeciw mnie służyłeś?" - "Tak jest najjaśniejszy panie" - padła odpowiedź. Na co Cesarz miał zdzielić tegoż oficera szpicrutą. Jak nazywał się ów oficer zdzielony przez Napoleona? - Ignacy Ledóchowski

336 W wyniku zaniechania ostrożności przez Konopkę w dniu 19 X 1812 pod Słonimiem rozbity został nowo tworzony pułk "złotych lansjerów". W wyniku pojawienia się kozaków gen. Czaplica musiały pośpiesznie umykać dwie generałowe - jakie? - Konopkowa i Dąbrowska

337. W swej bogatej karierze pełnił funkcję adiutanta Murata i fliegeladiutanta Fryderyka Augusta. Kościuszko zapisał mu w spadku 50000 fr., co Askenazy wiele lat później skwitował słowami: "niegodny pupil poczciwego Kościuszki, lichy charakter, lecz nie bez zdolności oficer". O kim mowa? - Franciszek Paszkowski

338. Jedną z najważniejszych lóż masońskich działających przy armii francuskiej byli "Bracia z Wielkiej Armii". Któż zajmował w niej urząd Pierwszego Dozorcy? - Wincenty Aksamitowski

339. W pewnym śląskim miasteczku, w lutym 1807 roku, dzielna Niemka Rosalia Joanna von Bonin wzięła do niewoli (z pomocą czterech żołnierzy) francuskiego generała brygady wraz z kasą wojskową zawierającą 70 tys. talarów. Jak ów generał się nazywał? - Jean Antoine Brun

340. Który z generałów napoleońskich w grudniu 1798 roku zajął Suez? - Generał Bon.

341. 25.03.1802 podpisany został traktat pokojowy w Amiens. Kto w imieniu obu stron złożył swoje podpisy na tym dokumencie? - Bonaparte, Cornwallis

342. W dniu 7. marca 1808 roku Legia Północna przekształcona została w 5. pułk piechoty Księstwa Warszawskiego. Francuscy oficerowie Legii, którzy stanowili 2/3 stanu oficerskiego otrzymali zezwolenie na powrót do Francji. Pozostał tylko jeden. Kto? - Kapitan Lemaire

343. Angielski generał, znanego powszechnie z odwagi a raczej heroicznej głupoty, po upadku Napoleona stał się zagorzałym bonapartystą. O kogo chodzi? - Robert Tomasz Wilson.

344. Pierwsza kobieta Kawaler Legii Honorowej, otrzymała też Order Św. Heleny. Jak się nazywała? - Angelique Brulon

345. Kto "ukrywa się" pod tytułem: comte d'Unsebourg ? - Dominique Joseph Vandamme.

346. Canova był autorem wspólnego grobowca ostatnich trzech przedstawicieli (w prostej linii) jednego z najstarszych rodów królewskich Europy. Jak mieli na imię i jaki to ród? - Jakub III, Karol i biskup Henryk Stuartowie.

347. Kto jest twórcą planu budowy Kanału Augustowskiego i na czyje polecenie ten plan wykonał? - Ignacy Prądzyński na polecenie gen. Hauke.

348. Jak się nazywał chirurg który amputował Horacemu Nelsonowi ramię? - Thomas Eshelby

_________________
"W polityce głupota nie stanowi przeszkody."
Napoleon


Ostatnio edytowano 3 lut 2010, o 21:50 przez Louis Nicolas, łącznie edytowano 2 razy

Góra
 Zobacz profil  
 
PostNapisane: 3 lut 2010, o 21:43 
Offline
Général de brigade
Général de brigade
Avatar użytkownika

Dołączył(a): 3 lut 2010, o 06:45
Posty: 1248
Post użytkownika Oudinot

PYTANIA RÓŻNE.

78. W jakich okolicznościach zginął generał Amadee Laharpe? - W nocy 10 maja 1796 został najprawdopodobniej zastrzelony przez swoich żołnierzy.

79. Jaka kara spotykała jeńca za próbę ucieczki z angielskiego pontonu? - Schwytanych wrzucano do ciemnego lochu na dnie pontonu.

80. W 1812 roku gen. Levebvre-Desnoettes uciekł z angielskiej niewoli, a związany był słowem honoru. Jak generał tłumaczył swe "niehonorowe" postępowanie? - Uznał, że do niewoli trafił przez podstęp przeciwnika, więc jego słowo honoru było nic nie warte.

81. W trakcie kampanii 1806 wzięty został do niewoli batalion gwardii pruskiej. Nie byłoby w tym nic dziwnego i nadzwyczajnego w tamtych dniach, gdyby nie fakt, że przejeżdżający właśnie obok jeńców dowódca 7. korpusu rozpoznał w starym sierżancie pruskim dawnego znajomego. Dlaczego takie zdarzenie było możliwe? - Dowódcą owego VII korpusu był marszałek Pierre Augereau, który służył w armii pruskiej w latach 80-tych XVIII stulecia.

82. Po tym jak Junot powiedział Napoleonowi, że Józefina nie jest mu wierna, ten związał się z pewną kobietą. Żołnierze nazywali ją "Kleopatrą" i "Matką Boską Wschodu" natomiast Bonaparte powiedział o niej "mała głuptaska". Kim była owa kobieta i jak dostała się do Egiptu? - Margueritte Pauline Bellisle, dostała się do Egiptu w przebraniu razem z żołnierzami Napoleona

83. W jaki sposób pożegnał się z życiem Alexander Berthier? - Wyskoczył z okna w mieszkaniu teścia w Bambergu, popełniając samobójstwo.

84. Kiedy i w jakich okolicznościach dostał się do niewoli jedyny raz w swoim życiu Louis Davout, a także jak z niej zbiegł? - Dostał się do niewoli podczas oblężenia Mannheim (1795). Austriacki dowódca gen. Würmser był przyjacielem stryja Davouta, majora Jakuba-Edne d?Avouta. Przez pamięć o nim pozwolił wrócić Davoutowi do Francji, pod słowem honoru. że tenże nie weźmie już udziału w wojnie.

85. 22 lipca 1812 r. doszło do bitwy pod Salamanką. Siłami francuskimi dowodził marszałek Marmont i popełnił kardynalny błąd, dostrzegłszy kłęby dymu na zachodzie myślał, że Anglicy wycofują się i rozkazał odciąć im odwrót. Następnie nie widząc żadnego zagrożenia zabrał się najspokojniej na świecie do wykonywania pewnej czynności. Jaka to była czynność? - Posiłek na świeżym powietrzu.

86. Jan Henryk Dąbrowski został w 1799 zraniony i życie mu uratowała książka, która zahamowała lot kuli. Jaka to była książka i jakiego autora? - "Wojna trzydziestoletnia" Fryderyka Schillera

87. Podczas przemarszu szwoleżerów z Polski do Francji, a następnie Hiszpanii, w nowym pułku doszło do konfliktu pomiędzy Kozietulskim a jego podkomendnymi. Sprawa była dla jednego z adwersarzy na tyle poważna, że podobno chciał sobie odebrać życie rzucając się do Renu. O co poszło? - Chodziło o nakaz wędrowania niejakiemu Niedermayerowi pieszo, przed trębaczami (a nie na koniu)

88. Co mają ze sobą wspólnego następujący oficerowie (pomijając fakt. że wszyscy są w stopniu kapitana i są baronami Cesarstwa): kpt. Devence, kpt. Faudas, kpt. l'Epinay, kpt. Talhouet? - Byli oficerami ordynansowymi cesarza.

89. Jaka była długość używanego przez francuską piechotę karabinu AN IX wzór 1777? - 1515 mm.

90. Na słynnym balu w Warszawie zainteresowanie Marią Walewską wykazał nie tylko Cesarz, ale i dwóch jego oficerów. Jak nazywali się ci ?konkurenci? Napoleona i jak zostali za ?rywalizację? z Cesarzem ?ukarani?? - Bertrand wysłany został za karę do korpusu Hieronima na Śląsk, Ludwik de Perigord wysłany nad Pasłękę do VI Korpusu Neya.

91. Jak nazywało się najpóźniej powołane napoleońskie ministerstwo i kto nim kierował? - Ministerstwo Manufaktur i Handlu, Jean Baptiste Collin de Sussy

92. Jaka była przyczyna śmierci generała Viala? - Zmarł pod Lipskiem na zawał serca.

93. W kampanii 1805 roku przeciwnikiem Francji były m.in. Austria na czele której stał cesarz Franciszek II. W roku 1809 władcą tego państwa był Franciszek I. Czyżby Austriacy numerowali swoich cesarzy wstecz? - Franciszek II był cesarzem Świętego Cesarstwa Rzymskiego Narodu Niemieckiego. Po jego likwidacji przez Napoleona pozostał mu tylko tytuł cesarza Austrii, więc odtąd występował jako Franciszek I.

94. Gdzie Francuzi gromadzili zreformowanych oficerów z półbrygad polskich po reorganizacji Legionów? - W Chalons-sur-Marne

95. Dekret z 01. marca 1808 przywracał dawne tytuły szlacheckie we Francji, oprócz dwóch. Których? - Wicehrabiego i markiza.

96. Do kogo należało i gdzie było położone (z punktu widzenia mapy politycznej ówczesnej Europy) księstwo Pontecorvo, zanim zajęte zostało przez Francuzów? - W Państwie Papieskim.

97. Ilu Polaków otrzymało tytuł Barona Cesarstwa i kim był najmniej znany z nich Józef Chłusowicz? - 17 Polaków, Chłusowicz to pułkownik w 2 pułku piechoty liniowej w 1812.

98. Generał Dąbrowski nie przepadał za alkoholem, wręcz wstręt w nim wzbudzały trunki oprócz jednego rodzaju. Jakiego? - Był to rum.

99. Jak brzmiał drugi, obok ?Starszego Bartka? przydomek, jakim poufale określali w korespondencji generała Bonaparte? - "Wawrek".

100. Podczas tragicznego odwrotu spod Moskwy w 1812 roku Wielka Armia zostawiała prawie w każdym mieście rannych i tych, którzy nie mieli sił dalej się wycofywać. Bardzo często (można powiedzieć nagminnie) zdarzało się, że Ci ludzie padali później ofiarą Kozaków, chłopów, partyzantów, a nawet regularnych żołnierzy. Jednak w jednym z takich miast za owymi nieszczęśnikami stawili się u Kutuzowa dwaj Francuzi służący w wojsku rosyjskim. Proszę napisać nazwiska tych Francuzów i w jakim to było mieście? - Wilno, Saint-Priest i Rochechouart

101. W połowie 1797 r. gen. Zajączek został obarczony przez Napoleona pewną misją, którą przyjął niechętnie, upatrując w niej okoliczność negatywną dla sprawy polskiej. O co chodziło? - W ramach Legionu Brescii miano utworzyć batalion z jeńców polskich. Zajączek uważał to za rozbijanie wysiłku legionowego.

102. Jaka była podstawa używania przez Józefa Poniatowskiego tytułu książecego i czy narodził się już jako książę? - Ojciec, Andrzej Poniatowski otrzymał wraz z braćmi na Sejmie koronacyjnym w 1764 roku tytuł książęcy (dziedziczny), a wkrótce potem tytuł księcia czeskiego od cesarzowej Marii Teresy, z prawem przeniesienia na pierworodnego syna.

103. Na jaką chorobę cierpiał podczas bitwy raszyńskiej książę Józef Poniatowski? - Fluksję

104. Kto był przewodniczącym Związku Reńskiego, jaką funkcje pełnił poza tym i jakie były organy czy też izby tegoż? - Kardynał Dahlberg, arcybiskup Moguncji, prymas, Wielki Książę Frankfurt. Kolegia: Królów i Książąt.

105. W 1809 jednym z korpusów austriackich dowodził ojciec człowieka, który po wypadkach '48-'49 został nazwany jednym z filarów monarchii habsburskiej. Proszę podać ich nazwisko. - Jellacic

106. 8 września i 10 grudnia 1808 na Woli odbyły się manewry w których wzięły udział: 2 brygady piechoty, w czym jeden pułk polski; 3 pułki francuskie i 1 polski. Proszę o podanie dowódców obu ćwiczących stron. - Morand i Poniatowski

107. Z ilu linii obronnych składały się fortyfikacje Torres Vedras oraz jaka była długość poszczególnych linii? - Trzy linie, po 48, 35 i 6 km.

108. W początkach 1798 roku, przed kampanią egipską Józef Sułkowski zaszczycony został ofertą objęcia stosunkowo wysokiego stanowiska w armii republikańskiej. Jakiego? - Szef sztabu armii włoskiej Masseny.

109. 12 artykuł Konstytucji Haitańskiej z 1805 roku zastrzegał, że żaden biały człowiek, jakiejkolwiek by był narodowości, nie postawi stopy na tym terytorium w charakterze pana lub właściciela i nie będzie mógł w przyszłości nabywać tu żadnej własności. Jednak wprowadzono pewne wyjątki od tej reguły, dokładnie trzy. Kogo wyłączono z tego postanowienia? - Kobiety, Niemców i Polaków.

110. Konstytucja Księstwa Warszawskiego przywracała wszystkie ordery istniejące dawniej w Polsce w ich pierwotnej formie. A więc i Order Virtuti Militari, którego pewien szczegół został jednak wkrótce zmieniony. O jaki szczegół chodzi? - Zastąpienie pogoni dewizą ?Rex et Patria?.

111. "Dnia 15 sierpnia 1769 r. przyszło na świat w Ajacco dziecię, które od rodziców otrzymało nazwisko Bonaparte, niebiosa zaś obdarzyły je imieniem Napoleon." - tak zaczyna sie jedna z tysięcy biografii Napoleona. Ojciec autora tej książki był francuskim generałem (choć nie do końca Francuzem), odegrał ważną choć niezbyt eksponowaną rolę w kampanii 1809. Jak nazywali się panowie? - Dumas.

112. Jakiej długości był bagnet karabinu piechoty model M 1801? - 79 cm.

113. W Kaliskim Korpusie Kadetów istniał pewien specyficzny sposób nagradzania i karania kadetów, mianowicie poprzez umieszczanie ich nazwisk na pewnego rodzaju tablicach. Ile było tych tablic, jakie miały kolory i ozdoby i co oznaczało umieszczenie nazwiska delikwenta na poszczególnej z nich? - 1. na dnie białym ze złotymi liniami, 2. na niebieskim ze srebrnymi liniami, 3. na szarym z czerwonymi, 4. czarnym z żółtymi liniami. Na 1. tablicy napisane będzie: aplikacja bardzo dobra, na drugiej dobra na 3. aplikacja mierna, na 4. aplikacja i sprawowanie złe. Na tych tablicach po skończonych egzaminach i rozdanych nagrodach i naganach wpisane będą na papierze imiona tych, którzy odebrali nagrody i tych. którzy zasłużyli na nagany. Na 1. tablicy przyklejone będzie imię tego, który dostał złotego orzełka, na drugiej tych, którzy dostali srebrnego lub wieniec, na czarnej zasługujących na naganę.

114. Wśród generalicji francuskiej tego okresu służyło trzech krewnych marszałków, noszących to samo nazwisko, co ich utytułowani bliscy. Jacy to byli generałowie? - Bracia Cesar i Victor Berthier, Franacois Etienne Kellermann, Pierre Soult, Jean Paul Augereau

115. W czasie walk pod Stralsundem 31.05.1809 poległ major von Schill. 11 oficerów jego korpusu zostało wziętych do niewoli i 09.07.1809 w jednej z należących do Francji twierdz na rozkaz Napoleona rozstrzelanych. Jak nazywała się twierdza? - Wesel nad Renem

116. Ile w roku 1808 było w Księstwie Warszawskim: młynów, gorzelni, browarów, hut żelaza i ludwisarni? - 6578 młynów, 8178 gorzelni,802 browarów, 107 hut żelaza, 0 ludwisarni.

117. Proszę wymienić wszystkich przewodniczących Instytutu Egipskiego, założonego przez Bonapartego w sierpniu 1798 r. - Gaspard Monge, Napoleon Bonaparte, Claude Louis Berthollet, Rene Nicolas Desgenettes, Nicolas Auguste Nouet, Nicolas Jacques Conte, Jacques Pierre Champy.

118. Ile razy, kiedy i w jakich okolicznościach został ranny Chłopicki? - 5 razy, raz w dzieciństwie w trakcie napadu na sąsiada, nad Trebbią, pod Saragossą, pod Tortozą, pod Rżatem i dwa razy pod Grochowem.

119. Jaka była ulubiona gra księcia Poniatowskiego? - Bilard

120. Gdzie i kiedy odbyła się pierwsza rewia WP, po wypadkach 11.IV.1814? Kto ją przyjmował i kto prowadził? - 24 kwietnia w Saint Denis pod Paryzem, przyjmował ją car Aleksander I i Konstanty Pawłowicz, a prowadził ją gen. Krasiński

121. Piękne miasto w Niemczech. W pewnym budynku Cesarz miał przebywać co najmniej dwukrotnie. W trakcie wojen napoleońskich kilkakrotnie zmieniało przynależność państwową, by ostatecznie być włączonym do Bawarii. Jakie to miasto? - Würzburg.

122. Kiedy i kto ustanowił medal św. Heleny i jaki dokładnie był na nim napis? - Napoleon III 12 sierpnia 1857, "Cesarz Napoleon I" i "swoim towarzyszom chwały ostatnią myśl poświęca, Św. Helena 5 maja 1821"

123. Znana powszechnie jest anegdota w której w czasie przeglądu wojska pułk lekkokonnych gwardii popełnił szereg pomyłek. Napoleon miał rzec: "Ces gens la, ne savent que se battre" - Ci ludzie tylko bić się umieją! Gdzie odbywał sie ów przegląd wojska? - W Schönbrunn w Wiedniu.

124. Po utworzeniu Królestwa Westfalii utworzono tam szkoły wojskowe, szkolące przyszłych oficerów. Powstały szkoła wojskowa oraz szkoła artylerii i wojsk inżynieryjnych. W jakich miastach utworzono te szkoły? - Brunszwik i Kassel

125. Napoleon oraz ks. Vicenzy wracający w grudniu 1812 roku z Rosji zameldowali się w jednym z warszawskich hoteli pod przybranymi nazwiskami. Jakimi? - Napoleon - Calauincourt, ks. Vicenzy - Rayneval.

126. "Istnieją narody bez szczęścia, jak i ludzie przeklęci bez szczęścia" Autor tych słów, Chłopicki uważał się za człowieka przeklętego. Za jaki czyn Józef Chłopicki miał być przeklętym? - Rzeź klasztoru w Saragossie.

127. Jakie były i gdzie się mieściły szkoły wojskowe w Księstwie Warszawskim? - Korpusy Kadetów w Chełmnie i Kaliszu, Elementarna Szkoła Artylerii i Inżynierii,Wyższa Szkoła Aplikacyjna Artylerii i Inżynierii w Warszawie.

128. W wigilię Bożego Narodzenia miał miejsce słynny zamach w drodze do Opery. Jaka opera miała być u była grana tamtego wieczora? - Oratorium Haydna "Stworzenie Świata"

129. W marcu 1811 w Warszawie odbyło się niezwykłe przyjęcie. Oto, co o nim napisała jedna z uczestniczek: "Na miejsce zebrania wybrano pałac Brühlowski; podwórzec wyobrażał obóz wojskowy, w podobnym duchu urządzono apartamenty pałacu. Karabiny artystycznie ustawione w kozły ze świecami nadzianymi na bagnety zapewniły oświetlenie". Z jakiej okazji odbyło się to przyjęcie? - Imieniny księcia Józefa

130. Jeden z dekretów wydanych przez Napoleona w Lyonie, w celu pozyskania sobie liberalnego mieszczaństwa, oznajmiał: "Kolegia elektorskie Cesarstwa zbiorą się na nadzwyczajne Zgromadzenie Pola Majowego". Kiedy zwołane zostało owe Zgromadzenie? - 1 czerwca 1815

131. Co to była tzw. teofilantropia? - Religio-filozofia stworzona przez La Reveilhere-Lepeauxa (1743-1824), zawierała trochę elementów filozoficznych, trochę chrześcijańskich.

132. Co łączy marsz. Lannesa z powstaniem listopadowym? - Brali w nim udział jego synowie - Hieronim Ramorino i Gustaw Oliwer de Montebello.

133. W jaki sposób zginął gen. Michał Sokolnicki? - Został potratowany przez konia w czasie rewii, zmarł w wyniku obrażeń.

134. Jaki kwiat był symbolem wierności Cesarzowi? - Fiołek

135. Konstytucja roku VIII powołała cztery zgromadzenia, dzielące między sobą władzę ustawodawczą. Jak nazywały się te organy? - Trybunat, Ciało Prawodawcze, Senat Zachowawczy i Rada Stanu

136. Członkiem której sekcji Instytutu Egipskiego był Józef Sułkowski? - III, historii naturalnej i ekonomii politycznej

137. Generałowi Beresfordowi przydarzyła sie przykra przygoda: dostał się do niewoli. Jednak nie francuskiej. Czyjej i w jakich okolicznościach? - Hiszpańskiej, w trakcie podróży do Buenos Aires.

138. Gdzie spoczywają Andrzej Niegolewski i Ignacy Stokowski? - Kościół św. Wojciecha w Poznaniu

139. Dwaj wysocy urzędnicy francuscy, minister spraw zagranicznych Talleyrand i minister policji Fouche, zostali zdjęci ze stanowisk miedzy innymi za działalność antyfrancuską i współpracę z rosyjskim wywiadem. Pod jakimi pseudonimami występowali w kartotekach wywiadu rosyjskiego? - Talleyrand: Anna Iwanowna, kuzyn Henri, księgarz, piękny Leander; Fouche: Natasza, prezydent lub Berzjen.

140. W którym z obozów jenieckich czasów wojen napoleońskich polski ułan zjadł swego kolegę jeńca i jaki los go za to spotkał? - Na hiszpańskiej wyspie Cabrera. Został oddany władzom hiszpańskim, osądzony i rozstrzelany na zamku Bellver na Majorce.

141. Gdzie zmarł generał Jean Joseph Ange d'Hautpoul, który jak wiadomo został śmiertelnie ranny podczas słynnej szarży kawalerii pod Preussisch-Eylau 8 lutego 1807? - Vornen

142. W Komisji Sztuk i Nauk zabranej przez generała Bonapartego w 1798 roku do Egiptu znajdował się wydział astronomiczny. Jaki był jego skład? - Beauchamp, Mechain, Nouet, Quesnot

143. 3 grudnia 1830 r. na Placu Bankowym zrewoltowany tłum warszawiaków prawie zlinczował dwóch generałów znanych z epoki napoleońskiej, którzy się jednak zniesławili za Konstantego. Obydwaj uratowani przez Chłopickiego i Szembeka zmuszeni zostali do publicznego złożenia przysięgi wierności narodowi. W tym celu warszawscy akademicy wyciągnęli sztandar z czasów Napoleona pod którym służyli Polacy. Pytanie - co to byli za generałowie, i jaki to był sztandar na który przysięgali? - Wincenty Krasiński i Zygmunt Kurnatowski, sztandar 113 półbrygady francuskiej

144. W kwietniu 1807 roku pewien cudzoziemski regiment w służbie brytyjskiej podniósł otwarty bunt w pewnym forcie zdzierając flagę brytyjską i wywieszając rosyjską. Kto był twórcą tego regimentu, z jakich narodowości był on złożony i jaki los spotkał owych buntowników? - Regiment hrabiego Froberga, który zbuntował się na Malcie. Składał się przede wszystkim z Albańczyków, Greków, Bułgarów i wcielonych do niego jeńców polskich. Po zdławieniu buntu 24 przywódców rozstrzelano a regiment Froberga rozwiązano wcielając ludzi do różnych pułków cudzoziemskich.

145. W jakich okolicznościach trafił do niewoli generał Exelmans i jak zakończyła się jego niewola? - Podczas misji z marszałkiem Moncey'em, Exelmans został wzięty do niewoli 16 czerwca 1808 niedaleko Walencji przez guerillas. Z Walencji przetransportowano go na Majorkę, a następnie do Anglii. Exelmans uciekł z Anglii razem z pułkownikiem Lagrange w 1811 i wrócił do służby

146. W wyprawie egipskiej 1798 r. uczestniczyło 6 oficerów polskich. Proszę wymienić wszystkich - Łazowski, Grabiński, Haumann, Szumlański, Zajączek, Sułkowski

147. Co właściwie pod Waterloo odpowiedział gen. Cambronne na propozycję Anglików dotyczącą poddania się czworoboku gwardii? - Najprawdopodobniej nic, bądź "pardon monsieur"

148. Proszę wymienić ordery i medale istniejące za Cesarstwa poza Legią Honorową. - Order Zjednoczenia, Order Żelaznej Korony, Order Westfalskiej Korony, Order Królewski Unii, Order Królewski Hiszpanii, Order Królewski Obojga Sycylii, Order Dwóch Szpad

149. W maju 1921 w Warszawie odbyły się uroczystości związane ze 100-letnią rocznicą śmierci Napoleona. Komitet obchodów zwrócił się do Republiki Francuskiej z prośbą o wypożyczenie relikwii po cesarzu. Rząd Francji wyraził zgodę i 2 maja 1821 relikwie te przybyły do Warszawy. Jakie to były relikwie? - Kapelusz, który cesarz nosił na Elbie i w którym odbył kampanie 1814 i 1815, Szpada Napoleona (jako pierwszego konsula), którą miał pod Marengo, Maska pośmiertna zdjęta przez doktora Antomarchi, Wielka wstęga, order i gwiazda Legii Honorowej noszone przez cesarza, Orzeł pułku grenadierów starej gwardii, Kula spod Waterloo

150. Po śmierci Kutuzowa w kwietniu 1813 roku naczelnym dowódcą mianowano Wittgensteina formalnie z dniem 1 maja. Proszę podać od kiedy faktycznie pełnił tę funkcję, tzn. od kiedy wydawał rozkazy jako naczelny? - 25 kwietnia

151. Gdzie znajdował się główny depot jeńców brytyjskich we Francji? Trzymano tam głownie oficerów na słowo. - Verdun

152. Co było treścią senatus consultum z 17 lutego 1810 roku? - 17 lutego 1810 wyszło Senatus Consultum ogłaszające przyłączenie państwa rzymskiego do cesarstwa francuskiego, niezależność tronu cesarskiego od wszelkiej władzy na ziemi i znoszące doczesne władztwo papieży

153. W styczniu 1814 r. ogłoszono pospolite ruszenie na zachodnich granicach Francji. Dotyczyło ono 10-u departamentów. Proszę wymienić przynajmniej 6 z nich? - Bas-Rhin, Haut-Rhin, Mont-Tonnere, Vosges, Haute-Saone, Cote-d'Or, Doubs, Jura, Isere, Mont-Blanc.

154. Pewną grupę polskich legionistów Dąbrowskiego nazywano "kogutkami" proszę podać jaka to była grupa i z jakiego powodu tak byli nazwani? - Grenadierzy VII batalionu nosili kaszkiety francuskie, dostarczone im przez Francuzów, stąd byli nazywani "francuskimi kogutkami".

155. Po zerwaniu pokoju w Amiens Anglia podjęła kroki w celu obrony kraju przed desantem francuskim. Dotyczyła tego m.in. ustawa z 27.lipca 1803 roku. Proszę o podanie, o co chodziło w tej ustawie? - Według tej ustawy wszyscy mężczyźni od 17 do 55 roku życia podlegają ćwiczeniom wojskowym. Innymi słowy powołanie pospolitego ruszenia.

156. Kiedy ks. Józef mianował swoim adiutantem polowym Ludwika Kickiego? W jakiej jednostce był wówczas Kicki i w jakim stopniu? - 31.12.1811, a Kicki był wówczas kapitanem 11. pułku ułanów

157. 6 grudnia 1800 roku przedłożono Bonapartemu raport zatytułowany ?Memoriał dla Pierwszego Konsula?. W raporcie tym zawarto wyniki badań dotyczących możliwości połączenia Morza Śródziemnego z Oceanem Indyjskim. Raport ten zawierał dwa projekty. Pierwszy z wykorzystaniem wód Nilu, drugi, którego przebieg odpowiadał w przybliżeniu wybudowanemu kilkadziesiąt lat później Kanałowi Sueskiemu, proponował wykorzystanie leżącego ?po drodze? Morze Czerwonego. Ten drugi projekt zawierał jednak pewien poważny błąd, który znacznie utrudniałby wykonanie kanału. Błąd ten miał w następnych latach moc pewnika i został skorygowany dopiero w 1847 roku. Na czym polegał ten błąd? - W pomiarach różnicy wysokości był błąd około 10 metrów, co przy jego realizacji mogło doprowadzić do zalania znacznych obszarów.

158. Twórca artylerii konnej armii Księstwa Warszawskiego Włodzimierz Potocki wziął w 1810 roku dymisję. Z jakiego powodu? - Celem było ratowanie majątku przed konfiskatą.

159. W dniu 30. czerwca 1807 roku ukazał się dekret cesarski dotyczący osoby księcia Poniatowskiego. Czego on dotyczył? - Przywrócenie wszystkich majątków zabranych przez Prusy

160. Proszę podać w którym roku i w jakim mieście powstała pierwsza cukrownia przerabiająca buraki cukrowe? - Rok 1801, Konary na Śląsku

161. Proszę opisać, jak wyglądała flaga Elby w czasie pobytu tam Napoleona? - Czerwony pas ze złotymi pszczołami przecinający ukośnie niebieskie tło.

162. W czasie rokowań w Tylży, Aleksander przyznał Napoleonowi i kilku dygnitarzom francuskim order św. Andrzeja. Przynajmniej jeden z udekorowanych dygnitarzy francuskich otrzymał ten order niezgodnie z obowiązującymi za Aleksandra I przepisami dla tego orderu. Kto to był i jakich warunków nie spełniał ? - Talleyrand, order otrzymywano za wygranie ważnej bitwy

163. 26 maja 1812 roku ukazał się w Księstwie dekret księcia warszawskiego. Czego on dotyczył? - Dekret przyznający "w przypadkach nagłych" nadzwyczajne kompetencje Radzie Ministrów Księstwa Warszawskiego.

164. W 1809 roku przeprowadzono w Księstwie Warszawskim zamach stanu. Zamachowcy usunęli władze lokalne, dokonali kilku aresztowań, zastępując władze lokalne swoimi zwolennikami i zerwali kontakt z Radą Stanu. W okolicach jakiego miasta mieli owi zamachowcy swoją siedzibę? - W departamencie łomżyńskim, a konkretnie w Tykocinie.

165. Co się stało ze sztandarami zdobytymi przez Francuzów pod Sławkowem? - Sztandary zostały 16.I.1806 r. złożone w katedrze Notre-Dame, gdzie wisiały do 31.III.1814 r. Tego dnia prefekt Paryża Frochot polecił zdjąć te sztandary i je ukryć, a polecenie to wykonano tak dobrze, że do dzisiaj nikt ich nie odnalazł.

166. Podczas spotkania w Wilnie z Bałaszowem, Napoleon wypytywał go bardzo dokładnie o drogi prowadzące do Moskwy. Rosjanin stwierdził, że jest ich wiele, ale główny nacisk położył na jedną z nich, prowadzącą przez pewne miasto. O jakie miasto chodzi? - Połtawa

167. Poznań - Konin - Kościelec - Kłodawa - Krośniewice - Kutno - Łowicz ? Warszawa. Czego dotyczy ta trasa? - Jest to trasa powrotu doczesnych szczątków Księcia Józefa

168. W jakim uniformie (jakiego regimentu) najchętniej pokazywał się w okresie napoleońskim wielki książę Konstanty Pawłowicz? - Ułański pułk cesarzewicza dowodzony przez Mellera-Zakomelskiego a następnie Czalikowa

169. W dobie napoleońskiej w strojach pań, szczególnie tych wysoko notowanych w towarzystwie można dostrzec w ubiorze pewien element bardzo wówczas modny. Był on dosyć luźno z ubiorem związany. Miała go na sobie pani Tallien przyjmująca młodego Napoleona w swoim salonie. Nosiła go często Józefina. Stał się ten element również niezwykle modny w salonach Księstwa Warszawskiego. Co to było? - Czerwony kaszmirowy szal.

170. Kiedy Fryderyk August wrócił do Drezna po wojnie 1809 roku? - 12. sierpnia 1809

171. Fryderyk August niechętnie podróżował. Ale zdarzyło sie, że raz jeden wybrał się z wizytą do Paryża. Kiedy opuścił Drezno, kiedy wjechał do Paryża i gdzie mieszkał w czasie tego pobytu? - Fryderyk August opuścił Drezno w początkach Listopada 1809 r. Już w Moguncji był witany przez Duroca. W Maux witał go Talleyrand, do Paryża dotarł 13 listopada 1809. Zamieszkał w pałacu Bourbon-Elisee. Następnego dnia odwiedził go Napoleon w towarzystwie Hieronima. 15 litopada zjedli razem śniadanie w Tuileries.

172. Na sygnał trąbki "trwoga" flankierzy rozpoczynali ostrzał przeciwnika z karabinków lub pistoletów. Jak nazywał się sygnał trąbki który nakazywał zaprzestać ostrzału? - "Odbój"

173. Wellington bardzo lubił kolekcjonować zegarki. Szczególne upodobanie miał do trzech z nich, które były zdobyczne. Proszę wymienić wszystkich trzech ich wcześniejszych właścicieli? - Tippoo Sahib, Jean Andoche Junot, król Józef Bonaparte

174. Jaki dzień nazwali Polacy dniem "pokuty Napoleona"? - 21 sierpnia 1812

175. Co to było Apsley House? - Apsley House to londyńska siedziba Wellingtona

176. Wszystkie buław marszałkowskie były identyczne oprócz jednej. Czyja buława różniła się od pozostałych, czym się różniła i dlaczego? - Duplikat, który kazał dla siebie wykonał marsz. Davout po tym, jak utracił swoją pierwotnie otrzymaną, pod Krasnem. Buława ta miała zielony aksamit.

177. Co było najwyższą budowlą Warszawy w okresie XW? - kościół Ewangelicki Szymona Bogumiła Zuga (ewangelicko-augsburski kościół Św. Trójcy w Warszawie)

178. Na Ugodzie między Rzeczpospolitą Lombardii i generałem Janem Henrykiem Dąbrowskim z 9 stycznia 1797 roku w sprawie utworzenia Legionów Polskich we Włoszech widnieje cztery podpisy. Czyje? - Bonaparte, Porcelli prezydent, Visconti reprezentant, Clevera sekretarz.

179. W 1798 roku Tadeusz Kościuszko, przebywający w Paryżu otrzymał bardzo specyficzny prezent z Polski. Proszę podać co był to za podarek i od kogo? - Murzyn od W. Krasińskiego

180. Co zdarzyło się w Salinas i jaki miało związek z Polakami? - 9 kwietnia 1812 7 pułk piech. z dywizji K.W. oraz towarzyszący mu oddział 400 Francuzów i około 300 eskortowanych jeńców został zaatakowany przez partyzantów Miny w jednym z wąwozów koło Salinas. Pułk 7 poniósł bardzo dotkliwe straty około 100 ludzi. Do jaśniejszych epizodów tego wydarzenia należy postawa Warzyńca Kozikiewicza, kaprala 7 pp., który zdobył jeden ze sztandarów Miny i za to otrzymał później (1.VII.1812) krzyż LH. Wydarzenia to miało miejsce w czasie wymarszu dywizji z Hiszpanii i było ostatnim epizodem walk dywizji K.W. w Hiszpanii. Wcześniej pułki 4 i 9 przeszły bez problemów.

181. Wśród wielu pozycji polecanych młodym wychowankom Korpusów Kadeckich Księstwa Warszawskiego znalazły się pozycje:
1. Kodeks Rycerski
2. Katechizm Kadecki
3. Nauki Obyczajowe dla ludzi
4. Prawidła początkowe nauki obyczajów do pojęcia uczącej się młódzi przystosowane
5. O cnotach towarzyskich i występkach im przeciwnych
Jacy byli ich autorzy? - Adam Czartoryski, Stanisław Potocki, Grzegorz Piramowicz, Kajetan Skrzetuski, Ksawery Dmochowski

182. Przekroczenie tej rzeki w kierunku południowym było symbolicznym " wkroczeniem do piekła wojny " Co to za rzeka ? - Bidassoa

183. Jak się nazywała szabla kozacka? - Szaszka.

184. Jaka przyczyna uniemożliwiła Muratowi wzięcie udziału w bitwie pod Pułtuskiem? - Choroba weneryczna (syfilis)

185. W 1867 r. powstało Archiwum Główne Akt Dawnych, jednak już w czasach Księstwa powołano instytucję, która miała zajmować się uratowaniem spuścizny dokumentowej po czasach dawnej Rzeczypospolitej. Proszę podać jej nazwę. - Instytucja to powołane dekretem księcia warszawskiego Fryderyka Augusta z 2 IX 1808 r. Archiwum Ogólne Krajowe.

186. W czasie pierwszego pobytu Cesarza w Warszawie, w dniach 21 i 22 grudnia 1806 roku odbyły się na dziedzińcu Saskim, w obecności Cesarza, dwie, kilkugodzinne rewie, w których wzięły udział kolejno dwie dywizje. Proszę podać dowódców tych dywizji. - Lannes i Suchet

187. Jak nazywał się order ustanowiony przez Józefa w Hiszpanii? - Orden Real de Espana

188. 20. czerwca 1810 roku powołano w Księstwie komisję w składzie: Matuszewicz, Linowski, Staszic, Woyda, Kochanowski, Sobolewski.Czym miała się zająć owa komisja? - Była to komisja mająca zająć się reformą administracyjną.

189. Kiedy miała miejsce pierwsza emisja biletów kasowych (banknotów) wydanych przez rząd Księstwa Warszawskiego? - 1 grudnia 1810 r. Wydrukowano je w Dreźnie.

190. Jaki jest źródłosłów nazwy: Wandea? - Nazwa departamentu została wzięta od nazwy płynącej przez niego rzeki Vendee. Ta nazwa z kolei wzięła się od celtyckiego słowa Vendo.

191. Proszę uporządkować 26 napoleońskich marszałków w kolejności od najdłużej do najkrócej żyjącego. - Jean Moncey 88,François de Kellerman 85,,Nicolas Jean - de - Dieu Soult 82, Jean Baptiste Bernadotte 81,Emanuel de Grouchy 81,Nicolas Oudinot 80, August Frédéric de Marmont 78, Perrin Victor 77,Jean Filibert Sérurier 77,Alexandre Macdonald 75,Jean Baptiste Jourdan 71, Adolphe Mortier 67, Hyeres Gouvion St. Cyr 66. François Joseph Lefébvre 65,Alexandre Berthier 62,André Masséna 59,Pierre François Augereau 59, ,Luis Gabriel Suchet 56,Louis Nicolas Davout 53,Wilhelm Brune 52,Józef Poniatowski 50,Joachim Murat 48,Michał Ney 46,Jean Baptiste Bessiéres 45 i Jean Lannes 40

192. Kiedy w 1809 r. do Warszawy wrócił rezydent Serra a kiedy Rada Stanu? - Serra przybył do Warszawy wieczorem 4 czerwca zaraz po wkroczeniu Kosińskiego, Natomiast Rada Stanu przyjechała z Tykocina 8 czerwca

193. Jakim terminem nazywano tajne zbrojenia Prus po pokoju w Tylży? - Krumpersystemm

194. Pierwszy traktat paryski podpisały z Francją sprzymierzone mocarstwa (Austria, Anglia, Rosja, Prusy) 30 maja 1814 r. W ciągu następnych dwóch miesięcy dołączyły do nich jednak 3 kolejne państwa. Proszę je wymienić. - Szwecja - 8 czerwca, Portugalia - 12 czerwca i Hiszpania - 20 lipca

195. Jaki rodzaj działania lekkiej jazdy określano potocznie "bieg zająca"? - Lekka jazda miała wciągnąć ciężką jazdę przeciwnika najlepiej na grząski teren a następnie poprzez częste i szybkie obroty zmęczyć konie przeciwnika oraz spowodować rozluźnienie jego szyków lub wręcz rozproszenie. Na sygnał trębacza lekkokonni powinni się przegrupować i uderzyć w najsłabsze miejsce wroga.

196. Kiedy 28 stycznia 1814 roku Napoleon wyjeżdżał z Paryża aby osobiście dowodzić armią regentką ustanowił Cesarzową, lecz u jej boku utworzył Radę Regencyjną złożoną z:?? - Józefa i Cambaceresa

197. W 1796 r. policja pruska aresztowała w Warszawie członków i współpracowników niepodległościowej organizacji. Proszę o jej nazwę i choć 2 aresztowanych.?-- W kwietniu 1796 Prusacy rozbili tzw. Zgromadzenie Centralne. Aresztowanymi członkami byli: Grabowski, Gielgud, Rose, Szulecki, Błeszyński, Dunqerque, Grozmani. A sprawa zaczęła się od aresztowania berlińskich emisariuszy Musoniusa i Forestiera. Szaniawski dzięki podróży do Paryża uniknął aresztu.

_________________
"W polityce głupota nie stanowi przeszkody."
Napoleon


Ostatnio edytowano 3 lut 2010, o 21:46 przez Louis Nicolas, łącznie edytowano 1 raz

Góra
 Zobacz profil  
 
PostNapisane: 3 lut 2010, o 21:45 
Offline
Général de brigade
Général de brigade
Avatar użytkownika

Dołączył(a): 3 lut 2010, o 06:45
Posty: 1248
Post użytkownika Oudinot

PYTANIA - CYTATY.

21. "Jego Cesarska Mość nigdy nie uczyni ze mnie duc d'Albe (księcia stryczka). Złamię juz prędzej swoją buławę, niż usłucham rozkazów, których sam pan będziesz żałował. Wojna jest już i tak dostatecznie straszna aby nie dodawać jeszcze do niej zbędnych okrucieństw." Jaki marszałek pisał tak do Napoleona i w jakich okolicznościach? - List taki wystosował Davout w Hamburgu, na sugestię Cesarza, aby był surowy dla mieszkańców po odzyskaniu miasta przez Francuzów.

22. Kto do kogo i w jakich okolicznościach miał powiedzieć: "Czyżby jazda przysięgła sobie zrobić dziś wszystko?" - Napoleon do Murata po bitwie pod Abukirem (25 lipca 1799)

23. "Napoleon ujarzmiał los mądrem, przezornem i śmiałem kombinowaniem sił materjalnych, ale szczególniej jako władca duszy żołnierskiej, umiejący z niej wydobyć najwyższe natężenie ofiarności i poświęcenia, stoi on w historiji samotny w swej nieporównanej wielkości". Kto jest autorem tych słów? - Marian Kukiel

24. Kto, do kogo i jakich okolicznościach powiedział te słowa: "Dzieci moje! Jeśli dzień dzisiejszy okaże się ostatnim dniem Francji, trzeba abyśmy wszyscy umarli"? - Generał Mikołaj Maison, dowódca 56 dywizji piechoty. Te słowa wypowiedział w pierwszym dniu bitwy pod Lipskiem na odcinku Wachau, aby zagrzać do boju podległe mu oddziały.

25. Kto do kogo i o kim powiedział te słowa: "Doprawdy, ten nasz sojusznik ukrywa swą grę aż za dobrze!"? - Car Aleksander do księcia Cariatiego o Joachimie Muracie.

26. "W ostatnim liście mówiłem Ci, że chcę złożyć wizytę księciu Gotha. Wysłałem więc Haumana do marszałka dworu, aby zapyta? kiedy mogę dostąpić zaszczytu zobaczenia Jego Wysokosci. Odpowiedziano mi, że książę czeka na mnie i mego majora z obiadem, zaproszono też Haumana.Przysłano o drugiej godzinie zaprzęg dworski po mnie, a ponieważ wiedziano, że jestem księciem Rzeszy, zwracano się do mnie jak do Wysokości, a straż księcia oddała mi honory wojskowe należne mej wysokiej randze." Kto jest autorem tego listu? - Antoni P. Sułkowski.

27. "W nadziei na wyraźną i tak oczekiwaną odpowiedź, rozkazałem wyruszyć moim wojskom i działałem zgodnie z zaproponowanym planem. Ale Wasza Cesarska Mość przez całe dwa miesiące zrobiła wszystko, by żadne słowo z Jej listów nie mogło mnie uspokoić, ani mną pokierować". Kto komu czynił ten wyrzut i w jakich okolicznościach? - Był to list z 14 stycznia 1814 roku w którym to Murat usprawiedliwiał się przed Napoleonem dlaczego podpisał traktat z Austrią, który potwierdzał fakt odejścia Murata.

28. "Bitwa zaczęła się fatalnie; pod gwałtownym ogniem wroga lansjerzy młodej gwardii ogarnięci paniką poszli w rozsypkę, wpadli na dywizję starej gwardyi konnej, zmieszali jej szyki. Tłum jeźdźców francuskich w dzikim nieładzie, zupełnie pomieszany z konnicą nieprzyjacielską, wpadł na orszak cesarski. Napoleon dobył pałasza i osłonięty przez pluton służbowy szwoleżerów polskich pod komendą Skarżyńskiego, przebił się do żelaznego czworoboku piechoty Nadwiślańskiej. Był to batalion...." Kto dowodził tym batalionem i jaka to było to bitwa? - Arcis-sur-Aube, Jan Skrzynecki bądź Paweł Muchowski

29. O kim Napoleon miał powiedzieć na Świętej Helenie "Gdybym miał dwóch takich generałów jak on w Hiszpanii, nie tylko zdobyłbym Półwysep, ale także utrzymałbym go"? - Marszałek Suchet

30. "Sire! Nigdy jeszcze nie byłeś tak wielki!" Kto i w jakich okolicznościach wzniósł ten okrzyk? - Ney i Macdonald po podjęciu decyzji o abdykacji w 1814 roku.

31. O jakim Wodzu Cesarz miał powiedzieć "miał serce w głowie"? - Henri La Tour d'Auvergne de Turenne

32. "Nayiaśniejszy Cesarz Napoleon, Zbawca Narodu Polskiego, przybywszy tu dziś o godzinie 3.20 po południu, stanął w domu JPana Kosierkiewicza z xięciem Neufchatel (marszałkiem Berthier)". O przybyciu do jakiego miasta informowała gazeta? - Łowicz

33. O jakim wydarzeniu i kiedy pisał książę Adam Czartoryski w liście do syna Adama w tych słowach: "Odkryło się, że Ponpernikle chcieli podpalić Pragę, rozumiem, że les auteurs du projet ont mal passe leur temps, ale o tym się nie mówi"? - Chodzi o Westfalczyków z VIII Korpusu Hieronima, chcących podpalić Pragę w czerwcu 1812 roku.

34. "Wszystkie towary pochodzące z angielskich fabryk i zakazane, które obecnie znajdują się we Francji bądź to w składach rzeczywistych, bądź to w naszych magazynach celnych z jakiegokolwiek tytułu, zostana publicznie spalone [...]" Jakiego dokumentu jest to fragment? - Dekret z Fontaineblau z 19.10.1810

35. "Jeśli człowiek z towarzystwa robi szaleństwa, ma kochanki, brzydko postępuje z żoną, nawet wyrządza ciężkie krzywdy swym przyjaciołom, to spotyka się oczywiście z potępieniem, ale byle był bogaty, potężny i zręczny, będzie jeszcze mógł liczyć na ludzką wyrozumiałość. Jeżeli jednak ktoś taki szachruje przy grze, przepędzają go natychmiast z dobrego towarzystwa, które nie wybaczy mu nigdy". Jakiego wydarzenia dotyczy ten komentarz? - Wydarzeń z czerwca 1808, wymuszonego zrzeczenia się pretensji do tronu Karola IV i Ferdynanda VII w Bajonnie.

36. "Zawiadamiam marszałka XX, że korpus ten przechodzi pod jego rozkazy. Moim zamiarem jest, gdy Polacy będą w odpowiedniej sile, abyś posłał ich do Marienwerder i niech blokują Grudziądz. Wydasz także rozkazy, ażeby ludzi gotowych z drugich batalionów wcielić do pierwszych, aby można było rozpocząć działania z jakąś znaczną siłą". Do kogo adresowany był ten list i czyje nazwisko zostało zastąpione literami XX? - XX to Bernadotte, list adresowany do Berthiera

37. "Doprawdy nie wiem, co począć z tymi ludźmi. Jeśli postawić ich za fortecznymi murami, to może ostatecznie będą walczyć, ale nie sposób prowadzić tę hałastrę do bitwy w jakim takim szyku. Obawiam się, że będą raczej ciężarem dla nas, niż pomocą". Kto, o kim i w jakich okolicznościach wypowiedział tę sentencję? - Wellington, w lipcu 1809 roku przed bitwą z marszałkiem Victorem pod Talaverą, po przeglądzie armii hiszpańskiej

38. "Dałbym trzysta milionów w złocie, które mam w piwnicach Tuilerii, by go ocalić". O kim i w jakich okolicznościach cesarz wypowiedział tę kwestię? - W trakcie bitwy pod Krasnem o marszałku Neyu, który chwilowo zaginął.

39. "Wspomnienie przy tym wypełnia już duszę dreszczem, jednak także gorącą modlitwą dziękczynną do Najwyższego, którego niewidzialna Wszechmocna ręka, Armię Waszego Cesarskiego Majestatu, która dla jego świętej sprawy walczy, widocznie chroniła". Są to słowa Suworowa napisane do cara Pawła. Jakie zdarzenie wypełniało dreszczem duszę feldmarszałka i za co dziękował on Bogu? - Chodzi o bitwę w Szwajcarii na "Diabelskim moście" 25 września 1799 roku.

40. "Daj Boże, abym mógł ojczyźnie mojej być użytecznym i to jest najżywsze moje życzenie. Ale nie widzę racji, aby stać się igraszką błahych i próżnych projektów cudzych ambicji". Kto w rozkwicie swej młodości napisał te piękne, mądre i co najciekawsze prorocze słowa? - Józef Poniatowski.

41. W jakich okolicznościach i kiedy Napoleon wypowiedział słowa: "o piątej rano będzie słońce, jak pod Austerlitz"? - 27 lipca 1812 pod Witebskiem.

42. "Młody ten człowiek łączył z wielu wiadomościami naukę i odwagę osobistą, poświęcenie stanowcze dla sprawy wolności i wszystkie uczucia dobrego Polaka"Tak pisał arystokrata o jakobinie. Kim byli? - M.K. Ogiński o Józefie Sułkowskim.

43. Kto, o kim i do kogo powiedział w 1796 roku następujące słowa: "To jest nasz wróg! A doprawdy nie wiem dlaczego"? - Dąbrowski do adiutanta Tremo o Józefie Sułkowskim.

44. "Leżały przerażające stosy ciał, jedne na drugich, blisko półtora tysiąca żołnierzy tu padło, a wielu jeszcze żyło, tylko przywalonych trupami. Niektóre ciała były sczerniałe, spalone wybuchami prochów; inni zginęli poszarpani na kawałki kartaczami, jeszcze inni padli od kul muszkietowych, a wszystkie ciała stygły teraz w kupie" O jakim wydarzeniu opowiada ten cytat? - O zdobyciu Badajoz.

45. "Zostawiony między trupami, bez życia prawie, w bitwie pod Kulm, postanowił resztę dni swoich Panu Bogu poświęcić w kapłańskiej służbie. Gdy ojciec chciał zwłoką całoroczną powołanie jego wypróbować, utrzymywał się w tej myśli świętej ciągłym umartwieniem, sypiał na gołej ziemi, wyrzekł się ulubionego napoju-kawy". Tak pisała o swoim bracie, oficerze pułku szwoleżerów gwardii, Paula Morawska. Jak nazywał się oficer? - Tadeusz Łubieński

46. "Odważni zginą tutaj, tchórze zdechną na Sybirze". Kto do kogo i w jakiej sytuacji wypowiedział te słowa? - Marsz. Davout do żołnierzy swojego korpusu pod Pruską Iławą 8 II 1807 r. w czasie przeciwuderzenia Lestoqa.

47. "Że Napoleon naprzód żądał ofiar bez żadnej pewnej obietnicy, a potem utworzywszy Księstwo Warszawskie ciągle żądał nowych, to jest prawda; lecz Polacy sami bez szemrania chętnie wypełniali jego życzenia ciesząc się, że mogą być użytecznymi, to także jest prawda. Największym dowodem służy to, że zostali mu wierni aż do końca. Tak ten oszust potrafił oczarować Polaków, że dziś imię jego w pałacu bogacza i w ubogiej chatce wieśniaka jest wspominane z błogosławieństwem." Kto to napisał? - Walerian Łukasiński

48. "Gdy pewnego dnia odbywałem sjestę po obiedzie, zostałem obudzony przybyciem kuriera. Dostarczył mi pismo Dyrektoriatu z dnia 13. lutego 1799 powołujące mnie na stanowisko głównodowodzącego Armii X, na miejsce odwołanego generała Z. Został on odwołany do Paryża, żeby wytłumaczył swój bojaźliwy sposób prowadzenia kampanii, rabunków i podpisanie w Kapui zawieszenia broni". Kto był autorem tych wspomnień? - Etienne Macdonald

49. Kto, do kogo i w jakiej sytuacji wypowiedział te słowa: "Idź! I niech się spełni Twoje przeznaczenie"? - Letycja do Napoleona w chwili odjazdu z Elby

50. "Pułkowniku, gdybym miał się w ręce nieprzyjaciół dostać, proszę was i nakazuję, abym z waszych rąk raczej zginął". Kto do kogo i w jakich okolicznościach wypowiedział te słowa? - Napoleon do Stokowskiego nad Berezyną.

51. "Frak granatowy z kołnierzem i łapkami u rękawów koloru fioletowego. Frak, kołnierz i łapki sukienne, spodnie tegoż koloru jak frak także sukienne; kamizelka takaż. Guziki obszywane w sukno tegoż koloru albo kamelowe. Krój munduru i forma taka, jak dla członków zawiadujących szpitalami. To samo tyczy się ilości guzików i zapinania. Chustka na szyję czarna, buty zwyczajne, kapelusz stosowany z kokardą narodową i z szlifką do kokardy czarną, jedwabną. Surdut i płaszcz podobnie jak frak z tegoż sukna i takiemiż łapkami i kołnierzem. Szpada francuska żółta z paskiem skórzanym czarnym." Czyj to mundur? - Kapelana wojskowego.

52. "Przez dwie godziny wojska obu stron tkwiły we mgle, później rozproszonej przez piękne, jesienne słońce. Obie armie dostrzegły, iż są od siebie oddalone zaledwie o strzał armatni. Lewym skrzydłem Francuzów dowodził marszałek Augereau. Od centrum, zajmowanego przez korpus marszałka Lannesa, oddzielała je gwardia cesarska. Prawe skrzydło stanowił korpus marszałka Soulta. Armia nieprzyjaciela była liczna, z piękną kawalerią, która wykonywała manewry szybko i precyzyjnie. Cesarz wolałby opóźnić bezpośrednie starcie o dwie godziny, aby ? na pozycji, jaką właśnie zajął po porannym ataku ? doczekać przybycia dodatkowych oddziałów, a zwłaszcza kawalerii.(...) Po niecałej godzinie wszyscy byli w akcji: 250 czy 300 tysięcy ludzi". O której bitwie mowa? - Bitwa pod Jeną

53. Kto powiedział: "Napoleon szukał cnoty, a nie znalazłszy jej, zdobył moc"? - Wolfgang Goethe

54. Z czego to cytat i kogo dotyczy: "(...) posiadał genialny zmysł tworzenia, olbrzymią energię, kolosalną ambicję i stalowy charakter. (...) twardy niczym skała, dziwaczny w odruchach, skrajny egoista, kłótnik i zawadiaka, z którym współżycie stawało sie katorgą, nie przyjaźniący sie z nikim - przez nikogo nie lubiany, lecz szanowali go wszyscy" - Las Cases o Suffrenie.

55. "Były to decydujące dni. Po raz pierwszy od roku 1789 rząd użył przemocy, by zgnieść powstanie ludu, i w ten sposób złamał samą zasadę rewolucji. Po raz pierwszy wojsko odpowiedziało na jego wezwanie i zerwało milczącą umowę, która od 14 lipca łączyła je z walczącym ludem: rozłam będzie się odtąd ciągle pogłębiał, lud nie powstanie więcej aż do roku 1830, a wojsko powoli zagarnie władzę w Republice, by ją oddać swoim generałom" - jakiego wydarzenia dotyczy ten cytat? - 1-2 prairiala roku III (20-21 maja 1795) lud wtargnął do Konwentu z zadaniem "Chleba i Konstytucji 1793 r."

56. "Więc po to krwią polską zafarbowała się Europa i Ameryka? Cóż na to powiedzą bracia nasi zamknięci w twierdzach Gdańska, Modlina i Zamościa, gdy dowiedzą się rychło, że koledzy ich, i to właśnie ci, których cała armia francuska uznaje za okrytych chwałą dwóch kampanii, doświadczyli teraz krzywdy podobnie ciężkiej? A cóż się stanie z tamtymi pułkami? Do jakiego należą one państwa? (..) Czymże więc są? Buntownikami, jeśli obowiązek swój spełnią, nędznikami, jeżeli porzucą posterunki". Kto był autorem tego listu? - Józef Wielhorski

57. "Generale, jesteś wielki jak świat, lecz świat jest dla Ciebie za mały!" Kto i w jakich okolicznościach wykrzyknął te słowa? - Kleber po zwycięstwie pod Abukirem

58. "Pierwszy to jest raz w czasie mej 36-letniej służby wojskowej, że się waham uskutecznić rozkazy mej zwierzchności. Pierwszy raz w czasie ciągłej 16-letniej służby mej Ojczyźnie, że proszę o uwolnienie od niej, skoro pod takimi prawami hańbiącymi wojsko mam komenderować, albo proszę o pozwolenie nie publikować, a razem nie egzekwować tych rozkazów w mej komendzie". Kto i z jakiego powodu tak napisał? - Dąbrowski do Poniatowskiego po wprowadzeniu tzw. "procederu wojskowego" z 1775 roku. Efekt osiągnął, w jego dywizji kar cielesnych nie stosowano

59. "... pod bronią z pewnością wierzyła zostawać nieprzestannie wspierana przez Anglików, lecz po wejściu marszałka zupełnie opuszczona być postrzegała się. Duch wojny zostawał jednak dla odparcia kary, jakiej się lękano. Postępowanie łagodne i ofiarowana amnestia mogły powrócić spokojność, ale surowość mieszkańców broniła przystępu, tak wiec mieszkaniec, ścigany przez nich, karany przez nas szedł w rozpaczy i chwytał się wściekłej zemsty". Jaki region Europy znajduje sie w miejscu kropek? - Kalabria

60. "Po wyjeździe moim z Warszawy stanąłem w 50 godzin tutaj, gdzie już zastałem kilka tysięcy rekrutów przeznaczonych dla nas. Natychmiast nas do roboty zapędzono i my we dwóch z Chłopickim pułkownikiem uformowaliśmy pierwszy batalion mocny do tysiąca ludzi zebranych, ale bez oficerów i unteroficerów, co długo egzystować nie może i dlatego pisano już do księcia Józefa do Warszawy o przysłanie, jeżeli można, oficerów, bo nasi włoscy, kto wie, kiedy się przywleką. Ten batalion za dni kilka ma iść pod Glatz dla blokowania tej fortecy, w której znajduje się do 10 000 Prusaków, ale trzeba się spodziewać, że po odebraniu Królewca i po ostatnim zwycięstwie (o którym się w tym momencie dowiadujemy) i ci się poddadzą, bo cóż by sami znaczyli na tym padole płaczu." Do jakiego miasta przybył piszący te słowa oficer? - Wrocław

61. "Wyszła z rozkazu naprzeciw niego i jego kolegów jedna kompania gwardyi narodowej, wyszło kilka cechów, nakłonionych do tego wódką, i trochę Żydów ze swoim zawsze gotowym baldachimem. Do okrzyków zaś zabrakło zupełnie zapału". Kto i gdzie był tak "radośnie" witany? - Powitanie Stanisława Kostki Potockiego i członków rządu

62. "Po kilkudniowym tam odpoczynku, ruszyliśmy w pochód na Szkłów ku Smoleńskowi. Droga wielka, prowadzona nad brzegiem Dniepru, była bardzo szeroka, tak że sekcjami piechota maszerować mogła (...) wybornie się szło. Generałowie naprzód jechali, żołnierze śpiewali. Wyruszyliśmy przed wschodem słońca z biwaków. W Szkłowie, miasteczku całkiem żydowskim, nad Dnieprem była dniówka. Tam dowiedzieliśmy się, że armia nieprzyjacielska Smoleńska bronić będzie." W jakiej miejscowości odpoczywała jednostka żołnierza, który napisał te słowa? - Mohylew

63. "Pomnąc na to, iż wywóz zboża wtenczas dopiero mieć miejsce powinien, gdy już wewnętrzne krajowi potrzeby są zaspokojone, widzimy oczywiście, iż nasz pochwalony handel zewnętrzny zbożowy więcej przeciw nam, przeciw naszej fałszywej rachubie, niż za kwitnącym stanem rolnictwa mówi". To słowa jednego z myślicieli i ekonomistów doby Księstwa i następnej. O kogo chodzi? - Dominik Krysiński

64. O jakim uczestniku kampanii 1812 roku napisano: "gotów pożerać Francuzów na śniadanie, obiad i kolacje"? - Wilson

65. "Szczęk broni budząc troskliwość każdego obywatela, tym bardziej dla mnie obojętnym być nie może, który od pierwszej młodości dzieliłem wszystkie niebezpieczeństwa kraju i sławę jego obrońców. Za łaskawym J.O.W.Ks. Mci wstawieniem się świeżo zaszczycony względami Najaśniejszego Pana, tym mocniej czuję obowiązek wypłacenia się ojczyźnie i Królowi, poświęcając usłudze publicznej moją osobę". O jakich względach wspominał autor tego listu? - Kosiński, który w 1811 został gen. dywizji oraz dowódcą korpusu weteranów

66. Komu przypisywane są słowa: "oficer kawalerii powinien być zawsze panem swego konia, inaczej koń oficerem, a oficer staje się koniem"? - Różniecki

67. "- Książę, ten dzień będzie dorzucony do wieńca Pana chwały!
- Mojej chwały? Powiem szczerze, nie mam o niej zbyt wysokiego mniemania... Ostatecznie, z jakimkolwiek wynikiem, to będzie moja ostatnia bitwa..." Kto był tak przekonany o swojej śmierci? - Lannes

68. "Jeśli książę ze swym nazwiskiem, fortuną i okolicznościami, które go faworyzują, miałby w piersi ogień Potemkina lub Orłowa, byłby cesarzem Rosji. lecz jego nic nie może ożywić. Kiedy mówi się do niego, jest zimny jak bryła lodu i nie potrafi ludzi rozpłomienić, ani podobać się im." Kogo dotyczą te słowa, kto to powiedział i jakie były okoliczności które faworyzowały bohatera tych słów? - Ambasador sardyński w Rosji Joseph de Maistre. Wypowiedź z III 1807 gdy Czartoryski odzyskał chwilowo znaczną część swoich wpływów, a ambasador oceniał jego styl kierowania resortem spraw zagranicznych i oceniał cechy charakteru. Chodziło o plany pokojowe i układ przyszłej Europy po zakończeniu wojny 1807.

69. "Pan Bonaparte wcale nie chce wyjechać, jednak będzie musiał pozbawić nas swego towarzystwa, gdyż jego obecność nam przeszkadza, naprzykrza nam się; szkodzi powodzeniu naszych negocjacji. jeśli ma nadzieję, że go jeszcze raz wezwiemy, myli się, nie chcemy go. Niech pan mu powie ode mnie, że musi wyjechać, i jeśli nie wyjedzie natychmiast, to każę go aresztować, że aresztuję go osobiście". Kto jest autorem tych słów? - Davout

70. "Sukno brane na mundury było jak pajęczyna, spodnie zaledwie do łydek dochodziły, a wszystko było tak ciasne, że z trudnością przychodziło się żołnierzowi zapiąć; kamasze czarne zachodziły poza kolana i przykrywały spodnie. Żołnierz nosił trzewiki, robione ze starego rzemienia, miał mundur granatowy, wycięty od przodka, do niego były przyszyte dwa kawałki sukna białego, mające wyobrażać kamizelkę. Binda czarna stanowiła chustkę na szyję, a musiała być koniecznie ciasno spiętą ażeby krew biedakowi nabiegła do głowy i nadawała mu cerę czerstwą i zdrową, lubo był rzeczywiście blady i chorowity dla nędznej strawy którą spożywał; każdy żołnierz musiał być upudrowany i nosił harcap, zwykle z konopi uwity, a zawieszony za kołnierz; na głowie miał kapelusik trójrogaty i loki, także sztucznie do kapelusza przymocowane, co go śmiesznej postaci nabawiało... Żołnierz nie dostawał w owej epoce koszuli ze skarbu, ale miał przyszyte w rękawach po kawałku płótna, pokazujące się u rąk, które wyobrażały mankiety. Wojsko nie miało płaszczy, pewna ich liczba, przeznaczona li dla szyldwachów, wisiała w czylerhauzie, a żołnierze przyodziewali się nimi, gdy stali na warcie." Jakiej armii i z jakiego okresu dotyczy ten opis? - Armia pruska z 1796

71. 23 kwietnia 1815 roku ks. Adam Czartoryski tak pisał do gen. Krukowieckiego: "Prosiłem cię i błagałem Kochany generale, abyś za przyjazdem twoim do Warszawy raczył się obejść jak rozważny rozsądek, surowa honor i potrzeba ustalenia powagi własnego charakteru wymagały. Uniesiony momentalną i nieszczęśliwą namiętnością zapomniałeś zupełnie rad ze szczerego serca ci danych. Uchybiłeś prawom, przystojności i wodzom, uchybiłeś sobie samemu...Wybić się można, jak rycerzom przystoi, lecz po cóż walać sobie usta i ręce plamiącymi słowy i postępkiem... Boli mnie mocno, że personalność do tak wysokiego stopnia góruje w uczuciach twoich nad miłością narodową. Tej ostatniej słuchając nigdy byś obcym nie był tak owego widowiska...Nie takie listy spodziewałem się do generała pisywać". Co było przyczyną napisania przez Czartoryskiego tego listu? - Sprawa konfliktu Krukowieckiego z Sokolnickim na tle postępowania Krukowieckiego pod Lipskiem, reakcji na to Sokolnickiego, wydanie przez niego broszury z zarzutami wobec Krukowieckiego, powołanie Komisji Śledczej i wreszcie obelgi Krukowieckiego wobec Sokolnickiego publicznie wobec generalicji i Konstantego w czasie festynu w Łazienkach 11 kwietnia 1815 na cześć wiadomości o wskrzeszeniu Królestwa
72. "Tu mszcząc się na mieszkańca prawa ludzkich pogardy,
Szedł w sławie na wyścigi Sarmata z Lombardy...
I rycerz, co odwagą w swej ojczyźnie słynął,
Walcząc mężnie za swoją, za cudzą tu zginął!"
Autorem tego wiersza był Cyprian Godebski. Kogo uwiecznił on w tych rymach? - Eliasza Tremo

73. "Siły obu stron są równe; oni mieli nadzieję, w nas zbierał gniew; nasze karabiny zatrzymywały ciosy ich szabel; ich konie natarły na nasze szeregi. Nigdy nie było okrutniejszej bitwy, bardziej błyskotliwego zwycięstwa, mniej przewidywanego rezultatu; sen, z którego pozostało tylko przerażające wspomnienie: zrobiłem dwa rysunki. Chciałem namalować wojnę taką, jaką jest, szlachetna i zacięta, okrutna i wyrafinowana". Którą bitwę widział autor tych wspomnień? - Bitwa pod Sedyman

74. Kto i na kogo skarżył się następującymi słowami: "Nie masz rodzaju grubiaństwa, których bym /.../ nie słyszał i za każdym uspokojeniem się, kiedy chciałem mu wytłumaczyć całą rzecz, znowu musiałem słuchać: foutre, bougre, merde i tym podobne komplimenta"?- Tymi słowami Kosiński skarżył sie na Lefevbre'a w liście do Dąbrowskiego.

75. Jaki incydent w czasie kampanii 1807 Dąbrowski skomentował następującymi słowami: "Napiszę natychmiast do stanów /.../, że to wszystko co się stało od komendy mojej w /.../ przeciw prawom wojny, było przeciwko mej woli i nigdy z mojego rozkazu"? - Dotyczyło bezprawnych kontrybucji dokonanych m.in. na rozkaz Sokolnickiego w zdobytym Słupsku, gdzie zażądano od mieszkańców miasta zegarków złotych i srebrnych, które to potem zostały na polecenie Dąbrowskiego zwrócone.

76. "Trzeba postawić dziesięć tysięcy kozaków polskich: lanca i koń wystarczą" Do kogo, gdzie i kiedy Napoleon wypowiedział te słowa? - Napoleon wypowiedział te słowa 10 XII 1812 w Hotelu Angielskim w Warszawie do Pradta.

77. "- Rozkazano mi wzbronić panu wstępu do miasta.
- A ja mam rozkaz wnijść do niego w imieniu najjaśniejszego cesarza Francuzów i spodziewam się, że mnie pan jenerał nie zechcesz zmuszać, abym przemocą sobie drogę otworzył." Kto z kim przeprowadził ten dialog? - Włodzimierz Potocki i Sievers

78. " -Jestem Lefebvre, jeden pozostały z rodziny, z której kilku braci moich, pułkowniku na tamten świat wyprawiłeś.
- Mój Pułkowniku, już bym teraz tak celnie nie potrafił strzelać: ręka drży i oko niedowidzi.
- Nie przychodzę tu wcale z zemstą, ale żebym cię poznał i zgodą serdeczną stare zatarł niesnaski."
Proszę o nazwisko drugiego pułkownika i odpowiedź na pytanie o co poprosił Francuz? - Był to Jakub Ferdynand Bogusławski, a płk Lefebvre odwiedził go w 1814 r., gdyż chciał poznać tego, kto w pojedynkach (w 1797 r. w Wenecji) wyprawił na tamten świat jego braci. A Francuz poprosił Bogusławskiego o pistolety, przy pomocy których zabił jego braci

79. Będąc już na św Helenie Cesarz wypowiedział się o pewnej koncepcji fortyfikacyjnej: "Ten system fortyfikacyjny wygląda jak narysowany przez oficera huzarów. Możliwe, że system Vaubana był słaby, ale był lepszy od tego."
Czyjej koncepcji dotyczyła ta krytyka? - Józefa Rogniata (1776-1840)

80. "Przed Grenoble byłem tylko awanturnikiem. Teraz jestem władcą!" Gdzie padły te słowa Napoleona? - Grenoble

_________________
"W polityce głupota nie stanowi przeszkody."
Napoleon


Góra
 Zobacz profil  
 
Wyświetl posty nie starsze niż:  Sortuj wg  
Utwórz nowy wątek Odpowiedz w wątku  [ Posty: 11 ]  Przejdź na stronę 1, 2  Następna strona

Strefa czasowa: UTC + 2 [ DST ]


Kto przegląda forum

Użytkownicy przeglądający ten dział: Brak zidentyfikowanych użytkowników i 21 gości


Nie możesz rozpoczynać nowych wątków
Nie możesz odpowiadać w wątkach
Nie możesz edytować swoich postów
Nie możesz usuwać swoich postów
Nie możesz dodawać załączników

Skocz do:  
cron
Powered by phpBB® Forum Software © phpBB Group
Przyjazne użytkownikom polskie wsparcie phpBB3 - phpBB3.PL